Jump to content

Wikipedia:Reference desk/Humanities: Difference between revisions

From Wikipedia, the free encyclopedia
Content deleted Content added
Line 602: Line 602:
Hi! I was reading our article on [[Adam Smith]], but i'm still not too sure on his political ideals, could anyone here state what his political ideas were or if he came up with any? All I can find as of now is some things about [[Political Economy]], am I missing anything? Thanks '''<font color="gold">★[[User:Retrolord|★]]</font>[[User:Retrolord|Retro]][[User talk:Retrolord|Lord]][[User:Retrolord|★]]<font color="gold">★</font>''' 10:44, 8 March 2013 (UTC)
Hi! I was reading our article on [[Adam Smith]], but i'm still not too sure on his political ideals, could anyone here state what his political ideas were or if he came up with any? All I can find as of now is some things about [[Political Economy]], am I missing anything? Thanks '''<font color="gold">★[[User:Retrolord|★]]</font>[[User:Retrolord|Retro]][[User talk:Retrolord|Lord]][[User:Retrolord|★]]<font color="gold">★</font>''' 10:44, 8 March 2013 (UTC)
:He wrote a whole book on morality, which contains ideas relevant to politics, and a multi-volume book on political economy, which contains a great many opinions on political issues. If you start reading ''The Wealth of Nations'' you will see how he has a view on many social issues of his time. In fact the book is largely about [[social policy]]. He critiques what he calls "the policy of Europe", especially England, where restrictive government legislation interfered with business, and he thinks, also interfered with people's freedom and ability to find work. [[User:Itsmejudith|Itsmejudith]] ([[User talk:Itsmejudith|talk]]) 12:14, 8 March 2013 (UTC)
:He wrote a whole book on morality, which contains ideas relevant to politics, and a multi-volume book on political economy, which contains a great many opinions on political issues. If you start reading ''The Wealth of Nations'' you will see how he has a view on many social issues of his time. In fact the book is largely about [[social policy]]. He critiques what he calls "the policy of Europe", especially England, where restrictive government legislation interfered with business, and he thinks, also interfered with people's freedom and ability to find work. [[User:Itsmejudith|Itsmejudith]] ([[User talk:Itsmejudith|talk]]) 12:14, 8 March 2013 (UTC)

== what are the causes of the rebellion? ==

what are the causes that led to the rebellion and the rebel alliance? what could the empire have done to prevent this? [[Special:Contributions/91.120.48.242|91.120.48.242]] ([[User talk:91.120.48.242|talk]]) 12:52, 8 March 2013 (UTC)

Revision as of 12:52, 8 March 2013

Welcome to the humanities section
of the Wikipedia reference desk.
Select a section:
Want a faster answer?

Main page: Help searching Wikipedia

   

How can I get my question answered?

  • Select the section of the desk that best fits the general topic of your question (see the navigation column to the right).
  • Post your question to only one section, providing a short header that gives the topic of your question.
  • Type '~~~~' (that is, four tilde characters) at the end – this signs and dates your contribution so we know who wrote what and when.
  • Don't post personal contact information – it will be removed. Any answers will be provided here.
  • Please be as specific as possible, and include all relevant context – the usefulness of answers may depend on the context.
  • Note:
    • We don't answer (and may remove) questions that require medical diagnosis or legal advice.
    • We don't answer requests for opinions, predictions or debate.
    • We don't do your homework for you, though we'll help you past the stuck point.
    • We don't conduct original research or provide a free source of ideas, but we'll help you find information you need.



How do I answer a question?

Main page: Wikipedia:Reference desk/Guidelines

  • The best answers address the question directly, and back up facts with wikilinks and links to sources. Do not edit others' comments and do not give any medical or legal advice.
See also:


March 3

Why are modern warships so much bigger than tanks?

Why are modern warships so much bigger than tanks? More specifically, I'll pose the question in both of the following ways:

  • Why not build giant tanks the size of modern warships? (I read the super-heavy tank article, but it pretty much just says that super-heavy tanks were found to be unnecessary without explaining why. My guess at the primary answer to the question was that militaries don't build giant tanks because they would then be giant easy targets, and the loss of each such giant tank would be an enormous loss in cost/resources, but going by that logic, militaries shouldn't be building any giant warships either, and yet modern militaries do just that, leading us to the next question below...)
  • Why not build a naval fleet out of tiny warships the size of modern tanks? (I can explain aircraft carrier sizes -- they need that length of runway for the airplanes to be able to take off -- but what about warships other than aircraft carriers?)

SeekingAnswers (reply) 04:30, 3 March 2013 (UTC)[reply]

1) For giant tanks: They couldn't drive on roads, or over bridges, due to their size and weight, and would tend to sink into soft ground, disabling them. They would also be far more visible, as you said, and cost more fuel per ability to deliver shells on target. Construction and maintenance would also be far more difficult, as it would require using cranes to lift parts a single person could lift before. StuRat (talk) 04:41, 3 March 2013 (UTC)[reply]
2) Tiny ships might be practical in the future. The largest ships today are aircraft carriers, and they need that size for the runways, in order to accelerate and decelerate a human safely from rest to flight speed. However, when we have all unmanned planes, they can be launched like missiles and caught in nets, allowing for much smaller ships. Or, it can just launch missiles and not worry about retrieving them. Humans can probably be eliminated at many other points, as well. Humans will probably always be needed to make the targeting decisions, but they don't necessarily have to be on-board, what with satellite communications and such. A maintenance staff is still needed on-board, though, unless we want to make the ships disposable, too. StuRat (talk) 04:48, 3 March 2013 (UTC)[reply]

Pretty much for the reasons Sturat stated. Also, most modern warships aren't that much bigger than their WW2ish equivalents. Look at USS Missouri or the planned Montana class, bigger than todays warships. The only class where there is a trend of getting bigger seems to be carriers, but that is a result of modern technology, ie, nuclear power and jet engines on aircraft. Hope this helps.04:44, 3 March 2013 (UTC)RetroLord

I'm not sure that you're strictly correct on that Retrolord; the new Zumwalt-class destroyer displaces 14,564 tons, while a WWII Brooklyn-class cruiser was only 9,767 tons. A WWII Fletcher-class destroyer was a mere 2,050 tons. There's a similar up-sizing in Royal Navy ships. However, you're right about big battleships - consider the Yamato-class battleships for instance. Alansplodge (talk) 13:01, 3 March 2013 (UTC)[reply]

YOu couldn't have a fleet of tiny warships because it would be unproductive. A modern warship might have a crew of 100. It might have a main gun, a radar station, a few missles and perhaps an AA gun. It will also have a full kitchen, long range fuel tanks, and the facilities for its entire crew. If you were to make 25 ships with 4 crew, no ship would have all these things. Meaning one ship is vulnerable to air attack, another to submarine attack and so on. It is just more efficient in terms of ship design to build a big ship with everything on it. RetroLord 04:48, 3 March 2013 (UTC)[reply]

If you made ships that small, you'd make them unmanned. Of course, such small ships could only go a short range, unless they each had a nuclear reactor. They could be deployed by a larger ship near a combat area, though, and then return (or not) when their supplies were exhausted. StuRat (talk) 05:03, 3 March 2013 (UTC)[reply]

Small ships wouldn't be able to make cross-ocean voyages, and it's unlikely we will be deploying tiny unmanned ships with nuclear reactors any time soon. And in todays world, you dont have some huge armada of ships battle it out firing artillery at each other for 12 hours like you might have had in WW2. Today its just a few missles and its over. Thats why one big ship> 100 small ships. RetroLord 05:11, 3 March 2013 (UTC)[reply]

Warships are much bigger than tanks, because they both serve very different purposes. A naval fleet's primary role is logistics - getting the boots on the ground, carrying the tanks and other vehicles, feeding the men, and, if necessary, naval bombardment. Tanks don't have that task. They are combat vehicles. All they have is a crew which is just enough to do the job, and enough rations of food and water to last them until the next lot arrives. KägeTorä - (影虎) (TALK) 06:28, 3 March 2013 (UTC)[reply]

Another reason is that tanks (other than Bolos) have to be able to maneuver around and take advantage of the terrain (e.g. defilade). The bigger the tank, the harder that is to do. Not much terrain out on the water. Clarityfiend (talk) 07:29, 3 March 2013 (UTC)[reply]

Also, the Maus is why we don't make huge tanks. KägeTorä - (影虎) (TALK) 08:19, 3 March 2013 (UTC)[reply]

One common political ideology

Québec political parties have common ideology: separation and Israeli political parties have Zionism. Is there any other areas or nations where their political parties have one common political ideology? — Preceding unsigned comment added by 65.95.104.127 (talk) 04:59, 3 March 2013 (UTC)[reply]

Your assumption is wrong. Zionism is not a common ideology for Israeli political parties. Hadash (Marxist) and Ale Yarok (classical liberal) are not Zionist parties. Israeli Arab parties such as United Arab List, Balad and Ta'al are anti-Zionist party. --PlanetEditor (talk) 05:18, 3 March 2013 (UTC)[reply]
To answer your question, all Indian political parties have one common ideology: socialism. The Constitution of India declares India a socialist state and to be a recognized party, it has to pledge alliance to socialism. Source. In Iran adherence to Islamic governance and law (Shia Islamic theocracy and Sharia) is must to gain recognition as political parties. See Political parties in Iran. In Bhutan, a political party has to pledge alliance to the monarch, all Bhutanese political parties are monarchist. --PlanetEditor (talk) 05:28, 3 March 2013 (UTC)[reply]
One comment: It's far easier for the various political parties to agree on one goal, when they aren't in power, especially if that goal is to get rid of whoever is in power now. It's once they achieve this goal, that the divisions tend to manifest themselves. For example, there were the Bolsheviks and the Mensheviks, which managed to tolerate each other while the Tsar was their common enemy. StuRat (talk) 05:50, 3 March 2013 (UTC)[reply]
And not all Quebec political parties support separation, see Politics of Quebec. CambridgeBayWeather (talk) 10:20, 3 March 2013 (UTC)[reply]
The two majority political parties in the US play like polar opposites, but they agree in their most basic goals- maintain cultural, military, economic dominance of the US worldwide, etc. The fact that they market themselves as being opposite ends of a "spectrum" (liberal vs conservative) distracts from the huge areas of agreement between them. Staecker (talk) 13:52, 3 March 2013 (UTC)[reply]
To be fair, they seem to disagree on things that Americans generally disagree on, like gun control, abortion, health care and so on. Further, they have their own agendas stretching beyond what they actually accomplish. IBE (talk) 15:37, 3 March 2013 (UTC)[reply]
maintain cultural, military, economic dominance of the US worldwide - [citation needed] RNealK (talk) 23:08, 3 March 2013 (UTC)[reply]

double loft bed reasonable?

a room currently has a sofa-bed in it but the room is really quite small. the sofa-bed does open for two, like this:

http://commons.wikimedia.org/wiki/File:Futon-america.jpg

and two do use it. the room is small. the question is, how reasonable would it be to try to use the same area for a loft bed of the same open size (2 person), so a desk can go under it? Would it be very expesnive - is this something a carpenter should do custom or an ikea-like solution is best? (or DIY)

basically i'd like to know if it's reasonable to try to put that same space up in the air. Also, I don't know how to ask in a different way, but, uh, can two people (a couple) "do whatever" if the ceiling is very high above anyway, meaning that a loft bed like that wouldn't make inordinate noise versus a grounded one or be a fundamnetally different/worse experience (except for needing to climb up)? Thanks. 86.101.32.82 (talk) 13:29, 3 March 2013 (UTC)[reply]

How high is the ceiling ? Getting a carpenter to custom build a solution will be quite expensive. An off-the-shelf loft bed should be far cheaper. Making it large enough for a couple might be problematic, though, as changing the sheets or flipping the mattress may be quite difficult. As for noise, I'd avoid bed springs, unless you have the ones where each is wrapped in fabric to prevent them from rubbing against each other. Wood can also squeak. Rubber feet should help keep noise down. Some type of hammock might be the quietest (provided it's not close enough to the wall to rock against it) and cheapest, although it's difficult to get proper back support from those. (You can place a mattress on top of the hammock.)
Something else to consider is some type of fold-up desk. This would be good if you just need a writing surface, but don't need to store lots of stuff inside. StuRat (talk) 17:22, 3 March 2013 (UTC)[reply]
Thanks. 86.101.32.82 (talk) 21:36, 3 March 2013 (UTC)[reply]
Ikea sells a full/double loft bed for $169 US: http://www.ikea.com/us/en/catalog/products/50161830/ The recommend a minimum ceiling height of 8 feet 10 inches (269 cm). The bottom of the bed platform is 64 5/8 inches (164cm) from the floor, but you should be able to position the desk so you are not standing up under the bed and hitting your head. Edison (talk) 02:04, 4 March 2013 (UTC)[reply]
They also sell a loft bed with a desktop underneath. No drawers, though: [1]. And they can sell the desktop and shelf separately: [2]. However, I think a freestanding desk would be a better idea, as you don't want "vibrations" from the bed knocking things off the desk and shelves. StuRat (talk) 02:13, 4 March 2013 (UTC)[reply]

rhetorical device on person idea or idealogy

write a passage using rhetorical device on person idea or idealogy?--106.198.236.214 (talk) 15:03, 3 March 2013 (UTC)--106.198.236.214 (talk) 15:03, 3 March 2013 (UTC) sarita 3rd march 2013[reply]

I shall call my essay, "Can you not do your own homework?". When is it due? IBE (talk) 15:40, 3 March 2013 (UTC)[reply]
While I agree with IBE's basic sentiment, the OP might find Scheme (linguistics), Trope (literature) and Figure of speech useful. Tevildo (talk) 15:46, 3 March 2013 (UTC)[reply]
Actually that's a better way of doing it. I just get a little peeved by the tone of some of these homework q's. Also, rhetorical device might be good. IBE (talk) 17:19, 3 March 2013 (UTC)[reply]
I wouldn't take the tone personally: someone asking here may well not understand yet how to properly form the questions they need to answer (which is half the problem), and many are not really aware that they are addressing actual people rather than a search engine. Consider how this question was formatted: I would take this as someone writing down what they need to do, then adding a question mark to indicate that they don't understand and need help. All we can do is point the person to general resources, and then if they ask something more specific we can help with that. It is usually better to post nothing, if the alternative is to get cross or sarcastic with the person asking.
Sarita, what is it that you need help with? Is it that you don't understand what to do? Is it that you don't know what a rhetorical device is? If you can explain your problem more clearly, by clicking the word edit written in blue to the right of your question, we can help you more. If you're completely stuck and don't understand it at all, you might do better explaining this to your teacher and asking for more help from them. 86.140.54.54 (talk) 17:32, 3 March 2013 (UTC)[reply]


yeah sir actually i dont understand how to write this passage.

Ancient Korean script

Hello,

I've seen several videos about Gojoseon, some say it had a script. Is this right? The dolmens of Gojoseon were pointed out to have astronomical drawings.

Greetings HeliosX (talk) 18:53, 3 March 2013 (UTC)[reply]

Somali British under which ethnic group category

Which ethnic group category would Somali British fall under?--Donmust90 (talk) 19:19, 3 March 2013 (UTC)Donmust90[reply]

People self-classify, so they can put what they like, but "Black or Black British: African" would be a logical choice. Some may prefer "Black or Black British: Other", or just "Other". Itsmejudith (talk) 19:35, 3 March 2013 (UTC)[reply]

why is the Independent allowed to publish racist propaganda? isn't it hate speech?

I do not want to reference a specific article but I have to just for you to see what I'm talking about. The Independent has just opened an article with this highly inflammatory sentence: "Israel will open new “Palestinian bus routes” in the West Bank tomorrow". Why is it allowed to say that? What recourse would the victims of such racist reporting have? Can they say just whatever they want about Israel? This does not seem to me at all balanced. 86.101.32.82 (talk) 22:04, 3 March 2013 (UTC)[reply]

See Hate speech laws in the United Kingdom for the laws that would be applicable, and Article 10 of the European Convention on Human Rights for why they're allowed to publish it. However, that headline would be _very_ unlikely to count as "incitement to racial hatred". The Press Complaints Commission would be the first port of call for anyone who was genuinely offended. Tevildo (talk) 22:33, 3 March 2013 (UTC)[reply]
The fact that the paper puts the words "Palestinian bus routes" in quotation marks is what protects them from any accusations of unbalanced or inflammatory reporting. It means that they're not using those words themselves, they're just reporting the words of others. --Viennese Waltz 22:44, 3 March 2013 (UTC)[reply]
What exactly is racist about that ? Presumably most people using bus routes between Israel and Palestine would be Palestinian (since many Palestinians live in the West Bank and work in Israel). I suppose it's not the best choice of words, as some non-Palestinians will use the bus routes, too, but that's not exactly being racist. StuRat (talk) 22:50, 3 March 2013 (UTC)[reply]
I'm with Stu. What's racist about it? HiLo48 (talk) 22:55, 3 March 2013 (UTC)[reply]
It quotes people using terms like 'apartheid' (right in the title). that is in no way neutral and slanders israel. this is clear isn't it? the question is about the legalities of publishing that. 86.101.32.82 (talk) 01:49, 4 March 2013 (UTC)[reply]
I don't see the term 'apartheid', or anything like it, in what you posted. StuRat (talk) 02:01, 4 March 2013 (UTC)[reply]
Right, the posts by the IP are very unclear. It's impossible to know what they refer to without context, either from the article [3] or from other sources about the bus routes. PrimeHunter (talk) 02:31, 4 March 2013 (UTC)[reply]
Just to be clear, Israel is absolutely an apartheid state as long as we're using actual dictionaries for all those words. People of particular ethnic groups are intentionally separated from others. Morever, it's not hate speech to direct criticism at a state, doing so at the Israeli people might be, but the government doesn't gain some immunity from criticism because their people do. i kan reed (talk) 21:15, 5 March 2013 (UTC)[reply]
Yes, just to be clear, let's stay on "A" in actual dictionaries and look up absolutely. There are roads (contested) and these buses in the West Bank that are segregated, and the well-reported "Separation Fence" Those doesn't constitute the state of Israel. Applying definitions indiscriminately while claiming to be absolutely clear isn't "criticism," but something like "defamation". -- Deborahjay (talk) 21:18, 6 March 2013 (UTC)[reply]
Even if some non-Palestinians will use the bus routes, the fact that they connect to Palestinian territories makes the name accurate. As for "can they say just whatever they want about Israel?", yes, to a very large extent. There's no law requiring news agencies to be "balanced", and any such law would be a gross violation of the freedom of speech. --140.180.251.41 (talk) 22:58, 3 March 2013 (UTC)[reply]
This is a great example of why the First Amendment is so prized by Americans... because the most innocuous statement can be found "racist" by some people... or a few decades ago, "communist," or whatever is out of fashion at the moment. Allowing other people to say things isn't an endorsement that they're right, just that you don't have a right to silence them. Good ideas win on the merits. Shadowjams (talk) 07:18, 4 March 2013 (UTC)[reply]
Gee, I wish that was always true. HiLo48 (talk) 07:37, 4 March 2013 (UTC)[reply]
It's either they win on the merits or else government enforces them at the point of a gun... in which case you're trusting government to be right... there's no magic formula for getting things right, but there are some obvious ones for getting it wrong. Shadowjams (talk) 07:59, 4 March 2013 (UTC)[reply]
Sorry. My comment came as an Australian who sometimes criticises some behaviour of some Americans. I have found the attitude of some of those American people to the freedom of speech of those who disagree with them to be virtually non-existent. But that's got nothing to do with this thread. HiLo48 (talk) 10:28, 4 March 2013 (UTC)[reply]
Ya I know :) Shadowjams (talk) 22:35, 4 March 2013 (UTC)[reply]
I would like the OP to explain what is racist. To me, "Mancunian bus routes" are bus routes that link with Manchester. "Luxemburghese bus routes" are bus routes that link with Luxembourg. "Palestinian bus routes" are bus routes that link with Palestine. --Lgriot (talk) 09:21, 4 March 2013 (UTC)[reply]
That's exactly how I read it. Where is the problem? HiLo48 (talk) 10:28, 4 March 2013 (UTC)[reply]
Israel is a country, not a race. Perhaps Jews is what you were equating Israel with? As far as I know there is no anti-countrist legislation. If the newspapers of one country weren't able to talk about other countries and their perceived failings that would certainly cut down on the verbiage somewhat! Dmcq (talk) 10:52, 4 March 2013 (UTC)[reply]
The OP claims something is highly inflammatory. Who says so, besides the OP? Is there a valid source on the internet that makes that claim? ←Baseball Bugs What's up, Doc? carrots→ 11:52, 4 March 2013 (UTC)[reply]
In today's news in Israel (print and broadcast media) is a story about a proposed bus line in the West Bank whose passengers will be Palestinian Arabs exclusively. There are three daily English-language mainstream newspapers in Israel: Haaretz, the Jerusalem Post, and YnetNews, the electronic edition of the Hebrew Yedioth Aharonoth. Read what they published (on the Web), as this complies with Israel's laws. How the foreign press reports stories for its readers presumably reflects their laws and readership, such that the term "apartheid" may be a matter of commentary rather than reportage. -- Deborahjay (talk) 12:59, 4 March 2013 (UTC)[reply]
But where is the term "apartheid"? This is a nonsense thread right now. HiLo48 (talk) 21:49, 4 March 2013 (UTC)[reply]
It's right there in the headline of the article. PrimeHunter linked to it upthread. --Viennese Waltz 21:54, 4 March 2013 (UTC)[reply]
That would have to be the most hidden thing to complain about that I've (only just now) seen in some time. Why on earth isn't the thing we're complaining about displayed here? To save others the trouble of searching, I'll do it now. The Independent's headline says "Israel’s Palestinian-only buses prompt apartheid comparisons". I still don't know what the problem is. HiLo48 (talk) 22:07, 4 March 2013 (UTC)[reply]
Ynetnews: "Ministry launches 'Palestinians only' buses."
Haaretz: " Israel introduces 'Palestinian only' bus lines, following complaints from Jewish settlers."
Jersusalem Post: "Palestinians-only bus lines open in West Bank"
How the Israeli press report news is pretty similar to how the English press reports news. I don't see in these cases any hint of inflammatory comments.OsmanRF34 (talk) 14:18, 4 March 2013 (UTC)[reply]
Agreed. While the attempt to segregate Palestinians from Israelis by bus route may itself be considered discriminatory, the Independent's reporting on the event seems quite fair, to me. StuRat (talk) 17:09, 4 March 2013 (UTC)[reply]
Is it possible the Palestinians themselves asked for this segregated bus service? ←Baseball Bugs What's up, Doc? carrots→ 14:11, 6 March 2013 (UTC)[reply]
Possible, but the article didn't make that claim. StuRat (talk) 16:18, 6 March 2013 (UTC)[reply]

Where to buy cocaine?

Where can a person buy cocaine, amphetamines, and heroin? In what state of the United States can a person buy these things? How much do they cost? How much does a buyer typically consume per day? How old do you have to be, and under what conditions can you buy these drugs? Sneazy (talk) 22:10, 3 March 2013 (UTC)[reply]

Assuming this isn't a troll question, certain types of phenethylamine drugs (commonly known as "amphetamines") are available by prescription. Cocaine is occasionally still used as an anesthetic by hospitals during surgical procedures, but it is otherwise illegal to possess, much like heroin. Evanh2008 (talk|contribs) 22:23, 3 March 2013 (UTC)[reply]
I'm also going to assume that this isn't a trolling question - in particular with regards to street drug pricing I can think of one very good reason for wanting to know street drug pricing: a way to estimate changes in supply and demand. For instance, an increase in the price of a drug means either:
  • a decrease in supply e.g. due to law-enforcement being more effective
  • an increase in demand e.g. due to (anti-)drug education not being less affective
Bearing this in mind, this site has some useful info, but its UK-centric I'm afraid
Many thanks, davidprior t/c 23:23, 6 March 2013 (UTC)[reply]
  • Per this I can tell you that a kilo of heroin costing 35,000 dollars in Mexico costs $71,000 once it crosses the border and can cost up to $131,000 in some large cities. Low quality marijuana from Mexico costs $80 per kilo there and $2000 per kilo here. Pure cocaine costs $120,000 per kilo in America. Ryan Vesey 18:46, 7 March 2013 (UTC)[reply]

james lasun and an internet vandetta

I don't understand james lasun's story about an iranian writer with a vandetta against him. as i saw reviewed, this wasn't one or two cases but a whole campaign. surely this is so obviously cut and dried stalking/slander/libel that after the first few times he could simply say, look, stop stalking me, stop talking trash about me, and threaten (with good cause) to sue. then she would have found better things to do with her time. could you explain this to me. Note: I disagree with the edit saying this is inappropriate. I would like to see references or other information that explains this. 86.101.32.82 (talk) 22:45, 3 March 2013 (UTC)[reply]

Googling James Lasun doesn't return anything that seems relevant to this. The closest article seems to be James Lasdun. Is that who you meant? Even there, I don't see anything about a vendetta. Can you point us at the story you're referring to? Rojomoke (talk) 13:10, 4 March 2013 (UTC)[reply]

How to become a Quaker?

How does a person become a Quaker in a non-Quaker community or Quaker upbringing? Sneazy (talk) 22:54, 3 March 2013 (UTC)[reply]

I doubt there are very many Quakers here, but if you check the links in Quakers, there appear to be a number of sites that you could probably consult and/or contact, and get a good answer. ←Baseball Bugs What's up, Doc? carrots→ 23:08, 3 March 2013 (UTC)[reply]
They don't shun the outside world, so you'll find their meeting halls in many places. Try this site to find one near you, if you're in the US or Canada: [4]. StuRat (talk) 23:19, 3 March 2013 (UTC)[reply]
Go to a Meeting House. In the UK at least there are no Quaker communities, just Quaker believers. --TammyMoet (talk) 10:09, 4 March 2013 (UTC)[reply]
Actually there are several residential Quaker communities in the UK (depending on what you mean by "community"), and plenty of Quakers editing Wikipedia I might add, but the place to start would be your local Quaker meeting house. You would be expected to attend the local meeting regularly for a year or two before being considered for membership.--Shantavira|feed me 11:30, 4 March 2013 (UTC) (former Quaker)[reply]


March 4

Menachem Begin--PM Of Isreal 1977-1983

I am trying to find a cartoon showing Menachem Begin Showing what the United states would look like if the U.S. gave back all the land annexed after each war. Can you help me out — Preceding unsigned comment added by 184.32.137.46 (talk) 00:47, 4 March 2013 (UTC)[reply]

This is the original 1977 version. Tevildo (talk) 00:53, 4 March 2013 (UTC)[reply]
A little off-topic, but I never saw this cartoon before. It is really funny and insightful. Futurist110 (talk) 02:50, 4 March 2013 (UTC)[reply]
Cute. Now, if everyone gave up land that they or their ancestors had confiscated, all 6 billion or so of us would wind up living in our original African village. ←Baseball Bugs What's up, Doc? carrots→ 04:02, 4 March 2013 (UTC)[reply]
Only if you count land we stole from Neanderthals and others. Of course, restoring the original ethnic group to each chunk of land would be problematic, as most of those ethnic groups no longer exist. Perhaps those on some isolated islands are the original ethnic groups. So those on Easter Island can stay. The rest of us have to leave. StuRat (talk) 04:08, 4 March 2013 (UTC) [reply]
What, and invade outer space? Wretched imperialist! הסרפד (call me Hasirpad) (formerly R——bo) 00:06, 5 March 2013 (UTC)[reply]

Fulgencio Batista

ERROR - NEEDS CORRECTION : In the page on Fulgencio Batista, the term used (bohio) for the word slum listed under a photograph is incorrect. The word bohio refers to the mud-floored thatched roof dwellings used by farmers. The term for slum used in the 1950's in Havana was solar with the accent on the 'lar'. I am sure of this. The photograph is the fourth photo on the English page. Thanks — Preceding unsigned comment added by Lolablanche (talkcontribs) 01:22, 4 March 2013 (UTC)[reply]

You can either edit the article yourself to make the correction, of ask someone else to do so on the talk page for that article: Talk:Fulgencio_Batista. StuRat (talk) 01:24, 4 March 2013 (UTC)[reply]
Be bold! If you are sure, why not just edit it yourself, afterall, it is for everyone! (If you have a reference it would help, maybe add it in your edit summary?) Kinkreet~♥moshi moshi♥~ 01:30, 4 March 2013 (UTC)[reply]

Countries with large-scale historical immigration

Which countries had large-scale immigration both in the decades before World War I and in the last several several decades (also, other time periods can be included if you want)? I can think of the United States of America, Canada, Australia, New Zealand, and maybe Israel/Palestine, but which countries am I missing/forgetting? I know that several countries in South America/Latin America had large-scale immigration (Argentina, Uruguay, perhaps Chile and Brazil) before World War I, but probably not in the last several decades. Futurist110 (talk) 02:53, 4 March 2013 (UTC)[reply]

Depends on what you mean by "large-scale", but the UK/British Isles has always had immigration, be it conquerors (Romans, Angles, Saxons, Danes, Normans), refugees (Huguenots, Jews, Vietnamese, Bosnians) or invitees (Commonwealth nations). --TammyMoet (talk) 10:07, 4 March 2013 (UTC)[reply]
Scots (from Ireland) and Jutes. Sleigh (talk) 10:22, 4 March 2013 (UTC)[reply]
France, I think, although I would have to double-check for the decades before WW1. Has had a lot of immigration from Italy, Spain, and from Poland to work in the coalmines of the north, followed by post WW2 immigration from various places. Itsmejudith (talk) 17:42, 4 March 2013 (UTC)[reply]
By "large-scale", I mean countries which had a foreign-born population as 10.00% or more of their total population at some point before World War I and again today. The United States of America would work for this -- History_of_immigration_to_the_United_States#Historical_foreign-born_population_by_state. Canada would likewise work -- http://www4.hrsdc.gc.ca/auto/diagramme-chart/stg2/c_6_38_1_2_eng.png?20121002195953815. The United Kingdom, though, would not work -- https://docs.google.com/viewer?a=v&q=cache:oeo6ObGsZzAJ:www.unece.org/fileadmin/DAM/stats/documents/ece/ces/ge.10/2012/Presentations/1__WP4__Measuring_Migration_in_the_UK_Census_Ian_White.ppt+u.k.+foreign-born+population+1841-2001&hl=en&gl=us&pid=bl&srcid=ADGEESiVfNoNx3K7vo_YlemzZhLSsce3H0uM_YhiVY9jHpjO-n3mv9VUM7GiNoUUAsKs8j_Be54KvwHauNIcgBEIPAXQ6noBEHp49xUwQV6uqBqgOFRyA27Ppyh8AQAb5KFPrsj6JyZI&sig=AHIEtbRa7FfjNj4q3IvpAsXWXbRPOo-xJw. Argentina would likewise not work for this. As for France, I don't have the historical foreign-born data for France with me right now, at least not yet. Futurist110 (talk) 22:02, 4 March 2013 (UTC)[reply]

Hong Kong fits the bill. DOR (HK) (talk) 03:19, 5 March 2013 (UTC)[reply]

Perhaps, but it's not a (separate) country, and thus unfortunately it doesn't count for the purposes of answering my question. Futurist110 (talk) 03:30, 5 March 2013 (UTC)[reply]
Is it possible to migrate to Hong Kong without finding you've migrated to China? -- Jack of Oz [Talk] 06:21, 5 March 2013 (UTC)[reply]
Hong Kong was only returned to China recently so it depends on how limited the OP's "last several decades" are. Rmhermen (talk) 15:18, 5 March 2013 (UTC)[reply]
Hong Kong still wasn't its own country before being returned to China in 1997, so unfortunately it does not fit the bill for my question. Futurist110 (talk) 05:35, 6 March 2013 (UTC)[reply]

Name the white stuff?

What is the name of the white square-shaped thing on the collar of a parish priest who dresses himself mostly in black? Sneazy (talk) 04:38, 4 March 2013 (UTC)[reply]

Clerical collar. --OnoremDil 04:53, 4 March 2013 (UTC)[reply]

German World War II films showing their surrender to the Western allies?

You see, Der Untergang was a German film that depicted their surrender to the Soviets. I would have felt more delighted seeing a scene of their surrendering to the Americans, British or French forces. There were plenty of newsreel footages, and some films that were American or British-made, that I already saw.

However, I have yet to find a single German-made film showing their forces in WW2 surrendering to / being captured by the Americans, or the British, or the French.

What such films would depict this? If you link it to any video hosting site, will they have captions / subtitles? Thanks. --70.179.161.230 (talk) 08:35, 4 March 2013 (UTC)[reply]

We cannot. Linking to a video hosting site would be a violation of copyright. KägeTorä - (影虎) (TALK) 09:24, 4 March 2013 (UTC)[reply]
Would that necessarily be true? Surely some of the films made shortly after the war are out of copyright by now? Or are we merely close, but not there yet? 46.30.55.66 (talk) 10:18, 4 March 2013 (UTC)[reply]
No assumptions can be made about copyrights that aren't stated. But you could use Google as well as we could. :) ←Baseball Bugs What's up, Doc? carrots→ 11:13, 4 March 2013 (UTC)[reply]
Films made shortly after the war would be out of copyright by now, of course, but if it is a video hosting site, they would surely have others, completely unrelated, which are not. KägeTorä - (影虎) (TALK) 13:46, 4 March 2013 (UTC)[reply]
Hosting films is not always illegal. There is also pay per view systems and archive.org also hosts videos, which appear to have no copyright issues. OsmanRF34 (talk) 14:08, 4 March 2013 (UTC)[reply]
This discussion has gotten far off track; all the OP is asking about is whether there are such films. Nobody asked about obtaining an illegal copy. So ...

... let's try to answer the OP's question: There are many such movies. One that readily comes to mind is Die Brücke (film). — Sebastian 18:59, 4 March 2013 (UTC)[reply]

Actually, that's not about surrender, but about defeat. I guess for surrender proper you would have to look at the leaders, hence the focus on the Führerbunker. — Sebastian 19:02, 4 March 2013 (UTC)[reply]
It doesn't have to be about the surrender made by high command in Berlin. It can show Jödl signing the unconditional surrender document in Rheims, or it can show a platoon / company / battalion / regiment of common soldiers deciding to surrender to the Allies after a drawn-out battle. What films fit this description? --70.179.161.230 (talk) 19:27, 4 March 2013 (UTC)[reply]
Are you referring to Alfred Jodl (no umlaut)? -- Jack of Oz [Talk] 06:19, 5 March 2013 (UTC)[reply]
Exactly. I thought the way his name was pronounced had it have an umlaut. Would it sound identical? But anyhow, what German films show a depiction of Jodl signing the surrender document? And as before, if anyone happens to know, what such films show common soldiers (and maybe the insane, guiltless Waffen SS units) making a surrender? Thanks again. --70.179.161.230 (talk) 08:11, 5 March 2013 (UTC)[reply]
Have you looked on YouTube? There are many such videos available in English (mainly WW2-era news reels), and I'm sure that there are German translations available. It's unlikely that any German cameramen recorded German soldiers surrendering for fairly obvious reasons (eg, they would have been too busy surrendering themselves, and the formal surrender ceremonies were held on Allied-controlled territory). The Waffen SS was anything but 'guiltless' BTW: Waffen-SS#War crimes. Nick-D (talk) 10:49, 5 March 2013 (UTC)[reply]
Well if there were no German-made newsreels showing their own surrender (what WAS the last Die Deutsche Wochenschau newsreel, and what'd that show?) then how about German-made films made years or decades later depicting this? Again, I have looked all over YouTube, and they're so hard to find, Nick-D. By the way, when I said "guiltless," I meant that they must've felt no guilt, no matter how heinous their actions were. --129.130.18.100 (talk) 18:18, 5 March 2013 (UTC)[reply]

Bartizan

A British concrete position, built at the North-Western corner of Sergei courtyard (Russian Compound), Jerusalem. This is probably the sole existing testimony of the British "Bevingrad" constructed in 1946.

Can this position be regarded as a Bartizan? Thanks - Etan J. Tal(talk) 08:49, 4 March 2013 (UTC)[reply]

I would say yes - albeit a modern version. It seems to have a small bulletproof glass window (instead of arrow slits) which would suggest a fortified building. I would not call it this: http://en.wikipedia.org/wiki/Oriel_window or this: http://en.wikipedia.org/wiki/Bay_window 196.214.78.114 (talk) 11:58, 4 March 2013 (UTC)[reply]
I suspect the glass is a later addition, but who knows for sure? Alansplodge (talk) 13:09, 4 March 2013 (UTC)[reply]
The British troops would most likely have called it a Pillbox; I know that they are usually sunk into the ground, but there are some pillboxes on stilts in London, to allow fire over a railway embankment. In more modern usage, it would be a sangar. Alansplodge (talk) 13:06, 4 March 2013 (UTC)[reply]
Thank you all. It seems Bartizan is still the best term. Etan J. Tal(talk) 19:26, 4 March 2013 (UTC)[reply]

is this presentation flawed due to not considering negative net worth?

In viewing this,

https://www.youtube.com/watch?v=QPKKQnijnsM&feature=player_embedded

I was continually asking myself where all the negative net-worth people are, which includes many a law student at the start of their career, medical students, indeed lots of people who owe more than they're worth?

Doesn't that completely skew "wealth"? For example, if the 'lowest wealth' 100 people incldue 10 people worth -80,000 and the rest just have between $7-$12, averaging $9k, then the average will be (90*9000 + 10* -80000 )/100 = $100. So if the "drastic poverty" were defined as having less than $5000 to your name, then 90% of our sample wouldn't qualify (have $7k-$12k wealth) while the remaining 10% actually will have income that lets them live fine despite their negative net worth.

So isn't a chart like that fundamentally wrong?

I don't know what the solution is - whether it would be showing "income", as anyone whose wealth/net worth is millions of dollars doesn't need any income at all to obviously not be poor.

But at the least, shouldn't a chart like that somehow differentiate between debt and wealth, or between wealth and income?

I'm not sure how to do it. Maybe it should instead show expenditure - how much people actually spend. Then rich people spending from their wealth would show up at the level they're spending at (which is obviously reflective of their quality of life) perhaps with an allowance for anything they enjoy the use of without having to spend on it, meanwhile people in debt can be shown based on how much they spend even if it is extra debt, and people on benefits can have the benefits calculated the same way as we try to calculate anything the rich get use of that they don't directly pay for as expenditure?

What would such a chart look like? 91.120.48.242 (talk) 10:38, 4 March 2013 (UTC)[reply]

The graph is based on this which in turn is based on this paper.
Quoting from the paper: "Wealth, also known as net worth, is defined as the total value of everything someone owns minus any debt that he or she owes. A person’s net worth includes his or her bank account savings plus the value of other things such as property, stocks, bonds, art, collections, etc., minus the value of things like loans and mortgages."
The graph does not consider income, which is fine considering it's only trying to show wealth distribution. As for the temporary negative net-worth individual I don't think there are *that* many of them to skew the graph. (no reference sorry too lazy)
P.S the last part in the video about CEO earning 380X times more than your average worker (see CNN link) does not match the data given in the Motherjones link (185X).
Here are the four links given at the end of the video Motherjones Dan Ariely blog Thinkprogress CNN Royor (talk) 12:03, 4 March 2013 (UTC)[reply]
I don't understand, I do think there are ton of people with negative net worth. I mean, imagine paying off a mortgage. You take on a thirty-year mortgage and take over the property: bam, you are now worth -$140,000 or whatever. Add back in your down-payment, and add in your payment every month, and eventually you are net-worth positive. Same if your home accrues in value for some reason, beyond what you owe on it. I just think that this kind of net-worth analysis is INCREDIBLY suspect if you consider rich people's owned networth but you don't consider anyone's debt. There's another thing too: the quality of living shows an incredible amount of diminishing returns on wealth. If in a few minutes I will become either someone with an annual income of $15,000 or an annual income of $2.5million by a roll of the die, but I can choose now whether to increase the $15,000 by another $7,000 if that's the one that comes up (no change on the other one) or increase the $2.5 million by another $120,000 if that's the one that comes up (no change if I draw the $15k) then hands-down absolutely no contest it is in my interest to balance out the $15k one. This is proof positive that $7k is worth WAY more at the $15k level than EVEN $120K (let alone $7k) at the $2.5m level. There is just no contest. So the graph that I would REALLY like to see is one that shows how much UTILITY the people have access to in America. Hands-down I absolutely agree that a person who goes from making $1.7million per year to $25 million per year would thereby experience a hugely difference in lifestyle. But that difference is logarithmic. If you want to show unfairness, it's not enough to show how much more money that guy has than the guy making $8k. I think not including debt is a particular skew because it makes it seem (as in my example) as if everyone who utilizes debt heavily - i.e. rich people - are even richer. Truly poor people just don't have access to that much debt. 91.120.48.242 (talk) 12:29, 4 March 2013 (UTC)[reply]
If you take on a mortgage you are not worth -$140,000. A mortgage is a liability balanced by an asset, the real estate. Unless you do it after a speculative bubble has burst and the real estate prices have plummeted, I don't see how that could happen. And even then, you could give back the keys and pass the loss to the financial backer. OsmanRF34 (talk) 14:32, 4 March 2013 (UTC)[reply]
You are right, you are not worth: -$140,000 the moment you sign a $140,000 mortgage. I did mention that if the real value of the home goes up (e.g. there is a huge explosive bubble two weeks after you sign, the house triples in value, you're a millionaire) then your net worth quickly goes up.
But before that happens ( :) ), the minute you buy it, you have just committed to paying a huge interest rate on it for a long long time: the value of the home at that moment MUST be less than value + interest rate! So the moment you commit to buy real estate, you very much do take on very significant debt. As an example, according to this site: http://michaelbluejay.com/house/interest.html a thirty-year loans shows Total Paid over 30 years at $358,971 and Total Interest at $208,971 - that shows that nearly two thirds of what you've committed to paying is in interest!! Is it really fair to say that that is not negative net worth in terms of net present value? The only way to say that is NOT negative net present value is if you say that your net present value includes the "built-in" appreciation of your house's value. But by that token, even a mdeical student with -$200,000 to their name after college + med school is not in debt, as their net present value includes their future salary. I think that's silly. Nobody is obliged to give you a salary, and nobody is obliged to make you a higher offer on your home later: but you are very much obliged to pay the interest rate. The only reasonable way to view a mortgage is as a huge amount of debt for the chance to live in the place you will own once you're finished paying for it, and perhaps for some chance to sell it to someone else later under potentially similar terms. 91.120.48.242 (talk) 15:20, 4 March 2013 (UTC)[reply]
Yet another proof it's debt: suppose you pay the monthly payments for a year or two, which, as the article indicates, are nearly 100% going toward interest rate (i.e. not really building any equity to speak of). Then you can't pay anymore and the home is foreclosed on. In this case: what did your monthly payments go toward? Were you paying rent? No. You were paying off your DEBT. Your house doesn't have to depreciate in value if you pay interest on it for two years and then are left with nothing. This proves that you were merely reducing your debt and doing nothing else during that time. There is no other way to view it. 91.120.48.242 (talk) 15:23, 4 March 2013 (UTC)[reply]


I still don't believe you have automatically into negative net worth when you buy real estate backed by a mortgage. Real estate generates income not just by prices going up. You can also rent your real estate and obtain something equivalent to the interest paid. In both cases, rent and loans paid, you are paying for using something that doesn't belong to you. Real estate should be treated as an asset that can be a good investment option or not, like every other option. BTW, you do not commit to pay an interest for x years just because you have a mortgage. A house, like any asset, can be sold, rent, and even given back. Even if you committed to paying interest for 30 years, you will also have a house to obtain rent from it for 30 years. And it doesn't matter if you live in it or rent it. It is providing something of value (shelter) for 30 or more years. In a good accountancy practice you'll have then liabilities for x $100,000 for 30 years and an asset, which will be valued in its potential for generating income through rent or being resold. Obviously, the bubble and burst of it were not 'normal' times. Valuating real estate and mortgages was a mess. OsmanRF34 (talk) 15:41, 4 March 2013 (UTC)[reply]
You can do all the funny accounting you want. For example, the medical student already gets to rent themselves out doing some kind of work that generates income thanks to the education they got that got them into that high debt. It's still debt! Saying "you didn't take ownership of the house - you're just using it" is so silly. Why do you even have to sign a mortgage then. Why do you have to find a seller, since you're really just using a bank's house? If all it is is a "usage right" how could you sell the underlying house, if it's just the bank? You can add all these "rights", like right to sell, right to use, and so on, while somehow claiming that you are "paying for using something that doesn't belong to you." You can buy a house from me, that I didn't own, oh no, I was just paying for the right to use it. And rent it out. And sell it to you. This is very silly, I hope you will agree. You are free to internally in your own mind think of it that way... but to say that this is how people should do their accounting is rather silly. In accounting terms, the fact that you have all these rights - pretty much anything you want. What if you destroy your house? Or neglect it? Do you not have the right to do certain negative renovations or neglect, since it's not your property, you are just paying to use it? If you sign a mortgage and then start making payments but neglect it, will the bank come and maintain your house, since it's theirs, you're just paying to use it? Silliness. Fact is, anyone with a mortgage they haven't put significant equity into, or which hasn't gone up in market value enough to cover their debt on it, is thereby in debt. 91.120.48.242 (talk) 15:53, 4 March 2013 (UTC)[reply]
  • Arghh. Nope 91.120, you are definitely not right. My accounting is certainly not 'silly.' You are confusing several concepts: the first, and fatal for your argument, is confusing being in debt to being indebted. The only way of reaching a negative net worth is when assets are smaller than equity. But, at least in the US and in other countries like the UK or Canada or Ireland, you can simply restructure your debt giving the keys back to the bank, if you believe you won't need a loan in the next years and the house is less valuable than the mortgage. I see, however, that your IP locates to Hungary, which was heavily hit by the bubble bursting, and it's, sorry for the pun, hungry now for growth. But yes, at the moment of signing the mortgage, the mortgage equal the asset, more or less. One drags your finances down, the other up. It's all balances, which doesn't mean you did a good business. OsmanRF34 (talk) 18:39, 4 March 2013 (UTC)[reply]
Incoming wall of text alert! First we need to go over some accounting basic.
US Census Bureau define household net worth as Asset minus liability. If you are familiar with accounting 101 Asset = Liability + equity. The two side MUST equal.
Suppose you brought a house for $100,000 (asset - book value), 20% down payment - $20,000 (equity) and took out a 80% mortgage - $80,000 (liability).
Your net worth is asset - liability = $100,000 - $80,000. You are worth $20,000.
Now let's say 1 years later you paid a total of $18,000, $10,000 toward principle and $8,000 in interest. Your house is now assessed at $150,000 (from your property tax assessment), what's your net worth?
Your asset is $100,000 + $8,000 (asset - book value), $20,000 + $18,000 = $38,000 (equity), remaining mortgage = $70,000.
Net worth based on book value = $108,000 - $70,000 = $38,000
Net worth based on market value = $150,000 - $70,000 = $80,000
Notice how you CAN NOT just take liability and calculate a negative net worth. Your asset's book value is based on how much it COST (interest is part of how much it COST so it's added to book value) and your asset market value will fluctuate based on the market.
Normally a car will depreciate in value over time and historically, the average home’s value has increased in value over time (not for the last couple year though). This depreciation/appreciation is reflected by the market value.
Now that's out of the way, for the average household their net worth is POSITIVE after a home purchase (down payment), whether that equity is liquid or not is another story. As for your indebted law/medical student they are a relatively small number compare to the entire US population. So in conclusion the argument about debt is not being accounting for is void and null, and the number of student with negative net worth is not large enough to make a difference to the graph. If you think net worth data is insufficient you can always look at it along with the income distribution data too.
Edit: The debt is already accounted for as part of liability. Royor (talk) 17:22, 4 March 2013 (UTC)[reply]
The average U.S. college student has $26,000 in debt and no/few real assets. I think negative worth is probably pretty common there. Rmhermen (talk) 15:13, 5 March 2013 (UTC)[reply]
All right, I can see your arguments. Do note that very little down-payment has been required for a couple of years. I also don't believe in the housing bubble. 91.120.48.242 (talk) 07:55, 5 March 2013 (UTC)[reply]
  • LOL. You "don't believe in the housing bubble". Now I know you are a troll. OsmanRF34 (talk) 19:04, 5 March 2013 (UTC)[reply]
I think I mean the opposite what you thought I meant. What I mean is that if I am given a mortage for a million dollar house on a shack that the market values at a million, then when you say that I have a million dollars in collateral, I just don't think it's true. I 'disagree' with the market, if you will. So the minute I take control of the shack in my own personal mental accounting (which you have justified as being out of keeping with actual accountancy) I have all the net negative worth that is the difference between the inflated price and the worth of the house once the house wakes up. I will grant you that nominally the 20% downpayment gives me nominal positive equity as nominally there is no net change when taking control of the collateral in exchange to counterpartying the loan. 178.48.114.143 (talk) 22:58, 5 March 2013 (UTC)[reply]
Houses do not have any specific intrinsic worth in dollars. Their worth varies depending on the market and the location. The "shack" you describe could be much pricier in Beverly Hills than in a small town in the middle of nowhere. ←Baseball Bugs What's up, Doc? carrots→ 14:05, 6 March 2013 (UTC)[reply]

Why hasn't Elie Wiesel a tattoo?

There are pictures of his left forearm without a tattoo circulating through the internet. The left forearm was the standard place to put it when Wiesel got into the concentration camp. I don't know whether these pictures are photoshopped, Wiesel let this tattoo be removed, or he is a liar, as some claim. Which is true? Is there a chance Elie Wiesel was not at a concentration camp? OsmanRF34 (talk) 15:24, 4 March 2013 (UTC)[reply]

This was already asked here: http://en.wikipedia.org/wiki/Wikipedia:Reference_desk/Archives/Humanities/2012_December_30#Elie_Wiesel.27s_tattoo --TammyMoet (talk) 15:27, 4 March 2013 (UTC)[reply]
That question from December would be mine, and to be fair, it was never fully answered. There is evidently video footage of him without a visible tattoo, so I doubt manipulation is at work, else unaltered footage should have surfaced at some point. I know tattoos often fade with age (I'm not sure exactly what mechanism is at work there, but I have observed it), so there is a chance it was just too faint to be picked up by the camera at a distance. The problem with researching this sort of thing online is that 95% of the results you get in a Google search are from websites operated by Neo-Nazi morons. One could probably find a photo of his tattoo by contacting news agencies that may have photographs archived, many of which may not be available online. Evanh2008 (talk|contribs) 15:33, 4 March 2013 (UTC)[reply]
Ditto. It's a valid question without a valid answer. OsmanRF34 (talk) 15:42, 4 March 2013 (UTC)[reply]
The answer is valid... just not satisfying. The answer is that we don't know why those particular pictures do not show him with a tattoo. We can speculate, but that is not the job of this reference desk. Blueboar (talk) 15:54, 4 March 2013 (UTC)[reply]
Surely it has a valid answer. We just haven't found it yet. Evanh2008 (talk|contribs) 15:55, 4 March 2013 (UTC)[reply]
Interesting and possibly pertinent: The person who claimed to have a tattoo with the number Wiesel gave as that of his own tattoo (if you looked into this in any depth, I'm sure you know who I'm talking about) has evidently never been interviewed by a news agency, as far as I or anyone else can tell. The "article" featuring his story showed up ex nihilo on several message boards within a few days of each other, lacking a byline or any mentioned of where it originated. Where, when, or by whom it was originally published is unclear, assuming it ever was published. Evanh2008 (talk|contribs) 15:55, 4 March 2013 (UTC)[reply]

No, there is no possibility that Elie Wiesel was not in a concentration camp. When I first read this question, I suspected that it might constitute some form of Holocaust-denial trolling. It is imperative that if this topic is going to be discussed at all in this form (and the value of doing so is not at all clear), that people be sensitive in how they phrase things. Newyorkbrad (talk) 16:14, 4 March 2013 (UTC)[reply]

Yes, it is imperative that this topic be discussed, because two editors in good standing asked the same question, and the Reference Desk's job is to provide references. Whether you personally find the question valuable is irrelevant. No, people don't need to be "sensitive". Sensitivity is just another excuse to restrict other people's freedom of speech, and Wikipedia is not censored. If you don't like it, don't edit Wikipedia. --140.180.251.41 (talk) 17:26, 4 March 2013 (UTC)[reply]
Oh yes certainly this and anything can be discussed. But there is no need to be insensitive. Bus stop (talk) 17:32, 4 March 2013 (UTC)[reply]
You're either missing the point or being deliberately obnoxious. No one is censoring anything. Of course everyone has the right to discuss whatever topic is pertinent to this desk, but they also have the obligation to not be a dick about it. Just because you can do/say/type/spout a thing doesn't mean you should. The reason I asked the question is that I was hoping someone had access to a picture of Mr. Wiesel's tattoo, to definitely establish (as if it hasn't been established enough already) that the Neo-Nazi trolls are nothing but an uniformed pack of bigoted morons. I would suggest that's reasonable cause to discuss the topic, but I certainly don't blame Brad or anyone else for being uneasy or suspect as to the motives of the person asking the question. Evanh2008 (talk|contribs) 17:41, 4 March 2013 (UTC)[reply]
Being suspect as to the motives of the person asking the question is by definition a violation of WP:AGF. You (Evanh) should know better than anyone how important WP:AGF is, because you asked this question and presumably wouldn't have appreciated accusations of neo-Nazism. Additionally, Newyorkbrad said "it is imperative [...] that people be sensitive..." No, it's not imperative. There's simply no policy requiring people to be sensitive. It's a good idea and highly recommended, obviously, but not "imperative". That's all I objected to; otherwise I agree with Bus stop & Evanh. --140.180.251.41 (talk) 18:01, 4 March 2013 (UTC)[reply]
"Motives" are a function of wording. Wording should be sensitive and not insensitive. Bus stop (talk) 18:11, 4 March 2013 (UTC)[reply]
Asking for sources that establish that Elie Wiesel currently has a tattoo is a different thing entirely from someone coming along and saying "Well, if he was in auschwitz how come he doesnt have a tattoo, huh?" One is an explicit request for sources for a stated purpose, while the other could easily be read as an invitation to debate by someone who has an agenda to push. I had the context of Osman's reply to my thread in December to refer back to in order to establish that he wasn't here to push an agenda. Brad may not have had the same benefit, and it was ambiguously worded enough that he had good cause to be suspicious. If Brad had blanked the question with a note to the effect that it was neo-Nazi spam, then AGF may be relevant. Merely stating that we should be careful not to give racist fringe theories more legitimacy than they deserve is not an AGF violation. Evanh2008 (talk|contribs) 18:17, 4 March 2013 (UTC)[reply]

This is a typical conspiracy theory. Here's a source from a few months ago, where the journalist themselves saw the tattoo. "Aha!" shout the conspiracists, who of course, have no agenda whatsoever in trying to somehow 'disprove' the Holocaust by attacking one tiny detail. "Where are the photos?". Weisel himself gives the answer in that article, "Usually I don't show it,", except, it seems, to that journalist, Obama and Merkel. I can certainly understand why. Because if he did react to the speculation, which is a fairly repugnant thing to wish to do, it'd do no good. The theorists would simply call it a fake, PhotoShopped, inked, or recently tattooed. Logical arguments aren't really the issue here - people trying, because their ideology say so, to refute the wholesale slaughter of 6,000,000 people, aren't really going to be too bothered about logic.Or truth. --Dweller (talk) 16:37, 4 March 2013 (UTC)[reply]

^What he said. Evanh2008 (talk|contribs) 17:05, 4 March 2013 (UTC)[reply]
  • Note: the question is as analog as asking if Alicia Esteve Head is a 9/11 survivor. It doesn't imply that you are constructing a conspiracy theory aroud 9/11. And BTW, no matter what you say, how relevant or well sourced something is, conspiracists won't change. It's not worth taking them into account. I have never seen one been converted. They always have a catch all argument for any evidence: "the evidence was constructed so you won't notice the conspiracy." OsmanRF34 (talk) 18:49, 4 March 2013 (UTC)[reply]
  • That sounds pretty reasonable. But tell me please, why then do you need to know whether someone who obviously doesn't want to show it, has a tattoo? — Sebastian 20:18, 4 March 2013 (UTC)[reply]
I don't need to, and it is not just whether someone has a tattoo. It's not as if I were asking about Britney Spears' or Angelina Jolie tattoo disaster. I was asking about a notable writer, who built his notability upon surviving the holocaust. If he appears on a picture showing his forearm without a tattoo, it leads to certain questions. OsmanRF34 (talk) 20:48, 4 March 2013 (UTC)[reply]
You say "leads to" as if it were a natural effect that you have no influence over. But you are a thinking and deciding human being. There are millions of questions one can follow in the world. Why, of all these questions, do you feel this one is worthwhile asking? He did not merely "buil[d] his notability upon surviving the holocaust", but by expressively writing about it. That doesn't "lead" me to want to see his tattoo. If someone tells you they are Jewish or Muslim, does that mean it "leads" you to check whether he is circumcised? I am asking this question of you because (1) I have a very hard time trying to understand what motivates you, and because (2) it will help us meet your need if you can explain it. — Sebastian 22:01, 4 March 2013 (UTC)[reply]
There are about 150 questions on the Reference Desk. Do you intend to ask each OP why they want the answer to a certain question? If the answer is simply "because I'm curious", is that good enough for you? The Reference Desk's job is to provide references for questions, not to question the OP's reasons for asking them. Nobody needs to justify why they want to know something, be it trivial or profound. --140.180.251.41 (talk) 23:41, 4 March 2013 (UTC)[reply]
How many of these 150 are repeat questions? If an OP repeats a strange question without mentioning that he/she participated in asking that same question before, then I might ask them, too. And if someone then says "is imperative that this topic be discussed", it adds significantly to the weight of the question for the motivation. — Sebastian 00:05, 5 March 2013 (UTC)[reply]

The widely varying reaction of survivors to their experiences are well recognised. Some have spent the decades since avoiding discussing or referring to it in any way, shape or form. Others have found thrive on telling their stories and giving witness to what they saw. I met a survivor once who proudly showed us her tattoo, referring to it sardonically as "Hitler's phone number". I can well understand that there'd be some survivors who'd rather keep it concealed. It'd come down to personal choice on an incredibly sensitive issue. Weisel can express himself fully with words, perhaps he feels no need to do so bodily, especially as many others have done so. Giving credence to the conspiracy by responding to it, would therefore be as personally offensive as it would be utterly pointless, as I explained above. --Dweller (talk) 21:24, 4 March 2013 (UTC)[reply]

police,fbi,cia,state,federal law inforcement imprisoned for crimes

looking to find data or records of the numbers of law enforcement convicted in the usa 206.125.74.123 (talk) 15:33, 4 March 2013 (UTC)[reply]

Interesting topic... I would assume that someone has researched the topic before, but if so I am not finding anything about it in Wikipedia, nor I can not find anything with a quick google search. Blueboar (talk) 16:53, 4 March 2013 (UTC)[reply]
Note that "convicted" and "imprisoned" are different things. Many people who are convicted never serve any time in prison, due to probation, plea deals, etc. StuRat (talk) 17:03, 4 March 2013 (UTC)[reply]

Notable person who was a late-speaker

I'm looking for a story of a famous person, with the following elements:

  • The person was male
  • They were a late-speaker (ie did not speak until a relatively advanced age)
  • Their first-ever sentence was one that was complex, lengthy and impressive

I think there were other elements:

  • The first sentence was addressed to a domestic servant
  • He may have called her "madam"
  • The notable person became notable for writing, possibly philosophy
  • I might have first read the story on Wikipedia!

Can anyone help? --Dweller (talk) 17:02, 4 March 2013 (UTC)[reply]

Thomas Babington Macaulay's first utterance has been claimed to be "Thank you, madam, the agony is abated" (answering an enquiry at age 4 after having had hot coffee spilt on his leg). It doesn't appear in our article and may therefore be apocryphal {The poster formerly known as 87.81.230.195} 212.95.237.92 (talk) 17:15, 4 March 2013 (UTC)[reply]
Put "the agony is sensibly abated" into Google, and you can find various versions of a story which has been circulating for many decades... AnonMoos (talk) 20:06, 4 March 2013 (UTC)[reply]

Which reminds me of a joke: Thomas didnt talk when he was one, or even two. By 3 his parents were worried. He was evaluated and no reason could be found but the doctors declared he was clearly developmentally delayed and Thomas' parents got used to the idea that he was mute and would never talk. So they were shocked one morning when he was 4 years old to hear him say, "the toast is cold". And they said in surprise "Thomas, you can talk! Why havent you talked before this?" And Thomas replied, "Up to now, everything's been fine." alteripse (talk) 01:43, 5 March 2013 (UTC)[reply]

That reminds me of the notoriously taciturn husband. He was wonderful in every way, and they had remained happily together for 40 years - but conversation was definitely not one of his strengths. One day his wife said to him "Darling, you never tell me you love me. I don't remember the last time you told me". He turned to her and said "What did I tell you the day I asked you to marry me?" She said, "Well, you said you loved me and wanted to be with me always". He said, "That's right. If anything ever changes, you'll be the first to know". -- Jack of Oz [Talk] 08:29, 5 March 2013 (UTC)[reply]
A family story concerning my goodself was that I was rather late in starting to speak and communicated by pointing at what I wanted. One afternoon, I was left in the care of a neighbour to whom I recounted the entire story of Goldilocks and the Three Bears; I had just been biding my time. I have a dim recollection of the event, but whether I really remember it or have pieced it together from later retellings, I don't know. Alansplodge (talk) 14:16, 5 March 2013 (UTC)[reply]
I heard this story about a late speaker: A child is nearly 5 and has never said a word. One evening at the dinner table, the child says, "Please pass the salt." The parents are stunned. "You can talk! Why haven't you said anything before?" The young'un says, "Well, up till now, everything was OK." ←Baseball Bugs What's up, Doc? carrots→ 14:46, 6 March 2013 (UTC)[reply]

Thanks. It's definitely the Macauley story, but oddly, [I think!] I've never heard of him before. Cheers. --Dweller (talk) 20:02, 5 March 2013 (UTC)[reply]

A question about formal logic

Logic is the widest subject for me. I am very interested in logic. I have encountered Infinitary logic, Fuzzy logic, and Set theory, but I get confused, which among these should be the focus of my readings? I think that these subjects are more on mathematics than actual logic. Anyway, I find propositional logic and calculus to be very practical and very useful. This motivated me to ask the question: Is propositional calculus and logic the core content of formal or mathematical logic? And, is it a form of classical logic? — Preceding unsigned comment added by 112.205.15.149 (talk) 17:10, 4 March 2013 (UTC)[reply]

It might help us to know what your interest in logic is. For example, do you want to be able to point out logical fallacies in order to win debates ? StuRat (talk) 17:31, 4 March 2013 (UTC)[reply]

I am interested in making and testing the validity of arguments, which is very useful in argumentation. However, I also would like to have some basic understanding about mathematical logic. I think logic and mathematics are still greatly different, although they are both formal sciences. What can you suggest? — Preceding unsigned comment added by 112.205.15.149 (talk) 17:43, 4 March 2013 (UTC)[reply]

fuzzy logic is usually part of applied mathematics and AI. IBE (talk) 17:54, 4 March 2013 (UTC)[reply]


Mathematical logic is not so much the logic of mathematics as it is the mathematics of logic. You seem to have more or less come to that conclusion on your own. So it's probably not what you're directly seeking.
That said, to me, it's one of the most fascinating studies ever entered into by the mind of Man. So I'd encourage you to spend at least some little time looking into the basics of it. Many of the very early concepts in mathematical logic (for example first-order logic) are relevant to the things you're asking about. --Trovatore (talk) 20:22, 4 March 2013 (UTC)[reply]

Skipping church due to illness?

Can a person skip church due to being seriously ill? Can the person feign illness in order to skip church? 140.254.226.251 (talk) 17:13, 4 March 2013 (UTC)[reply]

The answer to your questions are "yes" and "yes." To my knowledge, there is no country where church attendance is legally mandated, nor is there any Christian denomination that requires a record of perfect attendance. In fact, I suspect the vast majority of churches don't even keep records of such things. Evanh2008 (talk|contribs) 17:17, 4 March 2013 (UTC)[reply]
What if I change "church" to "house of worship"? 140.254.226.251 (talk) 17:21, 4 March 2013 (UTC)[reply]
I think the answer would remain the same. I'm sure there are some houses of worship somewhere that keep a record of attendance, but I doubt any of them would be prone to do anything about someone not showing up. I don't think failure to attend worship services is legally actionable in any jurisdiction (I could be wrong, though; some very conservative Muslim nations may have laws pertaining to this). Unless it's a very small community, there's a good chance the person's absence wouldn't even be missed, and a similar chance that no one would think to inquire as to its cause. If you are asked, and claim to be sick when you were not, it's unlikely anyone would care enough to launch a full-scale investigation into your recent state of health. Evanh2008 (talk|contribs) 17:27, 4 March 2013 (UTC)[reply]
(edit conflict) Can a person become a member at more than one church and alternate attendance at the chosen churches, or does the person have to attend the church to which he or she has given his or her loyalty or has been granted membership of the religious community? By the way, can a person become a Jew, Christian, and Muslim at the same time? 140.254.226.251 (talk) 17:35, 4 March 2013 (UTC)[reply]
Many of those religions are mutually self-exclusive. For example, you must believe Jesus was the son of God to be a Christian, and you must not believe that to be a Muslim or Jew. So, such a belief would be considered apostasy. In many cases, they might not actually do anything about it, but those strict Muslim communities might. StuRat (talk) 17:44, 4 March 2013 (UTC)[reply]
(edit conflict) Technically speaking you "can" do pretty much anything when it comes to church membership and attendance, constrained by your ability to juggle attendance in a way that won't cause suspicion or result in some form of sanction against you (a Catholic priest, for example, may not be too happy if he hears rumors of you frequenting the local Baptist church once a month, or the Methodist congregation every third Sunday). With regard to conversion, Islam regards any person who assents to the doctrines and teachings of the religion to be a de facto Muslim. Protestantism generally requires only baptism and a brief declaration of faith in the basic tenets of (the particular denomination of) Christianity. Catholicism will require baptism as well, but also confirmation and a course of catechism study in some form. Judaism is somewhat more complicated, as no rabbi of any Jewish religious movement would perform a conversion on anyone openly pledging allegiance to any other religion, least of all Christianity. You could lie about it, of course, but your conversion is likely to be invalidated once it becomes known that you converted under false pretenses (I imagine this would also be the case for Catholicism, but I'm not sure). Evanh2008 (talk|contribs) 17:51, 4 March 2013 (UTC)[reply]
Indeed. See also the helpful fictional reference Life of Pi, where the main character practices all the main religions at once. --Demiurge1000 (talk) 19:41, 4 March 2013 (UTC)[reply]
Yes, I suspect that in some conservative Muslim communities, never attending the mosque would get you in trouble. However, even they realize that dragging the seriously ill to the mosque is inappropriate, as they may get everyone else sick, and vomiting and having diarrhea in a mosque isn't exactly "respectful". StuRat (talk) 17:30, 4 March 2013 (UTC)[reply]
OP, there seems to be an unstated element to your first two questions: "Can I skip church, feign illness etc without incurring some penalty, punishment, or sanction?". But you haven't stated what these hypothetical fates worse than death might be. If you're talking about going to Hell for eternity, or some lesser penance commensurate with a lesser sin, you'd better consult your priest/minister about that. If you're talking about the Church Police turning up at your house and dragging you kicking and screaming to church, that doesn't happen in civilised societies. If you're talking about your own guilty conscience, that's a matter between you and yourself. -- Jack of Oz [Talk] 19:09, 4 March 2013 (UTC)[reply]
The OP geolocates to Ohio State. Hardly civilised society. μηδείς (talk) 19:28, 4 March 2013 (UTC)[reply]
I think I should refer to the Test Acts at this point - in seventeenth-century England (not _that_ uncivilized by absolute standards), it _was_ compulsory to go to church if one wished to hold any sort of public office. Tevildo (talk) 19:55, 4 March 2013 (UTC)[reply]
Not every Sunday however -- the point was to demonstrate affiliation with the Church of England, not to enforce regular Sunday attendance. AnonMoos (talk) 20:02, 4 March 2013 (UTC)[reply]

I wonder if it would be easier for such a person to hold an "Unaffiliated" status, because such a person is unaffiliated with any religious body. At the same time, that person may be a visitor at local congregations to savor different religious experiences in many different ways, while holding up the "Unaffiliated" sign. Such a person may alternate attendance at various churches but may raise his or her children in only one particular faith, which in the opinion of the person, is the most reliable one. 140.254.226.231 (talk) 21:45, 4 March 2013 (UTC)[reply]

Practically it is very hard to raise children in a faith you do not practise yourself. Many, perhaps most, places of worship welcome visitors. Rmhermen (talk) 23:04, 4 March 2013 (UTC)[reply]
Does the questioner understand the difference between "Can"(Is it physically possible?) and "May (Might the act incur the wrath of earthly or heavenly beings?). Edison (talk) 05:55, 5 March 2013 (UTC)[reply]
Yes, you can skip church due to an illness. You will find no salvation in church attendance, it is simply a means to an end. Plasmic Physics (talk) 06:26, 5 March 2013 (UTC)[reply]
  • I dated a Catholic once with a rather strict/religious father. The Catholic Church states that mass attendance is mandatory on Sundays and on other Holy days of obligation. Interestingly attending a Saturday service that followed the Sunday Mass appeared to fulfill this obligation. According to her father, at least, attending my church did not fulfill her obligation so if she went to church with me, she had to go twice. Later, when he began attending Latin Mass, she had to attend the Latin Mass even if she already attended the regular Catholic Mass. This was more of her father's restrictions than the Church; however, some Catholics have mentioned that they aren't allowed to receive the Holy Communion if they did not attend Mass the prior week. That said, there are many Catholics, like members of other denominations, who frequently miss church service. Aside from this, there have been, and still are, areas in the United States where church attendance is culturally mandated, but infrequent absences due to sickness would still be acceptable in these regions. The Bible Belt is a well known area where this culture exists, but there are many other small towns with similar characteristics. Town leaders and business owners make sure it is well known that they attend church, and while it is not uncommon to see a church fill from the rear, in many of these areas, members of the congregation will sit in the front so that they can be seen doing so. This said, many frequent attendees may not actively practice the tenants of their religion outside of church; conversely, there are those who actively practice their religion, but attend religious services infrequently. Ryan Vesey 06:50, 5 March 2013 (UTC)[reply]
A particular denomination may mandate and oblige as it pleases, but such are ultimatly irrelevant and may even be harmful, with repect to a member's salvation. Here are two facts from the Bible (paraphrased): salvation does not come by way of works, nor can holy devotion be compelled, for it must be desired, less it be empty and vain. Plasmic Physics (talk) 10:17, 5 March 2013 (UTC)[reply]
Which may be true for Seventh Day Adventists or whatever you are, but not for the Catholic Church, which is what Ryan was talking about. However, if that girl in Ryan's story went to mass and took communion more than once in a single day (or more than once in a single week), that's A Bad Thing, Catholically speaking. Adam Bishop (talk) 11:53, 5 March 2013 (UTC)[reply]
That doesn't seem to be true, Adam. See COMMUNION TWICE A DAY and Why does the Church teach that works can obtain salvation? which points out that Catholics don't teach that although it misrepresents the usual Protestant position. Rmhermen (talk) 15:07, 5 March 2013 (UTC)[reply]
I'm not making some sort of denominational interpretation, I'm just laying out the facts as they are written. I can insist that you are from Tanzania, but that does not make it so. Likewise, what any denomination insists, cannot override what is written in Scripture. The most I can do is pretend that you are from Tanzania, and the most a denomination can do is pretend to be God, and retract what was established, and we all know (I hope) what the implications are of such pretence. Plasmic Physics (talk) 23:33, 5 March 2013 (UTC)[reply]
Rather, you shouldn't be insisting that a) your interpretation of scripture is the only valid one and b) that because your denomination holds that the Holy Bible and only the Holy Bible contains the necessary rules for living a christian life (Sola scriptura) does not mean that Christian denominations that believe something different are wrong. They are wrong according to the rules of the denomination you follow which is fine, but they are not wrong according to the rules of their own denomination, which is what is relevant for this current discussion. What you or I believe to be correct based on what our own denominations follow is irrelevant to a discussion about what Catholic doctrine tells Catholics is correct or wrong. --Jayron32 00:42, 6 March 2013 (UTC)[reply]
I bolded 'not' for clarity. Plasmic Physics (talk) 02:38, 6 March 2013 (UTC)[reply]
Bolding it doesn't make it true anymore than when you first stated it. You are making statements which are specific to certain denominations and not others. To make a claim that the interpretation of your denomination can be applied to the interpretations of another, like Catholicism, and based on that interpretation claim that Catholicism is wrong; well that's just not valid. You can state that based on the interpretation of your denomination, you would consider Catholicism's position wrong, but the Catholic position on the matter is entirely internally consistent with Catholic doctrine. When you state "what any denomination insists, cannot override what is written in Scripture." is simply false, in the sense that Catholics don't see what they practice as overriding anything written in scripture. You may see it that way, and the official doctrine of your denomination may see it that way, but insofar as Catholics don't, you can't make unilateral statements that imply that Catholicism is in the wrong. You can state that you believe them to be in the wrong, but you need to make clear the distinction between views expressed based on your own personal beliefs versus principles which are not in dispute. --Jayron32 14:10, 6 March 2013 (UTC)[reply]
I am not making statements which are specific to certain denominations, I am paraphrasing. Paraphrasing and interpreting are two very different concepts. Of course the catholic position on the matter is consistent with Catholic doctrine, I am not agrueing against that. What they see their practise as, is irrelevant, opinions don't count when it comes to logic - add will always mean add, and substract will always mean substract, irrespective of what I opinionate concerning it. Plasmic Physics (talk) 07:48, 7 March 2013 (UTC)[reply]
Catholics would argue that what a denomination insists can easily override what is written in Scripture, see Papal infallibility. If a denomination accepts Papal infallibility, any requirements not explicitly stated in the Bible can be added. Ryan Vesey 08:06, 6 March 2013 (UTC)[reply]
Certainly, that does not contradict my comments at all. No one can stop you from accepting that which you fancy. In any case, Scripture still remains independent from the denomination - it remains their reference not their work. It would be like someone going to Albert Einstein's Theory of Relativity, and crossing out E = mc2 and writting E = m/cπ, and then saying that is correct and acceptable because they say so. Plasmic Physics (talk) 08:24, 6 March 2013 (UTC)[reply]
(edit conflict) Not to mention the fact that all denominations who agree on "sola scriptura" have managed to disagree with each other on very nearly every other theological topic you can imagine. Evanh2008 (talk|contribs) 08:26, 6 March 2013 (UTC)[reply]
"I'm not making some sort of denominational interpretation, I'm just laying out the facts as they are written." Please read with careful attention in the future before you comment. Plasmic Physics (talk) 08:32, 6 March 2013 (UTC)[reply]
When that person makes those changes, the work is no longer of Einstein, but of that person, albeit heavily plagiarised. Any subsequent studies based on the new work would be unique as well. Thus, if one were to modify Scripture in such a way, they would be creating a unique source for a unique religion. Plasmic Physics (talk) 08:38, 6 March 2013 (UTC)[reply]
Scripture itself was written and compiled by different denominations - the Bible most Protestants use is different than the one Catholics use. Which scriptures to use, in which translation, how much weight to give to them as opposed to other traditions, and how to interpret them are all decisions made by different denominations in different ways. thx1138 (talk) 14:37, 6 March 2013 (UTC)[reply]
"I'm not making some sort of denominational interpretation, I'm just laying out the facts as they are written." There you go, it is a unique religion, with unique scripture. For example, they deify Mary and the Pope, so it's not just the Father, Son, and Holy Spirit, but also Mary, and the Pope.
I've never heard of any religion that deifies Mary or the Pope. Which one were you thinking of? -- Jack of Oz [Talk] 19:42, 7 March 2013 (UTC)[reply]
According to their own encyclopedias, and authorised writings, Mary and the Pope are imbued with God-like abilities and natures. Plasmic Physics (talk) 00:23, 8 March 2013 (UTC)[reply]
When you left your comments unsigned, I assumed you were trolling, because nobody able to cope with editing and indenting properly could surely be so ignorant as to think that the Catholic Church actually imbues Mary or the Pope with God-like abilities. I mean, if you have access to the internet, the ability to use a search engine, basic reading comprehension, and an interest in the topic, it would take wilful ignorance to think that. But here you are, signing your comment, claiming to have read "their own encyclopedias and authorised writings" (I assume you mean the Old Catholic Encyclopedia and the Catechism of the Catholic Church), so maybe you really think that. Most puzzling. But then, there are people who reach adulthood convinced the moon only comes out at night, so I suppose it takes all sorts. 81.156.144.160 (talk) 13:56, 8 March 2013 (UTC)[reply]
Okay, unpicking that: adult Catholics have an obligation to try their very best to attend Mass every Sunday and Holy Day of obligation. Attending a different service at another church doesn't count, because it is not the Mass. If you're ill, or there's some other good reason (which doesn't include "I really wanted to watch this film" or "I had a football match"), then that's fine. Placing trivial things ahead of Mass and your duty to God (for example, by not attending Mass because you wanted a lie in, or because you wanted to go to a concert, or something) is considered grave matter in Catholic teaching. If you commit a sin involving grave matter, on purpose, knowing that it is grave, then that is a mortal sin. If you have committed a mortal sin, then receiving Communion is "eating and drinking judgement" on yourself, so you shouldn't receive Communion (as a Catholic) until you have been to the Sacrament of Reconciliation to wipe the sin away (and you're supposed to do that anyway). For the purposes of obligation, the day is considered to start the evening before, so Saturday evening counts for Sunday.
Adam is a little confused. Catholics are encouraged to attend Mass and receive Communion more than once a week, although the obligation (the minimum) is only weekly for attendance, and annually for Communion. It used to be that the Church restricted Communion to once a day, to avoid people treating it like a charm to be taken in greater and greater quantities. In more recent decades, this has been loosened to twice or more a day (as a maximum), provided that it is received in the context of a Mass, but the clarifications on this clearly suggest this is intended to let people receive Communion at Masses they were otherwise attending for some other reason (like the girlfriend who goes with her father).
The girlfriend didn't have an obligation to attend both a Tridentine Mass and an Ordinary form Mass, but may have had personal reasons for doing so. 86.140.54.54 (talk) 17:14, 5 March 2013 (UTC)[reply]
To answer the OP's original question: Yes, you can skip church whenever you like even if you are perfectly healthy. I've managed to avoid all houses of worship and religious observances for years without suffering any ill effects. If anything, it has enhanced my mental well-being tremendously. —D Monack (talk) 01:43, 6 March 2013 (UTC)[reply]
The "greater sin" axiom would apply here. If you're sick, attending might spread your sickness to others and cause them physical harm. That would be a greater sin than not attending. ←Baseball Bugs What's up, Doc? carrots→ 14:01, 6 March 2013 (UTC)[reply]
Maybe in the Church of Bugs. In the Catholic Church (the only one I know with an explicit obligation), there would be no sin at all in not attending because you were ill. No "greater sin" idea, simply no sin at all. Not anything about spreading illness: just no sin, because you're not doing anything wrong. 86.140.54.54 (talk) 19:29, 6 March 2013 (UTC)[reply]

Would Llywelyn the Last be the twelfth Prince of Wales before its conquest by Edward I of England?--Doug Coldwell (talk) 22:13, 4 March 2013 (UTC)[reply]

Not according to List_of_Princes_of_Wales#List_of_Princes_of_Wales, but I strongly suspect that this is an area where historiography plays an important role and different scholars will count the princes in very different ways.Our article is strongly rooted in an Anglocentric worldview. A historian less bothered about English recognition would have a very different list but then, I seem to remember, things get complicated, as you have claimants to the title that aren't universally recognised as such, even by Welsh historians.--Dweller (talk) 22:21, 4 March 2013 (UTC)[reply]
I've struck some of that - it's a long time since I studied the issues, it's late at night and I'm not convinced I'm right. Where's User:Clio the Muse when you need her? --Dweller (talk) 22:29, 4 March 2013 (UTC)[reply]
O.K. Thanks for reply Dweller. Looking at Llywelyn the Last's Family tree it shows him as one of the twelve before its conquest by Edward I of England, taking in consideration that English recognition might not be consistant on this. Hopefully I'll get some additional answers on this.--Doug Coldwell (talk) 22:38, 4 March 2013 (UTC)[reply]
That family tree doesn't exclusively list Princes of Wales. It listed his father who never ruled and his daughter and the descendants of his brothers who never ruled either but were just prisoners of the English. Only Dafydd ap Llywelyn, Llywelyn the Last and Dafydd ap Gruffydd used the specific title "Prince of Wales" before the English conquest, but there were many other kings and princes of the different Welsh kingdoms listed on List of rulers of Wales. Also counting rulers is really arbitrary since we got period of history where Llywelyn contended the rule of Gwynedd with his brother Owain Goch ap Gruffydd. --The Emperor's New Spy (talk) 07:49, 5 March 2013 (UTC)[reply]
It is also worth noting that nothing like the primogeniture inheritance of the right of rule in all of Wales ever existed in the way it did in the Kingdom of England. It may have been developing by the time period under discussion, but it never came to fruition because Wales was conquered by England. Instead, Wales (like Ireland and earlier Anglo-Saxon England) was full of petty states ruled by various clan/tribal leaders. At times, a particularly strong leader would establish a temporary Hegemony over several other kingdoms; this sometimes extended to most (though probably not ever all) of Wales, especially in the case of Llewellyn the Great, but this sort of hegemony was akin to a sort of high kingship or Bretwalda (to use the Anglo Saxon term): it was not a heritable title, was not passed down through families as a matter of law, and only lasted as long as the person in question could maintain his power through politics and force of arms. At the very end (after Llewellyn the Great) there came a time when the "Prince of Wales" title looked to be developing into a hereditary kingdom, akin to what happened to the Bretwalda that developed over the centuries into the King of England (through the long Hegemony of a single family, the House of Wessex), but that process never completed, as Wales lost its status as an independent land by the middle 1200s. --Jayron32 14:36, 5 March 2013 (UTC)[reply]
Thanks Jayron32. I see I have some studying to do here.--Doug Coldwell (talk) 20:29, 5 March 2013 (UTC)[reply]
If you want some of the original sources, Brut y Tywysogion is probably worth looking into. King of Wales is also some good reading; it discusses the putative Welsh kingdom; it represents a historiographical concept rather than a historical fact, as only one person ever ruled all of Wales, and then only briefly, and he never used the title "King of Wales", rather titles like "Prince of Wales" and "King of the Britons" was used. From the King of the Britons article, you can see how the title passed around between the various major Welsh principalities like Gwynedd, Powys, and Deheubarth but never represented a the sort of kingship we come to associate with places like England, France, etc. --Jayron32 00:36, 6 March 2013 (UTC)[reply]
Thanks for the additional information. This is great! I see this is going to keep me busy for some time.--Doug Coldwell (talk) 12:24, 6 March 2013 (UTC)[reply]

Where can I find an introduction to austerity vs. stimulus?

Recently someone pointed me to this Paul Krugman video as an example of Krugman's arguments for economic stimulus instead of austerity being "demolished" but as you can see from the segment starting at 49:25 Krugman gives three reasons that he and his colleagues have been proven right (interest rates, inflation, and the results of government spending cuts). This is all very confusing to me, and honestly, Wikipedia's articles on the subjects involved to not make the explanations any simpler.

So, my question is, where can I get a simplified summary of the major arguments in favor of supply-side austerity on one hand, and the major arguments for demand-side Keyensian stimulus for job creation on the other, with evidence and rebuttals, preferably geared towards something like a High School reading level? 71.215.68.213 (talk) 22:46, 4 March 2013 (UTC)[reply]

Which of Wikipedia's articles have you looked at so far? You could make it easier for us to help you if you helped us avoid posting something here that isn't helpful for you. — Sebastian 22:49, 4 March 2013 (UTC)[reply]
Austerity, supply-side economics, Keynesian economics, macroeconomics, liquidity crisis, financial crisis of 2007–2008, Economic Stimulus Act of 2008, American Recovery and Reinvestment Act of 2009, Dodd–Frank Wall Street Reform and Consumer Protection Act, regulatory responses to the subprime crisis, subprime mortgage crisis solutions debate, Great Recession, causes of the Great Recession, consumer spending, aggregate demand, anti-austerity protests, pensions crisis, and several others which aren't as closely related as any of those, so I won't bore you with them. I do have two specific suggestions:
Wow, that's an impressive list! If you looked at all of these, you certainly did your homework! I agree that "demand-side economics" or "demand side economics" is an important term that deserves an article. The latter currently is a redirect to Keynesian economics, but that page only mentions that term in passing. — Sebastian 23:44, 4 March 2013 (UTC)[reply]
Be aware here that framing the debate as austerity versus job creation is, in some people's views, to take a negative stance on austerity. The two are not necessarily opposing poles. --Dweller (talk) 23:06, 4 March 2013 (UTC)[reply]
Granted, I've replaced "job creation" with "stimulus". 71.215.68.213 (talk) 23:18, 4 March 2013 (UTC)[reply]
Also note that austerity and stimulus aren't necessarily mutually exclusive, either. That is, stimulus can be used during economic downturns, and austerity when the economy is booming, to pay off all the money borrowed during the stimulus (or better yet, to save up money for when it's needed for the next stimulus period). Actually getting politicians to practice austerity in good times, however, is darned near impossible. StuRat (talk) 00:25, 5 March 2013 (UTC)[reply]
That happened in the US in the late 1990s, but tax cuts and war in the early 2000s undid it several times over. Most of the time war involves a tax increase, so I wonder if "darned near impossible" reflects what actually would have happened under a more typical series of events. 71.215.68.213 (talk) 00:42, 5 March 2013 (UTC)[reply]
[[5]]. 23:39, 4 March 2013 (UTC)
Thanks, but I'm looking for something in a pros-cons-evidence-rebuttals format, not an ideological reflection on proponents. 71.215.68.213 (talk) 00:42, 5 March 2013 (UTC)[reply]
Well, try as you might, you're probably in the wrong place. Honestly, if you're going to delve into monetary policy, you're going to be largely on your own. Don't think this is new. It's been true since the first fiat currency was created. There are a lot of smart, active traders on wall street that believe deeply in the gold standard, or whatever variation is up at the moment. This has been going on since Madison. There are very few true fiat currencies in play. Many are tagged to the Euro, or the Pound, or the Dollar. If you really want to blow your mind, look at bitcoin. Shadowjams (talk) 01:16, 5 March 2013 (UTC)[reply]
Stimulus and austerity are fiscal policies as opposed to monetary. (But fiscal policy doesn't have a good summary of both points of view, either. So far, austerity comes closest, but it's written at a graduate or postgraduate level, mostly.) 168.103.90.42 (talk) 03:26, 5 March 2013 (UTC)[reply]
One thing I should point out is that stimulus helps in the short term, and hurts in the long term, when the bill comes due. Austerity, on the other hand, helps in the long term, but can hurt in the short term. StuRat (talk) 01:25, 5 March 2013 (UTC)[reply]
While that's a probably accurate platitude, I don't know if any economists back it up. Shadowjams (talk) 01:27, 5 March 2013 (UTC)[reply]
'Cause it's probably not accurate, either. The economy is not a household budget. --Mr.98 (talk) 02:47, 5 March 2013 (UTC)[reply]
As quoted in Austerity#Balancing_stimulus_and_austerity:

IMF managing director Christine Lagarde wrote in August 2011: "For the advanced economies, there is an unmistakable need to restore fiscal sustainability through credible consolidation plans. At the same time we know that slamming on the brakes too quickly will hurt the recovery and worsen job prospects. So fiscal adjustment must resolve the conundrum of being neither too fast nor too slow. Shaping a Goldilocks fiscal consolidation is all about timing. What is needed is a dual focus on medium-term consolidation and short-term support for growth. That may sound contradictory, but the two are mutually reinforcing. Decisions on future consolidation, tackling the issues that will bring sustained fiscal improvement, create space in the near term for policies that support growth."

StuRat (talk) 03:49, 5 March 2013 (UTC)[reply]
What both parties have learned over time is that government spending helps fuel the economy, and government austerity helps to shrink it. The monkey wrench in the system is that the "job creators" in America have been creating lots of offshore jobs rather than hiring Americans, which counteracts the theoretically beneficial effects of govenrment spending. ←Baseball Bugs What's up, Doc? carrots→ 03:47, 5 March 2013 (UTC)[reply]
  • Really? Austerity means tax increases, and stimulus means spending increases. That's less than half the actual potentiality. μηδείς (talk) 03:43, 5 March 2013 (UTC)[reply]
  • Austerity can mean both spending cuts and tax increases. It basically means that the government is trying to live within it's means, both by reducing spending to match it's means, and by increasing taxes (or reducing loopholes) to increase those means. The austerity is on the part of the public, which now gets fewer benefits, and, perhaps, pays more taxes. StuRat (talk) 21:27, 5 March 2013 (UTC)[reply]


March 5

Popular vote versus congressional representatives in the United States

Obviously, in the most recent general election, the democrats won the popular vote for President, but the republicans got more representatives in the house. But how different was the distribution of the popular vote in each? There were more people voting democrat for president, but were there more people voting republican for the house of representatives? Or is the disparity merely a result of how districts are broken down? Speaking of which, what percentage of Americans did vote for a republican for the house in the last election? Historically, does that proportion tend to match the percentage that votes the same way for president? Thanks. Someguy1221 (talk) 01:59, 5 March 2013 (UTC)[reply]

As our article United States House of Representatives elections, 2012 clearly shows, Democratic candidates received a plurality of the votes in House elections in 2012. (Democrats received 49% of the vote, Republicans 47.7% of the vote, and smaller parties received the remainder.) So, if the United States had proportional representation, Democrats would be the largest party in the House. However, we have first-past-the-post voting in which the geographic definition of voting districts can change election results. The Republican majority is, as you say, an artifact of Republican gerrymandering that divides areas with large numbers of Democratic voters among districts with Republican majorities and/or concentrates Democratic voters in a small number of districts within a state and dilutes Democratic votes elsewhere in the state by making sure that any Democratic population is counterbalanced by a larger Republican population in a given district. Marco polo (talk) 02:14, 5 March 2013 (UTC)[reply]
For some nice statistic analysis of the role of gerrymandering in the last election, this is a good source. --Mr.98 (talk) 02:45, 5 March 2013 (UTC)[reply]


(ec) Given that the large states with the large congressional delegations tend to have Democratic legislatures (Texas being the obvious exception), I'm skeptical of the claim that the discrepancy is from Republican gerrymandering. The top level of "districting" for the House of Representatives is just state boundaries. Maybe Democratic voters are concentrated in heavily Democratic states, resulting in the same effect, but without it being the result of any deliberate policy, just where people happen to live). Also, every state, no matter how small, gets at least one representative, and states small enough to benefit from that tend to break Republican. --Trovatore (talk) 02:46, 5 March 2013 (UTC)[reply]
States with larger cities tend to lean more toward Democrats. Smaller states, by definition, lack large cities, hence they lean Republican. And more than one commentator has pointed out that thanks to manipulation of the district lines, there are very few "swing" districts, meaning that nearly every Congressman feels secure in his job, meaning that they have no reason (at the momen) to compromise or try to reach consensus. ←Baseball Bugs What's up, Doc? carrots→ 03:39, 5 March 2013 (UTC)[reply]
Yeah, tiny states like Texas lean republican because they don't have any major metropolitan areas .. Whereas big states like Connecticut are liberal because they're full of large cities. Shadowjams (talk) 08:36, 5 March 2013 (UTC)[reply]
Urban areas tend to be more Democratic than rural areas. In the south, the Democrats used to be the party of choice because they were the white supremacist party. When the Republicans took on that role and the Democrats abandoned it, the south switched from Democrat to Republican. ←Baseball Bugs What's up, Doc? carrots→ 14:35, 5 March 2013 (UTC)[reply]
(EC with below) I don't want to get in to this whole debate but I think it's fairly well established Democrats tend to do better in urban areas and Republicans in more rural areas. Of course it depends on the state, but I believe these trends are fairly common in most states even if how well they do may vary. See e.g. [6] and [7]. While these are going by the presidential vote rather then the congress vote, they do highlight the general trend, the later even gives some statistics for Texas showing it holds there to some extent (in fact very few cities in the US with a population over 100k voted had a majority for Romney).
I believe in a number of states, rural areas get a greater proportion of the vote. This is by definition Malapportionment (but not necessarily gerrymandering depending on your definition), but it isn't unique to the US and there are various arguments discussed on wikipedia as to why some degree of bias to towards rural areas may be fair (but of course even for those that agree there is argument about what level).
But anyway I think what Trovatore and Shadowjams are getting at is that it's more complicated then gerrymandering. As discussed in the earlier links and also in [8] and [9] which do relate to the congressional elections, one problem is that urban areas often very heavily lean Democrat whereas rural areas don't lean Republican quite so heavily. In other words, Democrats voters are far more concentrated then Republican voters. Since the US uses FPTP rather then a form of proportional representation, a Democrat getting 90% of the vote in district A is just as much the congressperson for that district as a Republican getting 51% of the vote in District B. So if you want to produce contiguous (and perhaps somewhat reasonably shaped) districts no matter what you try to do, Democrats will generally be punished because you can't get around the fact a lot of their voters are concentrated in geographically small areas so there's always going to be far more wasted votes for Democrats then Republicans. (Your solutions would be some strange non contiguous districts or at least very weird shapes, proportional representation or just accept the system warts and all.)
(Note that this doesn't address whether or not gerrymandering is happening.) Nil Einne (talk) 14:28, 5 March 2013 (UTC)[reply]
For historical comparison, consider rotten boroughs in the unreformed House of Commons in the UK, due to moving populations and the expansion of cities. 86.140.54.54 (talk) 16:52, 5 March 2013 (UTC)[reply]
What I've read, per Nate Silver, is that currently Democrat-leaning districts tend to be more strongly Democrat than Republican-leaning districts tend to be Republican. That is, the average "safe" Democrat seat might see an 80/20 election split while the average "safe" Republican seat sees 70/30. This means more "wasted" votes for the Democrats, and sets up the scenario where the Republicans hold a House majority despite the Democrat plurality of votes. Gerrymandering is something of a factor, and there were more states with Republican-majority legislatures than Democrat-majority legislatures after the 2010 census, but both parties are guilty of trying to shift the odds in their favor by that means. — Lomn 14:14, 5 March 2013 (UTC)[reply]
To address Nil Einne's suggestion and the mention of rotten boroughs, that kind of malapportionment is unconstitutional in the United States and wouldn't withstand a legal challenge, so it doesn't exist. Electoral districts for any representative body have to be equal in population within fairly narrow limits. This is because a decennial census is written into the U.S. Constitution. Marco polo (talk) 20:56, 5 March 2013 (UTC)[reply]
Marco polo is correct and I apologise for any confusion as I missed it in the articles yesterday. I thought I'd read discussions on this before but I must have been mistaken, perhaps confused by discussions surrounding the senate or the merits of the electoral college (the electoral college because it's the sum of the senate and house does have some small bias towards smaller states and more significantly the winner takes set-up used in most states and current voting patterns mean that smaller more rural states are often key battlegrounds). Note however per our article, until the 1960s and Baker v. Carr + Reynolds v. Sims, it did occur in state legislatures. Nil Einne (talk) 13:41, 6 March 2013 (UTC)[reply]
Actually the "smaller more rural" states usually don't tend to be key battlegrounds, because most of them are safe for one party (Democrat for Hawaii, Republican for most of the rest of them). The only such state that's really in play on a regular basis is Nevada. --Trovatore (talk) 16:41, 7 March 2013 (UTC)[reply]
  • Regarding the OP's last question on "does [the House] proportion tend to match the percentage that votes the same way for president": Not always, because of split-ticket voting, where people vote for one party for president, while they vote for the congressional candidate from the other party. A good example of this was during the 1984 election when Republican Ronald Reagan got 58.8 percent of the popular vote to help him win re-election. However, the Democrats still maintained control of the House, getting 51.9 percent of the popular vote. Zzyzx11 (talk) 07:12, 6 March 2013 (UTC)[reply]

Tipping in the US

During a visit to the US I was quite amazed one of the members of the party carried a cheat sheet called a "tipculator", to calculate the right amount for the tip for the waiter. This image http://imgur.com/gallery/637i9Tv amazes me even more, showing "this is the bill, you are pretty much supposed to pay the amount stated times 1.18". I thought tipping was: "I liked the guy, he made this a fun evening which was better than you'd expect, he certainly deserves some extra money because he did things that were not on the menu. Here's $20 extra for that". The bill I found on imgur pretty much says "Your waiter is paid just enough to cover his bus ride, you have to do the paperwork to pay him the salary the IRS thinks he earns". I think it is even meant to say that the guest is morally a thieve by not tipping 18%. I even suspect the "voluntary" 18% is stated by law to make it even worse. Tipping used to be (it still is in Europe) such a wonderful thing: you know the waiter (or taxi driver or whoever) is paid decently, and you pay extra for extra services, with a high probability of not seeing the service provider ever again, defying any theory of economics. Anyway, to turn a long story into a real question: does an average US citizen consider the line "Tip 18%" as perfectly normal? Joepnl (talk) 03:38, 5 March 2013 (UTC)[reply]

Please just use the "search reference desk archives" function at the top of this page. This subject has been beaten to death to death to death several times to death within the last year or so to death. μηδείς (talk) 03:40, 5 March 2013 (UTC)[reply]
15 to 20 percent is typical. Many restaurants will automatically add 15 percent for large groups. No law compels you to tip. But if you don't tip, they're liable to remember you the next time. Bu if the service was so bad that you didn't tip, you won't be going back anyway. ←Baseball Bugs What's up, Doc? carrots→ 03:42, 5 March 2013 (UTC)[reply]
18 percent seems in line with standard convention. The usual standard is 15-20%, tending towards 20% for better service, and 15% for tolerable, 18% splits the difference nicely, but I find 15% and 20% to be easier to calculate, so I usually leave one or the other (15% is the amount plus half again, move decimal over one place. 20% is the amount doubled, move decimal over one place. 18% has no easy trick to calculate in your head.) As Medeis has already hinted at, expect a conversation to follow below where people will raise levels of vitriol and rage against the practice of tipping itself which is normally only reserved for genocidal dictators and fans of the wrong sports team. Pay them no mind. In the U.S., standard restaurant tipping is 15% to 20% of the bill. That's all you need to know. --Jayron32 05:05, 5 March 2013 (UTC)[reply]
You could always calculate 17.5%, which is considerably easier than 18%. Joe should also remember, in reading any subsequent explosions about tipping, that tips are not included in calculations of minimum wage and tax in countries like the UK, whereas they are assumed in calculations of minimum wage and tax in the US. Adjust expectations and outrage accordingly. 86.140.54.54 (talk) 08:18, 5 March 2013 (UTC)[reply]
17.5% is 10%+5%+2.5%. 18% is 20%-2%; both are reasonably easy to calculate in your head. CS Miller (talk) 06:10, 6 March 2013 (UTC)[reply]
Depends where you are! The old trick in NYC, for example, is just to double the tax, which is currently 8.875%. ~ Amory (utc) 15:30, 6 March 2013 (UTC)[reply]
Most places charge automatic gratuity of 18% on tables over a certain size, usually 6 or 8. If the waiter is good that backfires because most decent patrons tip beyond that, but are less inclined to do so when they're told that they appreciate it 18% (could you imagine an 18% tax hike, and what kind of political response?). Some, less enlightened, restaurants do that for smaller groups. Most Americans tip somewhere between 15 and 25%. That's my colloquial answer. Shadowjams (talk) 12:49, 5 March 2013 (UTC)[reply]
For ease of reference, here is a link to one extended previous debate on this issue. --Viennese Waltz 13:10, 5 March 2013 (UTC)[reply]
Servers where I live (in any restaurant where you can get a full meal for one for less than $20) actually get paid less than half of "minimum" wage because the government figures they'll make up for it in tips. It actually is a moral issue, because the minimum wage is actually less than a liveable salary in the US, so half that (or less) begins to borders on voluntary slavery. Ian.thomson (talk) 13:46, 5 March 2013 (UTC)[reply]
On the other side of the coin, good waiters in good restaurants can make a decent living on tips. My brother has been a waiter for 15 years; he's also had a 9 to 5 job for the past 10 years, but he has never given up waiting because the tips are too good. He has the experience and skills to get the best shifts at good restaurants, has a known reputation among regular clientele as a good waiter, and the money is too good to give up. He wouldn't be able to do so on a salary; the fact that he can parlay his skills and experience into working the best shifts on Fridays and Saturdays makes it very worthwhile for him. --Jayron32 14:24, 5 March 2013 (UTC)[reply]
I have never heard of carrying a printed sheet but my calculator and cell phone have built in tip calculating features. Rmhermen (talk) 14:50, 5 March 2013 (UTC)[reply]
Some Americans seem to be obsessed with getting the tip just right, carrying around a calculator or cheat sheet. What I can't fathom is why they don't just round up to the nearest whole to give a suitably sized tip depending on the quality of the service. So, spend $9, pay $10, maybe $11. Spend $9.50, so leave $11 or $12. Spend $50, pay $60. And so on. Spend $34.93 and had really bad service? pay $35! Astronaut (talk) 18:41, 5 March 2013 (UTC)[reply]
I do that. I do either 15 or 20% and round up to the nearest whole dollar. --Jayron32 19:01, 5 March 2013 (UTC)[reply]
As for paying less than the minimum wage, why isn't that illegal? Astronaut (talk) 18:43, 5 March 2013 (UTC)[reply]
Because the minimum wage law has many exceptions built into it, it only covers hourly wage employees holding down certain jobs, and there are many jobs (not just waiters, it should be said) for whom it does not apply, the article at Minimum wage in the United States only lists tipped labor (but notes that for employees whose tips do not make up the difference, employers still have to pay the minimum wage to). However, I am pretty sure that other forms of pay, including commission, piece work (esp. farm labor) and annual salary may also be exempt from minimum wage standards. I could be wrong about that, however, and I am not sure how non-hourly methods of remuneration are calculated for minimum wage purposes. However, waiters are entitled to minimum wage if their tips do not meet that standard. In practice, most good waiters at most good restaurants make much above the minimum wage (YMMV). In high school, for example, I worked in pizza delivery (basically a car waiter), and back then the minimum wage was $5.00 per hour. My boss paid me that up front (before tips); in tips on a good night I always cleared better than that. I would frequently make $100 or more on a 6 hour shift on a really good evening, $30 on a really bad evening in tips alone. So, just on tips, and just for pizza delivery tips easily matched minimum wage. YMMV as always... --Jayron32 19:29, 5 March 2013 (UTC)[reply]
I delivered papers for about a week. I don't know if it was explicitly listed as an exception or not, but the setup that existed was that I was technically an entrepreneur who purchased the papers from the publisher and sold them to the subscribers. I didn't handle any of the transactions, but if paper delivery wasn't a listed exception, that was probably the workaround. I also did some rock picking (anyone want to create an article?) on a farm for a couple of summers. In addition to being an exception to the minimum wage requirements, we were also able to start when we were around 11. Rock picking can be fun, but I doubt there are many other jobs in America that give as little amount of pay relative to the amount of effort required. The most ridiculous pay scheme I've seen in America is babysitting. Prices are rarely worked out beforehand and parents often return home late. Sometimes babysitters are generously paid, while others receive pay that doesn't come close to compensating them for their time. Ryan Vesey 06:39, 6 March 2013 (UTC)[reply]
I'm a Math teacher in Australia, and I know that some of my less successful students would struggle with almost any of the percentages mentioned here. (Maybe they could do 10%, but.....) Does the existence of tipping everywhere in the US make everyone there better at percentages, or do some people just give up? HiLo48 (talk) 06:22, 6 March 2013 (UTC)[reply]
Well if you pay by debit/credit card there is a % option for tips, so no thinking/math require. Royor (talk) 06:55, 6 March 2013 (UTC)[reply]

Dysfunctional families

  1. What are the core characteristics of dysfunctional families? Unfortunately, the long list of "features", "signs", "styles", etc. on the page do not give clear answer to the question.
  2. How do dysfunctional families come into existence? How do families become dysfunctional? How do individuals form dysfunctional families? How do you explain the long list of "features", "signs", "styles", etc. on the page?
  3. What are the relationships between dysfunctional family, personality disorder (either full diagnosis, PD-NOS, "subthreshold" PD, "personality problems", etc.), and "emotional immaturity"?
  4. What are the branches of psychology most commonly used to understand, explain, and heal dysfunctional families?
  5. Aside from DSM-IV's V61 codes, ICD-9's V61 codes, and ICD-10's Z codes (all very sparse), are there efforts to categorize and classify family dysfunction? Are relational disorders going to be included in DSM-5? Are there psychological tests and psychiatric rating scales used to rate family dysfunction?

Thanks for any answer. If answers to my questions are deemed too long for this page, you can always refer me to a book (scientific one, please). 36.81.31.157 (talk) 05:05, 5 March 2013 (UTC)[reply]

I think one of the problems with "dysfunctional families" is who is defining what is dysfunctional (leaving aside the definition of a family), and to what end are they applying that definition. Depending on how you look at the world and what you think the aim of a family should be, then your approach to the family will differ. Just at the moment I can't think of the main theorists regarding what I've just written (brain fog combined with it's been 10 years since I taught this), but I'm sure someone will come up with it. If not I'll come back later with some names and references. --TammyMoet (talk) 10:37, 5 March 2013 (UTC) Just had a look at the article. No criticism of the concept mentioned at all. Jeez. --TammyMoet (talk) 10:41, 5 March 2013 (UTC)[reply]
Well, we have an article on that. Which is about the best answer the reference desk can provide. But if you want random opinions, I guess you'll get a bunch of them here too. Shadowjams (talk) 12:45, 5 March 2013 (UTC)[reply]
That article's in awful shape, I might add. Shadowjams (talk) 12:46, 5 March 2013 (UTC)[reply]
I think that's highly appropriate (as above, so below, and all that). I'm surprised nobody's mentioned that all families are dysfunctional in certain ways. And probably no family is dysfunctional in every possible way. So it's just a question of degree. -- Jack of Oz [Talk] 19:18, 5 March 2013 (UTC) [reply]
Some other articles and stuff that might help here: Structural family therapy, Talcott Parsons, Structural Functionalism, Family Therapy, Family Stress theory --TammyMoet (talk) 13:36, 5 March 2013 (UTC)[reply]

Oldest Church in Britain

St Martin's Church, Canterbury is considered the oldest church in Britain but are there any other that predate this. Are there any churches from pre-Anglo-Saxon Britain during the late Roman Empire and the subsequent century after its fall.--The Emperor's New Spy (talk) 07:31, 5 March 2013 (UTC)[reply]

No, it's "England's oldest parish church in continuous use", not "the oldest church in Britain". -- Jack of Oz [Talk] 08:21, 5 March 2013 (UTC)[reply]
I think there are some in Cornwall. Itsmejudith (talk) 08:28, 5 March 2013 (UTC)[reply]
Many churches in Wales (and also in Cornwall) are dedicated to saints who lived in those centuries, and it is likely that those churches were founded in their lifetimes or shortly afterwards. The earliest unequivocal evidence of Christian worship in Wales is a plate found with a chi-rho inscription at Caerwent in Monmouthshire - http://www.bbc.co.uk/wales/religion/sites/timeline/pages/religion_in_wales_3.shtml. Many church sites - traditionally believed to be indicated by circular or oval churchyards - derive from that period, but most early church buildings would have been built of wood and have not survived. Another candidate is the Candida Casa or White House established by St Ninian in Whithorn, southern Scotland, in the mid fifth century - again though, I don't think that material of that church survives - http://www.whithorn.com/saint-ninian.htm. Ghmyrtle (talk) 08:54, 5 March 2013 (UTC)[reply]
See also Roman Britain#Christianity which, unsourced, suggests that there may have been churches earlier than Candida Casa. CambridgeBayWeather (talk) 09:07, 5 March 2013 (UTC)[reply]
St Piran's Oratory, currently being excavated, is probably the oldest church in Cornwall surviving in part and may well be an older establishment than St Martin's. Warofdreams talk 10:02, 5 March 2013 (UTC)[reply]
That's good timing - Happy St Piran's Day! Alansplodge (talk) 15:13, 5 March 2013 (UTC)[reply]
A building excavated in Colchester in the 1970s was identified as a church because of its similarity to other early Christian buildings, and its position in the middle of an identified Christian cemetery. The construction was thought to have commenced between 320 and 340 AD based on coins discovered in the foundations. Assuming that it has been correctly identified, "it is probably the earliest known Christian church in Britain." See Butt Road Roman Church and COLCHESTER ARCHAEOLOGICAL REPORT 9: Excavations of Roman and later cemeteries, churches and monastic sites in Colchester, 1971-88. According to the latter report, there are no churches anywhere that predate the Edict of Milan in 313 AD, which ended the persecution of Christians in the Roman Empire. Here is a picture. Alansplodge (talk) 13:47, 5 March 2013 (UTC)[reply]
Our list of oldest church buildings lists three churches now thought to predate the Edict of Milan: Etchmiadzin Cathedral (completely rebuilt; was in the Christian Kingdom of Armenia, so unaffected by the Edict), plus two relatively recent discoveries. Warofdreams talk 15:17, 5 March 2013 (UTC)[reply]
Not in Britain though.... Ghmyrtle (talk) 15:18, 5 March 2013 (UTC)[reply]
I stand corrected, I was paraphrasing the archeologists in the report that I linked to. It would be more accurate to say "anywhere in the Roman Empire". Alansplodge (talk) 11:04, 6 March 2013 (UTC)[reply]
  • There's also a Ship of Theseus problem when defining "oldest church". Are you asking after the oldest church building or the oldest church congregation? And by oldest do you mean oldest extant and still active/in use as such or do you mean earliest? How you mean your question will likely change the answers. --Jayron32 14:20, 5 March 2013 (UTC)[reply]

Does the Catholic church have any position on extraterrestrial life?

Did God create aliens? Do they believe there is no life anywhere in the universe? What's their deal. — Preceding unsigned comment added by 24.228.84.210 (talk) 10:20, 5 March 2013 (UTC)[reply]

That's briefly described in Extraterrestrial_life#Recent history.  Sandstein  11:24, 5 March 2013 (UTC)[reply]
As is it more than possible that any such aliens won't have signed up to an Internet Service Provider which can connect their laptops/Apple wrist-watch computers/brain-implants/etc. to Google and Google Earth, so they may not know about Jesus of Nazareth. Therefore, the Catholic Church will need to send missionaries out to them in-order to convert these aliens to Christianity and by so doing save their heathen souls. [10] Amen.. or in the words of Captain Jean-Luc Picard of the Star Ship Enterprise: “Make-it-so”. As an aside: The Vatican might find that Erich von Däniken was right and that the boot is really on the other foot.--Aspro (talk) 12:12, 5 March 2013 (UTC)[reply]
Oh yes it does! AndrewWTaylor (talk) 12:33, 5 March 2013 (UTC)[reply]
  • Not quite the same scope, but that is really cool. — Crisco 1492 (talk) 12:37, 5 March 2013 (UTC)[reply]
God created the univese, so it follows that if ET's exist, God created them also. I expect the religionist view is practical: "If ET's show up, then we'll deal with it." ←Baseball Bugs What's up, Doc? carrots→ 14:58, 5 March 2013 (UTC)[reply]
  • This has been discussed repeatedly, and any user could have looked it up at the top of the page in the archives. What's the deal with that? μηδείς (talk) 16:30, 5 March 2013 (UTC)[reply]
James Blish's novel A Case of Conscience is about a missionary who to whom these questions are crucial. --ColinFine (talk) 17:23, 5 March 2013 (UTC)[reply]
Yes that's one of the topics that was discussed before--certainly no harm in plugging that most excellent novel again though. See also the wonderful The Sparrow. μηδείς (talk) 19:25, 5 March 2013 (UTC)[reply]
It may be an excellent novel, but if the synopsis in the Wikipedia article on it (A Case of Conscience) is accurate, then it completely misrepresents Catholic teaching and likely issues. 'Moral' beings with no religion are not actually counter to Catholic teaching, since Catholic teaching is that moral law is "written on men's hearts" (that is, natural law exists and can be derived by all) no matter whether or not they believe in God. Alien beings who follow Catholic moral teaching, despite never having heard of God, would be taken as confirmation of Catholic teaching. The Catholic Church isn't one of the groups that expects in-group members to be paragons of virtue, and out-group members to be completely depraved. So, excellent novel or not, if the article is accurate then it is a poor read for the given topic. 86.140.54.54 (talk) 07:52, 6 March 2013 (UTC)[reply]
Wales about 1217

In this image it looks like to me that Llywelyn the Great ruled by his client princes all these lands. Did he also rule the lands in green? How many lands did he rule total (directly or thru his client princes) in 1217?--Doug Coldwell (talk) 14:03, 5 March 2013 (UTC)[reply]

If you read the article you just linked, there's a caption under the picture which states "Green: Anglo-Norman lordships." These would have been lands ruled by "client princes" of the King of England and not Llywelyn. See Marcher Lord and Welsh Marches for more background. There's also a map in the "Welsh Marches" article that shows who controlled what. There are a few differences over some of the lands in the south of Wales between the two maps; this may have been simply a difference between the specific times when the maps are depicting, or perhaps interpretations of conflicting claims. --Jayron32 14:17, 5 March 2013 (UTC)[reply]
Thanks Jayron32. Is Deheubarth, Ceredigion, and Ystad Tywi three (3) different lands?--Doug Coldwell (talk) 14:39, 5 March 2013 (UTC)[reply]
Borders and "lands" are a little fluid and fuzzy around the edges, especially at this time. At its greatest extent, Deheubarth included ALL of southwestern Wales; it is roughly equivalent to the modern land of Dyfed, which historically was only the southwesternmost tip of Wales. Ceredigion (sometimes Cardigan) was the northern part of Deheubarth and at times independent. Ystrad Tywi is roughly south central wales. It was never an independent land, but changed hands frequently between various rulers. --Jayron32 18:47, 5 March 2013 (UTC)[reply]
Thanks Jayron32! I have a lot of studying to do.--Doug Coldwell (talk) 12:26, 6 March 2013 (UTC)[reply]

Patrol 35 or Patrol 36 (Israeli Neo-Nazi organization)

A question was posted at the WP:Teahouse (diff), explaining the name of the article Patrol 35 should actually be Patrol 36. The explanation was:

Some of the references given within this article also make the same mistake, calling the respective group "Patrol 35", so please don't look at just those references cited therein - if one checks the group's name by doing a general search outside of Wiki (e.g., by using Google), one can see that virtually all of the search results come up with Patrol 36. The other important point about Patrol 36 (versus Patrol 35) is that 36 (the number) is twice 18 - in effect, the name "Patrol 36" reflects the fact that it is the second style of group built along the lines of the neo-Nazi group Combat 18 (but is not a scion thereof, such as White Wolves): two times 18 is 36! (Not, of course, 35... To find out why "18" is so central to neo-Nazi ideology, check out Wiki's own article on Combat 18.

Can anybody help on it. (Google shows both[11] and I cannot determine which one is correct)? Thanks···Vanischenu「m/Talk」 14:58, 5 March 2013 (UTC)[reply]

Jpost, Haaretz, and Ynetnews report them as Patrol 36. Patrol 35 might appear in many search results, but it's something else. OsmanRF34 (talk) 15:46, 5 March 2013 (UTC)[reply]
From what I can find, most high-quality sources use "Patrol 36." I've gone ahead and moved the page accordingly, but if someone thinks I'm mistaken, please feel free to revert. Evanh2008 (talk|contribs) 20:59, 5 March 2013 (UTC)[reply]

Thanks a lot to both of you···Vanischenu「m/Talk」 20:12, 8 March 2013 (UTC)[reply]

Making bread and wine 2000 years ago?

How were bread and wine made approximately 2000 years ago? How strong was the wine? How much could one drink before becoming intoxicated? What did the bread taste like? Is it possible to make this type of old-fashioned unleavened bread and wine in a modern-day kitchen? 140.254.226.252 (talk) 15:05, 5 March 2013 (UTC)[reply]

The methods were very similar to the methods that are used nowadays. Yeast is a natural part of the environment; there is no difficulty in acquiring it. Here are the main differences: (1) bread was made using a "starter" -- a kind of fermented batter that was kept always on hand and maintained by adding flour to it each day. Many high-quality breads are still made that way. (2) The flour was generally coarser than the kind you can buy in the grocery store. (3) Wine was made in pottery jugs rather than glass bottles. Most bread was definitely leavened -- people have been making leavened bread since early biblical times. You can easily make old-style bread and wine in a modern kitchen -- all you need is flour or grapes. Looie496 (talk) 15:28, 5 March 2013 (UTC)[reply]
First take 150 gallons of water... - Cucumber Mike (talk) 17:34, 5 March 2013 (UTC)[reply]

To take one geographical area, what was then the Roman province of Syria Palaestina, the mishnah, compiled around 2000 years ago, uses the verb "mix" in place of "pour" when talking about wine. It seems that wine was stored in concentrate and then watered down when served. So to answer your question, extremely strong and not very strong, depending on whether you're talking about before or during consumption. --Dweller (talk) 20:20, 5 March 2013 (UTC)[reply]

I'm always amazed that ancient wine didn't turn to vinegar more often, but after reading our article, I get the impression that it did regularly, and so it was drank quickly. We have 3 articles of interest; the one above, and also a similar one on Rome and Phoenician wine.
The answer to your question is highly dependent on when/where you're asking about. But as far as Greece and Rome goes, ancient wine was flavored with a lot of sweetener from what I can tell. There's also some suggestion that it was quite strong; there's some line about it lighting on fire if held near a flame. How they did that without distilling, I don't know. My guess is it was pretty nasty by today's standards. Shadowjams (talk) 21:41, 5 March 2013 (UTC)[reply]
For Passover Seder, see "Matzo" and "Kosher wine".
For Passover (Christian holiday), see http://wol.jw.org/en/wol/d/r1/lp-e/1200002773?q=memorial+emblems&p=par.
Wavelength (talk) 22:01, 5 March 2013 (UTC)[reply]
Yes, the OP seems to be under the impression that the bread was unleavened. In the case of Passover, it was unleavened because they were in a hurry and didn't have time to wait for the dough to rise. ←Baseball Bugs What's up, Doc? carrots→ 14:42, 6 March 2013 (UTC)[reply]
If there is enough fermentable sugars in the starting liquor then the wine could be as every bit as strong as today (there are some special yeasts available now, specifically for brewing but the the total alcohol content is not much different from wild fermentation – the modern stuff are just more pure strains). Home brewers will know that a good wife could brew excellent wine with a very simple pot and a covering cloth to stop it souring. As for bread. Khorasan wheat, Einkorn wheat and Emmer where used – sometimes with other things added. Eg., Ezekiel 4:9-17: “Take thou also unto thee wheat, and barley, and beans, and lentils, and millet, and fitches, and put them in one vessel, and make thee bread thereof, according to the number of days that thou shalt lie upon thy side, three hundred and ninety days shalt thou eat thereof.” These ancient grains are still available and your supplier will no doubt be able to furnish you with some good home unleavened recipes. The ancients also sprouted some of the grains before grinding, thus producing a kind of unleavened black bread malt loaf. --Aspro (talk) 22:46, 5 March 2013 (UTC)[reply]
As far as I know, the highest [drinkable] alcohol content from live yeast is the Sam Adams yeast they use in Utopias, which comes out at 27%. I guess there's Armageddon (beer), which I just now heard of... but it's technically distilled... by freeze distillation, so I don't think that counts. Does something at 27% abv light on fire? Shadowjams (talk) 22:48, 6 March 2013 (UTC)[reply]
See proof spirit. 100° proof (the value below which gunpowder won't ignite) is 57.15% abv. I'm not sure what the minimum strength for a drink to be flammable is, but I'd expect it to be more than 30%. Tevildo (talk) 01:53, 7 March 2013 (UTC)[reply]

Visas for Catholic bishops?

How do Catholic bishops get the travel documents needed to go to Vatican City if they don't have visa-free access to the EU? Would they get a transit visa from Italy? Visa policy of Vatican City redirects to Visa policy in the European Union, but the latter page doesn't give the relevant info. Indeed, I suspect that this issue gets even more complicated since Vatican City isn't part of the EU or the Schengen Area. Edge3 (talk) 15:37, 5 March 2013 (UTC)[reply]

I doubt that Catholic bishops have any problems traveling to the Vatican. In the worst case, they will have to apply for a transit visa for Italy, but since they have a sound reason to pass through the country, and a stable job, by the grace of god, getting it shouldn't be a problem. Another case would be some third world tourist, who could have problems to obtain an Italian visa. OsmanRF34 (talk) 15:53, 5 March 2013 (UTC)[reply]
You kind of have to assume the oldest continuing government on the planet has a bureaucracy in place to handle these matters ahead of time. μηδείς (talk) 16:25, 5 March 2013 (UTC)[reply]
As they will probably pass through Rome's international airport, it will not take much time to get to the Vatican, and they are very likely to be allowed in, without a visa. The 90-day tourist visa for Italy would be long enough. Even if they needed a visa to get into the Vatican (which most people don't) it would be long enough for their trip. KägeTorä - (影虎) (TALK) 17:17, 5 March 2013 (UTC)[reply]
That's a very interesting question - and one which, it turns out, the Internet has already answered. According to this blog, any possible issues would be quickly smoothed out due to the international magnitude of the Conclave. In particular, "The first issue is that they all need to get to Rome quickly and undoubtedly, many will need visas to go there. Imagine the international outcry if one of the Papabile (favourites for election) were refused a visa to attend the Conclave. Given the global importance of the event, the Italian authorities would never even consider doing this. Were it ever an issue, then it would be open for a Cardinal to fly direct to the Vatican City State as opposed to flying to Rome airport. Given that the Vatican City State is recognised in international law, European or Italian visa restrictions could be easily circumvented.
It would also be possible for the Vatican to issue a passport to a Cardinal having immigration trouble. I have once given advice to a very senior non-European Cardinal about whether it would be easier for him to enter the UK on his Vatican (Holy See) passport or that of his home country. Either way, almost all of the Cardinals will be able to get to Rome to carry out their sacred duties."
Of course, if you actually are a Catholic cardinal travelling to Rome, this should not be taken as legal advice. If you have doubts over your immigration status, please contact The Boss. - Cucumber Mike (talk) 17:28, 5 March 2013 (UTC)[reply]
Cardinals on their way to the conclave. PrimeHunter (talk) 15:35, 6 March 2013 (UTC)[reply]
Wait, what? Unless you're taking a helicopter and landing in St. Peter's Square I'm not sure how you could fly directly to the Vatican City State; unless the Vatican somehow enjoys some extraterritoriality in some nearby airport. The entire country is 110 acres, and there are no landing strips for fixed wing aircraft within it. Eyeballing the two on Google Maps, it looks like Leonardo da Vinci–Fiumicino Airport is actually a bit larger than the territory of Vatican City itself. --Jayron32 18:16, 5 March 2013 (UTC)[reply]
Hmm. Good point. Actually, there is a heliport, as well as a station, though presumably it would still be practically impossible to enter the Vatican without first entering Italy. Nonetheless, the point made by the link I posted is that ways would be found by the Vatican, working with the Italian government, to overcome any potential issues before they occur. - Cucumber Mike (talk) 18:40, 5 March 2013 (UTC)[reply]
There is a difference here between US and European airports. If you have to stop over a US airport, you'll be entering the US (hence, you'll need at least a transit visa). In Europe, for stopovers, you are not required to apply for a visa, you won't be leaving the international area. Therefore, a bishop from any country could travel to the Vatican without any interference by Italian authorities. Although I doubt Italy has even considered blocking legit Catholic bishops of crossing its territory. OsmanRF34 (talk) 19:08, 5 March 2013 (UTC)[reply]
I'd also take issue with the issue of a papabile being unable to get to the conclave. If the cardinals who were there wanted him to become pope, they'd just elect him in absentia. (He would not have been allowed to vote for himself anyway, so his absence would actually mean an even greater vote for him than would otherwise have been the case.) Then try and keep him out of the Vatican, and see how much luck you have. -- Jack of Oz [Talk] 19:09, 5 March 2013 (UTC)[reply]

Thank God, no need to send the hired Swiss guns to raid the Rome Airport to rescue the bishops! Otherwise, this shall make a great Hollywood movie! Ha! Ha! -- Toytoy (talk) 15:01, 6 March 2013 (UTC)[reply]

Nations without a head of state

The above question has got me to thinking: Presumably today, and for most of the past week, the Vatican City has had no Head of state. Depending on the Cardinals, it is also possible that this state could continue for some time. In most countries, heads of state who resign, are removed, or die are replaced either immediately or with little delay. My questions, therefore, are these: What other examples of head-of-state-less countries have there been? (Would Germany following the surrender of Admiral Dönitz be one such?) Do we know what is the longest time a sovereign nation has been without a head of state? And are there any penalties in international law or international relations for not having a head of state? (Note, by the way, that I understand the difference between head of state and head of government - I'm aware that countries can function for extended periods without a head of government; I want to know specifically about the role (mainly ceremonial in some countries) of the head of state. Thanks!) - Cucumber Mike (talk) 18:53, 5 March 2013 (UTC)[reply]

Penalties? Who would impose such a penalty on a sovereign nation? -- Jack of Oz [Talk] 18:56, 5 March 2013 (UTC)[reply]
Presumably he's using the word in the disadvantage sense[12].Dncsky (talk) 19:03, 5 March 2013 (UTC)[reply]
I don't know - that's why I asked! :-) One possibility, according to our HoS article, is that "The Vienna Convention on Diplomatic Relations operates under the presumption that the head of a diplomatic mission (i.e. ambassador or nuncio) of the sending state is accredited to the head of state of the receiving state." I thought, therefore, that maybe a country without a functioning head of state would not be able to send diplomatic missions to other countries. I have no idea who would enforce it though. (And yes, I am using penalties as in 'some advantage that would be due to a country with a HoS that is not due to countries without) - Cucumber Mike (talk) 19:07, 5 March 2013 (UTC)[reply]
Just as a point of order, the Vatican City does have an acting head of state. During Sede vacante, the role of Head of State is officially devolved to a Regency made up of the entire College of Cardinals; the secular administrative roles held by the Pope as head of state of the Vatican City continue unabated, as certain offices maintain a level of continuity. It should also be noted that there are nations that permanently have no singular head of state. Switzerland does not have a head of state; it has a Federal Council that acts as a committee to fill that role. The current situation (where the role devolves to the regency under the College of Cardinals) is not much different than that. --Jayron32 19:26, 5 March 2013 (UTC)[reply]
And Australia is chronically unable to make up its mind as to exactly who is its head of state: see Australian head of state dispute. -- Jack of Oz [Talk] 19:48, 5 March 2013 (UTC)[reply]
Hmm. And Ireland went for 12 years without knowing who was head of state. Nevertheless, they at least knew that someone was - as do Australians - it's just a case of which one it is. I guess that these examples suggest that it's not particularly vital to have a head of state, but in that case why would the Vatican go to the trouble of explicitly declaring what happens to the position in the absence of the Pope? And why can't I find an example of a country without one? (Or two, three or seven. - I'd seen the case of Switzerland; see also the Co-Princes of Andorra, the Captains Regent of San Marino and the Presidency of Bosnia and Herzegovina. None of these countries could be argued not to have a head of state.) - Cucumber Mike (talk) 20:49, 5 March 2013 (UTC)[reply]
Well, it could be argued that for much of the past two decades, Somalia has had no functioning head of state, in any form. The issue is that you're describing a situation that is not logically able to exist. Insofar as a political entity known as a "state" exists, that "state" has certain functions that need to be done. Every state takes a somewhat different take on how to do those functions, but to the definition of state itself contains the functions, the "head of state" is whatever person or persons is tasked with those responsibilities. Just off the top of my head, all states do the following basic functions:
  • Maintain a defense of the national borders
  • Maintain order within the national borders
  • Enforce the laws of the land
  • Establish and maintain relations with other states
  • Maintain a means of trade and commerce (monetary system)
  • Provide symbolic national/cultural unity for the state
  • Pay for doing all of the above (taxation and treasury)
The head of state is traditionally the organ of the state which is responsible for all of the above functions. Now, in no state does a single person do all of these jobs for the entire country. It is impossible. Different countries approach these roles differently. For example, in constitutional monarchies and some countries with a weak presidency (The UK or Israel for example) the official "head of state" only provides the symbolic role, the other roles are split off under the "head of government". In Switzerland, each role is given co-equally to a different member of the Federal Council. In nations with a strong presidency, a single person is ultimately responsible for all of these roles (the U.S. President for example) and heads an administration or government where various departments are tasked with each individual issue, but the President is still ultimately responsible as the sole Head of State. It simply isn't possible to have a functioning country where no one does these jobs, and yet is still a "state". --Jayron32 21:11, 5 March 2013 (UTC)[reply]
For a recent notable example, the office of President of Moldova was vacant for two and a half years from 2009 until 2012. The country had three successive acting presidents during that time. — Kpalion(talk) 21:52, 5 March 2013 (UTC)[reply]
Hmmm... I would think the stance of the Vatican would be that upon the death or resignation of the pope, the position of head of state devolves back to Jesus Christ. Blueboar (talk) 01:39, 6 March 2013 (UTC)[reply]
Do you have any sort of reference or citation for that thought? -- Jack of Oz [Talk] 01:54, 6 March 2013 (UTC)[reply]
It's possible he's being funny. Although if they managed to get Jesus on the Vatican hot line, that would be a major news story. ←Baseball Bugs What's up, Doc? carrots→ 06:14, 6 March 2013 (UTC)[reply]
Well, the Second Coming has been predicted for over 2,000 years. Maybe we're gonna get Jesus Christ returning and becoming Pope Jesus I. -- Jack of Oz [Talk] 07:14, 6 March 2013 (UTC)[reply]
The Pope's assistant dashes into his office and reports, "Jesus has returned and is headed up the stairs right now! What should we do?" The Pope says, "Well, look busy!" ←Baseball Bugs What's up, Doc? carrots→ 13:55, 6 March 2013 (UTC)[reply]

Medieval Iceland had no head of state or indeed any central government during the Icelandic Commonwealth. Any legal disputes that could not be sorted out by the local chieftains, were taken to the annual parliament or Althing. Alansplodge (talk) 10:57, 6 March 2013 (UTC)[reply]

British prisoners of war in WWI Germany

While researching in an archive, I came across some postal cards that were mailed from Karlsruhe to England (presumably via the Red Cross) in February 1918. I'm trying to figure out the postal handstamps; what do they mean? They're all the same, but they're all faint, so I'm not sure that I've transcribed them correctly.

[Top of handstamp] Gelesen und gepräft
[Bottom of handstamp] Offizier-Kriegsgefangenenlager
[Middle of handstamp]F.A.[line break]Karlsruhe

de:Kriegsgefangenenlager is a prisoner-of-war camp, so apparently this is a POW camp for officers. But what does "Gelesen und gepräft" mean, and what would F.A. Karlsruhe be? Nyttend (talk) 20:11, 5 March 2013 (UTC)[reply]

Gelesen und geprüft means "read and checked/examined". --Wrongfilter (talk) 20:14, 5 March 2013 (UTC)[reply]
This site explains "F.A." as Frist abgelaufen, "waiting period expired" — for security reasons, these letters and cards were only sent off 10 days after they were written. --Wrongfilter (talk) 20:19, 5 March 2013 (UTC)[reply]
And Karlsruhe refers to the Offizierlager in the grounds of the Karlsruher Schloss, as opposed to the Durchgangslager in the former Europäischer Hof at 39, Ettlinger Strasse. - Cucumber Mike (talk) 20:31, 5 March 2013 (UTC)[reply]

Amityville Horror

close trolling by indef blocked user
The following discussion has been closed. Please do not modify it.


does anybody know if Father Ralph Pecoraro is still alive? — Preceding unsigned comment added by FMicronesian (talkcontribs) 21:54, 5 March 2013 (UTC)[reply]

According to this site he "is now deceased". --Saddhiyama (talk) 23:07, 5 March 2013 (UTC)[reply]
Invoking the Francisco Franco principle, the OP could check back in a week or two and see if Father Ralph is still deceased. ←Baseball Bugs What's up, Doc? carrots→ 13:47, 6 March 2013 (UTC)[reply]

Searching for a long-lost book

Lately I've been looking up old favorite books from my childhood. Between my above-average memory for all things book-related and the wonders of the modern search engine, few can elude my grasp for long . . . but there's one I just can't remember and it's driving me fairly batty. It's a short novel for readers of late elementary school age, published probably in the 1970s. My copy was paperback; I don't know if it was ever issued in hardcover. I don't remember anything about the author. The title was one word, the name of a dog, not a particularly unusual name but not too common either, and not gender specific. The titular dog is the protagonist of the novel and narrates the story in the first person, addressed to a mysterious "you" whose identity is only revealed in the final paragraphs. The dog, originally a stray, is taken in by a young married couple. They assume their new pet to be male - until the first time they take her to the vet. Over the course of the story, the couple take in two or three other dogs, but all except the protagonist die by the end. The last of the dogs to die is named Happy, who runs into the highway at a rest stop while the family is moving to their new home. At the end of the book, the couple introduce their dog to their newborn baby, towards whom the dog feels an instant rush of love and protectiveness, and the baby turns out to be the "you" to whom the dog has been narrating her life story. How can I go about identifying the title and author of this book? It was so precious to me - the ending moved me to tears. — Preceding unsigned comment added by 172.3.129.135 (talk) 22:12, 5 March 2013 (UTC)[reply]

Would this have been published in the US ? Any chance it would have won a Newbery Medal or Caldecott Medal ? That would certainly limit the search. StuRat (talk) 00:39, 6 March 2013 (UTC)[reply]

Yes to the first question. No to the second. I think it was one of those books that come into the world without fanfare and never get so far as a second printing. I have searched for literally dozens of dimly remembered books and found nearly all of them - I think it's safe to say this one's going to be cussedly obscure. — Preceding unsigned comment added by 172.3.129.135 (talk) 05:03, 6 March 2013 (UTC)[reply]

And the name of the book was a boy's name ? StuRat (talk) 05:46, 6 March 2013 (UTC)[reply]

It was a dog name. Not a human name, and not gender specific. — Preceding unsigned comment added by 172.3.129.135 (talk) 06:22, 6 March 2013 (UTC)[reply]

That's a good clue. There are only so many non-human, common, unisex, dog names, like Rex, Rover, King, Blackie, etc. It might be worthwhile to search for a book under each of those names, like: "children's book title Blackie". StuRat (talk) 22:19, 6 March 2013 (UTC)[reply]

Try here .http://www.whatsthatbook.com/ or here.http://www.goodreads.com/group/show/185-what-s-the-name-of-that-book Hotclaws (talk) 11:14, 6 March 2013 (UTC)[reply]

Thanks. I'll give those a try. — Preceding unsigned comment added by 172.3.129.135 (talk) 16:30, 6 March 2013 (UTC)[reply]

Title: Spunky. Author: Dori Brink. Published 1980. It got to where I was just obsessively poring over lists of dog books. And I didn't actually find it in any of them, but one of them had the word "spunk" and something clicked in my brain and I did a search for "spunky" on Amazon and THERE IT WAS. It's actually back in print, which surprises me because you can look at DOZENS of lists of dog books without finding it. Anyway, I'm happy now. Thanks to everyone who offered advice.

Wow, glad you found it. I'll mark this Q resolved. StuRat (talk) 04:01, 8 March 2013 (UTC)[reply]
Resolved

What happens when a priest plagiarizes a sermon?

What happens when a priest plagiarizes a sermon? Are sermons copyrighted? How does a person cite a sermon for an academic paper? 140.254.226.186 (talk) 22:39, 5 March 2013 (UTC)[reply]

"...several ministers have been fired or severely rebuked for crossing the line", according to "5 Leaders Examine Plagiarism in Preaching". Then there's the prospect of eternal damnation for breaking the commandment thou shalt not steal, even if most sermons aren't copyrighted. Clarityfiend (talk) 22:56, 5 March 2013 (UTC)[reply]
  • Whether or not Intellectual property was recognized in Biblical times, it is clear that "the prospect of eternal damnation for breaking the commandment thou shalt not steal" is a bogus claim. Ryan Vesey 23:02, 5 March 2013 (UTC)[reply]
In regards to citing it for an academic paper, I suggest using Purdue OWL to find it for the specific style you are using. I would presume a sermon should be cited as a speech. Ryan Vesey 23:02, 5 March 2013 (UTC)[reply]
Sermons are 'derivatives' of the words of God. Thus the priest would probably have to agree that his expansive verbiage is copyrighted under the Creative Commons licence of: Attribution-ShareAlike CC BY-SA --Aspro (talk) 23:48, 5 March 2013 (UTC)[reply]
Not true, first the Words of God are not released under a CC BY SA license. A large number of Biblical translations are now PD; however, a possibly larger number of biblical translations are still under copyright. In any case, a derivative of a PD source is not PD or CC BY SA. Ryan Vesey 00:05, 6 March 2013 (UTC)[reply]
Those that are PD (like the original source (providing they are not the King James II version)) are because they are now out of copyright. Yet, if I should render and blend my own interpretation with a current day context ( ie, a sermon) then I hold the copyright to my words and should you reproduce them without attribution I can sue you.--Aspro (talk) 00:21, 6 March 2013 (UTC)[reply]
I said nothing different. I only pointed out that sermons are not creative commons licensed. Ryan Vesey 04:20, 6 March 2013 (UTC)[reply]
For nations under the Berne Convention (most of them) all works such as sermons are copyrighted automatically as far as secular law is concerned. There is nothing legally tricky about that, despite attempts to be clever by other responders above. --Mr.98 (talk) 02:34, 6 March 2013 (UTC)[reply]
I stand corrected. Clarityfiend (talk) 04:16, 6 March 2013 (UTC)[reply]
Tangentally to all the above, I believe (he weaseled) that, up until at least the 19th century, books of pre-composed sermons were printed and published explicitly for priests to use either verbatim or with their own variations. Composing one or several original public addresses of reasonable quality per week would have been a quite daunting task for many clergy hard pressed by other parishional duties. {The poster formerly known as 87.81.230.195.} 212.95.237.92 (talk) 12:13, 6 March 2013 (UTC)[reply]
This still happens, although I don't know that they're always published books. I've certainly heard priests read sermons they had not written, and given the particular priests in question it was usually an improvement (and very clear when they deviated from the written words). 86.140.54.54 (talk) 19:20, 6 March 2013 (UTC)[reply]

"Hips on shoulders, Place!"

"It turned out that Bloom had been relieved from NCO School a week before. The story was that Bloom had been called out of ranks to give calisthenics. His first exercise had begun with the command: "Hips on shoulders, Place!" The platoon of Candidates had immediately degenerated into a disorganized and howling pandemonium. Bloom had been excused and sent back to the ranks. That afternoon he had been relieved." - James Jones, "From Here to Eternity," Chapter 34. I think there's something going on here that I'm not getting. What is it about "Hips on shoulders, Place!" that is so disgraceful and/or hilarious (besides simply sounding weird)? — Preceding unsigned comment added by 172.3.129.135 (talk) 22:47, 5 March 2013 (UTC)[reply]

It's a roundabout phrasing for "stick your head up your ass" (think through the physical implications of "hips on shoulders"). — Lomn 01:55, 6 March 2013 (UTC)[reply]

Makes sense. Should I understand then that Bloom was acting out deliberately? In context, based on my understanding of Bloom's character and the way his fellow soldiers treated him afterwards, I got the impression it was some sort of mortifying blunder. — Preceding unsigned comment added by 172.3.129.135 (talk) 05:06, 6 March 2013 (UTC)[reply]

My reading of it was that he meant to say "hands on shoulders" or "hands on hips" but got in a muddle, issuing a command that was physically impossible. This newsreel sets the scene. Alansplodge (talk) 10:21, 6 March 2013 (UTC)[reply]

Thanks. That makes sense. Poor Bloom. — Preceding unsigned comment added by 172.3.129.135 (talk) 16:27, 6 March 2013 (UTC)[reply]

Tom Short's citation

Tom Short is a campus preacher who frequents college campuses year-round to preach. Often he says roughly the same things on each visit. There was one time when he cited something about so-and-so many civilizations collapsed because of lack of strong something in order to justify the importance of his religion - Christianity - in American society. I am wondering if I may find the exact reference that this guy is referring to. — Preceding unsigned comment added by 140.254.226.186 (talk) 22:47, 5 March 2013 (UTC)[reply]

It's certainly not his idea. Extremist preachers have been saying that for decades. ←Baseball Bugs What's up, Doc? carrots→ 00:21, 6 March 2013 (UTC)[reply]
You should be able to find the exact reference this guy is referring to... if you want our assistance in finding it you would have to tell us a lot more than you have. Blueboar (talk) 01:48, 6 March 2013 (UTC)[reply]

I don't understand the premise. If you made a list of all of the civilisations that have ever existed, it'd be a very long list indeed. The ones that survive today ones are a) very few and b) not all Christian. Furthermore, c) many of the ones that have disappeared were Christian. Every civilisation, Christian or not, that no longer exists, collapsed because of a lack of something. The extreme vagueness of your recollection may be doing this preacher a disservice, or it may be bunkum. --Dweller (talk) 11:05, 6 March 2013 (UTC)[reply]

Those characters used to make the claim, supported by no facts, that the "average" civilization lasts about 200 years at best, and that moral degredation causes it to collapse. That notion was being floated, by an amazing coincidence, around the year 1976. The British civilization, in its current form, has been around for nearly 1,000 years. The Indian civilization far longer, and they're primarily polytheists. Oops. :) ←Baseball Bugs What's up, Doc? carrots→ 13:45, 6 March 2013 (UTC)[reply]
If "moral degredation" (seen through theist spectacles) causes a civilisation to collapse, it's a wonder there are any existing civilisations. --Dweller (talk) 14:40, 6 March 2013 (UTC)[reply]
Amazing, ain't it? That, along with the fact that each successive generation is worse than the previous one - according to the previous one's members, at least. ←Baseball Bugs What's up, Doc? carrots→ 14:48, 6 March 2013 (UTC)[reply]

"The world's great civilizations have progressed through this sequence: From bondage to spiritual faith; from spiritual faith to great courage; from courage to liberty; from liberty to abundance; from abundance to selfishness; from selfishness to complacency; from complacency to apathy; from apathy to dependence; from dependence back again into bondage." Attributed to Alexander Tyler [13] but our article says that the first record of "The Fatal Sequence" was a speech by Henning Webb Prentis, Jr., President of the Armstrong Cork Company in 1943. Alansplodge (talk) 17:20, 6 March 2013 (UTC)[reply]

This is the best I could find, extending Alansplodge's contribution. 140.254.226.228 (talk) 19:49, 6 March 2013 (UTC)[reply]

oh hey just an update that i was right and the reference desk was wrong on the negative net worth thing

so it seems that if you have no debts and $10 in your pocket, you have more net worth than 25% of americans. this was not given by the reference desk, although i explicitly asked.

so just to give an update, here is fortune saying the same thing. http://www.forbes.com/sites/timworstall/2011/12/14/six-waltons-have-more-wealth-than-the-bottom-30-of-americans/

it would have been nice to get a correct answer like that, but i do appreciate that you tried, and are volunteers. My question on the utility of money still did not go answered though - i am genuinely interested how the utilty of all americans compares, including the lowest, highest, and their distributions, on a logarithmic scale. 178.48.114.143 (talk) 23:10, 5 March 2013 (UTC)[reply]

Keep in mind, the old accounting equation: Assets = Liabilities + Capital. ←Baseball Bugs What's up, Doc? carrots→ 00:27, 6 March 2013 (UTC)[reply]
First of all, the youtube video used 2005 wealth data. The Forbes article used 2009 wealth data (page 5, table 2). You would know this if you done your homework. :)
Now looking at the 2009 data, note the lowest bottom four-fifths own -1.4% (negativity net worth). The actual data set: [% of population, % of wealth] [0 ~ 19, -1.4], [20 ~ 39, 0.3], [40 ~ 59, 3.3], [60 ~ 79, 10.6], [80 ~ 89, 12.2], [90 ~ 94, 11.6], [95 ~ 99, 27.9], [99 ~ 100, 35.6]. So even with a 4 year differences the graph from the youtube video (@03:53) still roughly match the shape of the 2009 wealth data (difference is that -1.4% of wealth for the lowest 20% of the population is not there).
Chart fundamentally wrong? I think not.
As for the utility part of the question, call me petty but I don't think I want to help you. :P Royor (talk) 04:08, 6 March 2013 (UTC)[reply]
Hmm ... the numbers seems a bit dry. To help you visualize, let say a jar of delicious Nutella equal 100% of U.S net worth. One piece of bread represent the entire U.S population. Now cut off one fifth of that bread. There is no Nutella on that piece, in fact it will supply the other pieces an extra 1.4% of a jar of Nutella to the other pieces. Now take a bread crumb (perhaps a small piece of the edge) (the 1%) and spread a bit more than 1/3 of the entire jar of chocolate on it, and so on. There you go, enjoy your sandwich. Royor (talk) 04:48, 6 March 2013 (UTC)[reply]
The lowest quintile of Americans has had negative net worth for the past 50 years[14], so it doesn't matter if the video used the 2005 data or the 2009 data. The video showed a slightly positive net worth for the lowest quintile for illustrative purposes only, since it's apparent that the lowest quintile and the second lowest quintile are shown with exactly the same amount of net worth[15].Dncsky (talk) 05:35, 6 March 2013 (UTC)[reply]
True, but the original question was whether negative net worth would skew the graph. With a -1.4% area (for 2005 it would probably be -0.5%) under the x-axis spread over from 0 ~ 19 (translation: a sliver of a line drawn under the x-axis for the bottom 20%) vs a 101.4% (100.5% for 2005) area above the x-axis the graph is in no way "fundamentally wrong". Royor (talk) 06:00, 6 March 2013 (UTC)[reply]
Hi, OP here. This is what I mean: the chart implies, though does not state, that those people near the x-axis have practically no utility, maybe the air they breath but that's it. It evokes imagining a crumb. But if we saw the people who enjoy less than no net wealth, we quickly see that they must still breathe and eat something, otherwise they would have died years ago. So, the nutella example shows the truth: if nutella were utility, you wouldn't have negative nutella on anyone, because they would not even be alive if they didn't enjoy any utility at all. The real truth is this. We should all be clear that a gourmet meal is logarithmically several times better than a $1 bag of chips which the person might not even want to eat, knows is unhealthy, but is the only thing they can afford. But if the gourmet meal cost $249 at a great new york restaurant, only linearly can it be considered to grant "249 times as much utility" - and, in fact, due to diminishing marginal utility maybe not even linearly. I am sure some people eating a gourmet meal would have preferred junk food - chips - at some points in time. So what I am saying is that the curve would look like that: http://i.imgur.com/uSNPbfa.jpg and I would like to have your help determining the distribution of utility at the right. 91.120.48.242 (talk) 08:34, 6 March 2013 (UTC)[reply]

March 6

Chavez death international reaction

So far, Argentina, Cuba and Ecuador have proclaimed 3 days of national mourning, and even the president of Bolivia Evo Morales broke down in tears.Sometime in modern history, the death of some president provoke such a international reaction, including national mourning in several different countries?. — Preceding unsigned comment added by CubanEkoMember (talkcontribs) 03:01, 6 March 2013 (UTC)[reply]

JFK ? Not mourned quite so much in Cuba, though. StuRat (talk) 03:03, 6 March 2013 (UTC)[reply]

Yitzhak Rabin might work as well. As for non-assassinated leaders, maybe Ronald Reagan (just a guess). Futurist110 (talk) 04:23, 6 March 2013 (UTC)[reply]

After the death of Kim Jong-il, Cuba declared official mourning. --PlanetEditor (talk) 04:38, 6 March 2013 (UTC)[reply]
When Australian Prime Minister Harold Holt disappeared while swimming in the surf in 1967, we started inventing rumours. They haven't stopped yet. HiLo48 (talk) 04:54, 6 March 2013 (UTC)[reply]
Don't forget the swimming pool named after him! [16] Nick-D (talk) 10:40, 6 March 2013 (UTC)[reply]
It would speak very poorly of a sitting head of state if his death goes unnoticed in the international arena. The death may or may not generate political changes, but international people sending their condolences, that's for sure. Cambalachero (talk) 12:21, 6 March 2013 (UTC)[reply]
For us, here in Cuba and Latin America(including head of state), Hugo Chavez was and always will be more than just a president, is a symbol of broterhood between our countries and people, and specially the cuban people has Chavez like his own son, and our pain is much more than just a formal condolence message, you know that, Cristina Fernandez was a close friend of Chavez. CubanEkoMember (talk) 12:39, 6 March 2013 (UTC)[reply]
The problem with places like Cuba, Venezuela, and any number of other countries, is that cults of personality inevitably come to an end, and then there's the risk of a period of chaos. In countries with a "system" in place, there's less risk of that chaos occurring. The presidency is merely a "job", held for a limited amount of time, while the "system" keeps going. ←Baseball Bugs What's up, Doc? carrots→ 13:39, 6 March 2013 (UTC)[reply]
How can we know you are telling the truth? According to Internet in Cuba you have reasons to be weary about what you say. OsmanRF34 (talk) 13:49, 6 March 2013 (UTC)[reply]
I would say that it's actually that the Castros are weary, and the citizens are wary. :) ←Baseball Bugs What's up, Doc? carrots→ 13:52, 6 March 2013 (UTC)[reply]
While I can sympathize with his socialist and anti-colonial leanings, his anti-democratic/authoritarian leanings, as demonstrated by his participating in a (failed) military coup, ending the independence of the courts, and shutting down opposition media outlets, I can't abide. Had he stayed in power, he might have managed to establish a dictatorship. Hopefully, with him gone, democracy can be restored. I hope the same for Cuba, after the Castro brothers die. StuRat (talk) 21:05, 6 March 2013 (UTC)[reply]
I'd be curious to know if there's ever been a Communist-oriented government that wasn't a dictatorship. ←Baseball Bugs What's up, Doc? carrots→ 22:18, 6 March 2013 (UTC)[reply]
Well, Chávez hasn't made it onto our list in the Dictator article, possibly because he has submitted himself to four presidential elections and two referenda. He came close to losing the last election, so there must be a some element of fairness about them. In answer to your question, the administration of Salvador Allende comes to mind, and who knows where the Prague Spring might have led? Alansplodge (talk) 22:55, 6 March 2013 (UTC)[reply]
Communist is a bit strong for Chavez. After all, it's not like he prohibited the private ownership of property. I'd call him socialist, and there are some fairly socialist strong democracies, such as the Nordic nations.
But now I have to fulfill Godwin's Law by pointing out the comparison with Hitler. Both led an unsuccessful coup, both were later elected, both then set about dismantling democracy, both had unsavory dictators as friends, and both hated Jews. StuRat (talk) 22:42, 6 March 2013 (UTC)[reply]
I've just looked at the Wikipedia article that you linked to, and note that "...once such a comparison is made, the thread is finished and whoever mentioned the Nazis has automatically lost whatever debate was in progress", with which sentiment I wholeheartedly concur. I bid you all goodnight. Alansplodge (talk) 23:02, 6 March 2013 (UTC)[reply]
Fact, fact, highly sketchy, yeah I'd agree, super mega highly sketchy.
Are there any savory dictators?  :) -- Jack of Oz [Talk] 23:00, 6 March 2013 (UTC)[reply]
Certainly, if properly cooked and seasoned. StuRat (talk) 23:02, 6 March 2013 (UTC) [reply]
Dictators are often at their most savory after they've been shot. Like Mussolini and his pals, who were no longer empowered, but hung around with their appreciative subjects for a while. ←Baseball Bugs What's up, Doc? carrots→ 23:07, 6 March 2013 (UTC)[reply]
The problem is that the system itself is a dictatorship, the dictatorship of the capital, where the people like you say just play a minor role,just a merely job imposed by capital logic.I'm not wary or anything of that, maybe here in Cuba we have some censorship, excessive control on some internet conections and other issues that need solutions and open debate, and we're working on it Cuba has changed a lot in last years, but i'm free to say whatever i want as you can see, i'm not afraid that's stupid...the death of Castros will not stop the way Cuba is: a socialist and martian country.Now note that 11 countries have proclaimed national mourning, is a record i'm sure, it seems like the dictator is popular.CubanEkoMember (talk) 01:07, 7 March 2013 (UTC)[reply]
History demonstrates that Communism, or extreme socialism, is an unsustainable system, and eventually it works its way back towards capitalism. Once the Castros are gone, the inch-by-inch changes in liberating Cuba might accelerate. I say "might" because no one knows how their next leader will operate. The Castros dug a deep hole for their country when they allowed the USSR to build missile bases there (not that they had any real choice in the matter), and they've paid the price ever since. Once the Castros are gone, hopefully the punishing sanctions will be eased, and then Cuba will have a chance to prosper again. ←Baseball Bugs What's up, Doc? carrots→ 13:10, 7 March 2013 (UTC)[reply]
Communism ended about 75 years after it started in Russia, and about 40 years after it started in China (although they retain the dictatorship, and Russian democracy is somewhat suspect, too). So, based on those examples, I'd expect communism to be on it's last legs in Cuba by now. I believe there's actually quite a bit of capitalism right now, where waiters who cater to foreign tourists make far more money than doctors working for the government, under the old communist system. Such bizarre inequities can't last long, or all doctors will quit to become waiters. Ironically, if the US drops it's sanctions, the rush of US tourists might just push over the house of cards. StuRat (talk) 03:37, 7 March 2013 (UTC)[reply]
Other countries supposedly mourning the loss of Chavez reminds me of the old saw that when a mob boss dies, all the rival mob bosses send big bouquets to the funeral. ←Baseball Bugs What's up, Doc? carrots→ 13:04, 7 March 2013 (UTC)[reply]
Cuba is My Favorite Martian country. Edison (talk) 03:54, 7 March 2013 (UTC)[reply]

Problems

Disregard

[17] , [18] , [19] . First of all, this is English Wikipedia. Please write your comments in English. I don't speak Russian. I would like to ask you to read the WP:NFCC|non-free content criteria and in particular WP:NFCC#1|criterion 1. Files like :File:Doc balt flot1.jpg are not permitted because someone else can draw a freely licensed map of the same area. There are already freely licensed maps of all parts of the world. For example,Openstreetmap can be used for this purpose. Photos like :File:SMX-25 - Diving frigate.JPG are not permitted because it seems that vehicles of the same model still exist. It is possible to take other photos of the same vehicle model and publish those photos under a free licence. For example, see WP:NFC#UUI §1 which says that you can't upload unfree photos of buildings which still exist. Stefan2 16:05, 5 March 2013 (UTC) File:Подземоход Требелева.jpg ? Podzemohod Trebeleva was tested in the Urals, Mount Grace, in 1946. Trebelev intended to use his podzemohod in various fields: digging tunnels for urban communication, exploration, mining, etc. However, the design proved to be unreliable, and the project was abandoned. - Now it does not exist. File:SMX-25 - Diving frigate.JPG - it does not exist at all. It is only on paper. SMX-25 a gunship project of the 21st century, a hybrid of surface ship and submarine. it seems that vehicles of the same model still exist. - there is no such ships. How can redraw what does not exist outside the project on paper? File:Doc balt flot1.jpg - It is not a geographical map, it is map of the military facilities. This is the result of several experts to repeat that an outsider can not. Vyacheslav84 (talk) 04:18, 6 March 2013 (UTC)[reply]

It's very hard to read all that and get a question out of it. Please state your question in one short sentence. StuRat (talk) 05:43, 6 March 2013 (UTC)[reply]
I think what happened is that OP posted File:SMX-25 - Diving frigate.JPG with a non-free fair-use tag, and then User:Stefan2 disputed the fair-use claim by invoking Wikipedia:CSD#F7. They're currently resolving this on Stefan2's talk page. In any case this matter does not belong on the RD at all. Dncsky (talk) 05:57, 6 March 2013 (UTC)[reply]

How to pronounce "bethsaida"

trying to memorize part in play, have not been able to satisfactory pronounce, bethsaida — Preceding unsigned comment added by 67.170.164.76 (talk) 06:35, 6 March 2013 (UTC)[reply]

I added a header for you.Dncsky (talk) 07:22, 6 March 2013 (UTC) [reply]
Using the IPA pronunciation given at Bethsaida, the best simple transliteration I could give you is "beth-say-ee-duh" the last vowel sound isn't actually the u sound, but rather a Schwa you can see a few examples at Schwa#DescriptionRyan Vesey 07:27, 6 March 2013 (UTC)[reply]
Perhaps: beth-sah-he-dah? Plasmic Physics (talk) 10:53, 6 March 2013 (UTC)[reply]

What nationality are you playing in the play? That might affect things. Also, is it important to the audience's understanding that all the actors say it similarly, so they don't get confused? In which case, you should get consensus from the relevant other actors. --Dweller (talk) 11:00, 6 March 2013 (UTC)[reply]

Right, I was in a childrens theatre back in the day where the directors acted as well. They always used accents and whatnot without having any of the other actors use an accent so it was incongruous with the rest of the play. Ryan Vesey 15:59, 6 March 2013 (UTC)[reply]

maybe a weird question

idea derrived from a legal copy of a copyrighted work is not covered by copyright. how about idea derrived from a pirated copy? please answer without using other's statement typed in exact manner.121.97.111.151 (talk) 13:07, 6 March 2013 (UTC)[reply]

Ideas are not copyrighted, period. The source of the idea is irrelevant. One can suffer a fine for possessing, creating or distributing illegal copies, but not for deriving ideas from them. - Lindert (talk) 13:16, 6 March 2013 (UTC)[reply]
As long as they don't rip off too much of the source and try to claim it as their own. "Ah, my Lord / Doo lang doo lang doo lang / My sweet Lord / Doo lang doo lang..." ←Baseball Bugs What's up, Doc? carrots→ 13:57, 6 March 2013 (UTC)[reply]
I'd caution against saying that you can't get in trouble for deriving ideas from a copyrighted work. If you're sufficiently inspired and create another work, it could very well be considered an unauthorized derivative work. ~ Amory (utc) 15:19, 6 March 2013 (UTC)[reply]
Yes the line between an uncopyrightable idea and the copyrightable expression of that idea isn't always entirely clear cut. The OP must know this since it's been explained to them at least once in the 5 or so times they've asked highly similar questions about this area (or ranted about the unfairness). The OP's specific question here appears to be whether there's something unique about ideas derived from sources viewed without permission of the copyright holder when it comes to their protection by copyright. The answer here is no, it's the same as we've explained to them every single other time they asked or ranted, ideas themselves can't be copyrighted. Nil Einne (talk) 16:24, 6 March 2013 (UTC)[reply]

supreme court opinions

do supreme court justices write their own opinions with their own two hands or can clerks write it for them per their instructions, as is common with other similar opinions?

has a supreme court ever produced 9 written opinions for a decision? (regardless of its vote). 91.120.48.242 (talk) 13:20, 6 March 2013 (UTC)[reply]

Which one of these many Supreme Courts were you thinking of? -- Jack of Oz [Talk] 18:36, 6 March 2013 (UTC)[reply]
Too lame for words
The following discussion has been closed. Please do not modify it.
we've had this discussion before, you are quite aware that America is the default standard and only when other countries are involved do they need to be specified. μηδείς (talk) 18:44, 6 March 2013 (UTC)[reply]
Your imperialism stops at your national borders, and a good thing too. AlexTiefling (talk) 19:02, 6 March 2013 (UTC)[reply]
As compared with Britain, whose didn't, but effectively does now. ←Baseball Bugs What's up, Doc? carrots→ 22:16, 6 March 2013 (UTC)[reply]
How justices write their opinions depends on the justice. Clerks are generally very involved in the process of opinion writing, but the extent, especially on the SC, is highly dependent on the individual. Here is an article on some of the recent justices. Shadowjams (talk) 23:01, 6 March 2013 (UTC)[reply]

Hi, the question is about the united states. Could you express whether there are any referenced cases where instead of a single unanimous opinion everyone signs on, or a single opinion and dissenting opinion a divided court signs on, there are as many opinions as possible because the justices don't fully agree with the appointed majority opinion writer and/or minority opinion writer (regardless of their votes)? Basically, I know there are cases where there are three opinions: a majority opinion for example and two differing opposing opinions. Are there cases where there are more total written opinions for 1 decision? (i.e. 4, 5, 6, 7, 8, or 9)? Thanks. 91.120.48.242 (talk) 07:49, 7 March 2013 (UTC)[reply]

The line would be are there any non per curiam 9-0 opinions since 1938 or thereabouts... the answer is yes... i know of a few but I can't name them off the top of my head. Hopefully someone else will answer you here, although I suspect if you google for the phrase I just used you'd find them. Protip: most come out of the 9th circuit. Shadowjams (talk) 12:31, 7 March 2013 (UTC)[reply]
Apparently the majority of SCOTUS opinions are 9-0. http://sblog.s3.amazonaws.com/wp-content/uploads/2012/05/SB_votesplit_053112.pdf Shadowjams (talk) 12:34, 7 March 2013 (UTC)[reply]
Guys, correct me if I'm wrong but can't there be two written opinions even in a 9-0 vote? A majority opinion and a justice differing on at least some points of it? Therefore my question can have a response from any of those columns! The question is what is the most number of textual written opinions (regardless of the vote) that justices have produced? The minimum is two: any decision that is not unanimous must have at least one supporting and one opposing decision. But it also could have two minority/opposing/dissenting opinions (raising the number of written opinions for that case to 3), or for any other reason and regardless of the vote there could be 4, 5, 6, 7, 8, or 9 opinions. The only posssibility that does not exist is any more than 9 opinions, since one justice cannot write two opinions for the same case. So out of the possible 2, 3, 4, 5,6, 7, 8, or 9 differing opinions I know that the number "3" exists: does 4, 5, 6, 7, 8 or 9 exist? (Meaning this many justices chose to write an opinion.) 91.120.48.242 (talk) 14:39, 7 March 2013 (UTC)[reply]
"Majority" opinions are those in which the judges (SCOTUS or otherwise) all agree that it's the majority opinion. "Plurality" opinions are those where judges agree in decision, but write separate opinions, usually because they disagree on some central point. Shadowjams (talk) 19:39, 7 March 2013 (UTC)[reply]

% of UK people who know who the Prime Minister is

Hello, I am trying to find information about what percentage of the UK population know the name of the Prime Minister, Leader of the Opposition, Chancellor of the Exchequer, etc. Someone told me only 50% of the population know who the PM is, but I can't find any evidence or surveys for this quite remarkable statistic. Can anybody help? 86.26.225.187 (talk) 15:34, 6 March 2013 (UTC)[reply]

A companion question could be, what percentage think that it matters, i.e. that it has any direct effect on them. ←Baseball Bugs What's up, Doc? carrots→ 15:35, 6 March 2013 (UTC)[reply]
I'd be interested in the same percentages with the US. Without checking, I'm thinking it's David Cameron, but wasn't some type of a coalition formed since no party got a simple majority? What happens to the leader of the other party in the coalition, is there a vice PM position? I'd assume that the percentage of US citizens who care about who the PM is would be less than 10%. Ryan Vesey 16:05, 6 March 2013 (UTC)[reply]
The leader of the other party in the coalition is Nick Clegg, who is the Deputy Prime Minister. --TammyMoet (talk) 16:25, 6 March 2013 (UTC)[reply]
...though that's not an automatic appointment in such (rare) circumstances, but came about as a result of the Coalition agreement. There are details of some results for the US here, showing how things have changed over time. I can't find any similar results for the UK. (There's a story (can't find links) that "who is the Prime Minister?" used to be a question used to test elderly patients for dementia, but it started to become useless when Margaret Thatcher had been PM for a while, because everyone knew who she was.) AndrewWTaylor (talk) 17:01, 6 March 2013 (UTC)[reply]
Not quite what you asked for, but one polling company did a survey in 2008 asking whether people would be able to recognise some potential leadership challengers to Gordon Brown if they saw them in the street. [20] 79% claimed they would be able to recognise then-Justice Secretary Jack Straw. Hut 8.5 17:31, 6 March 2013 (UTC)[reply]
There's something similar from 2010 here. They showed pictures of politicians to people and asked them to pick a name from a list. 96% of people identified David Cameron. Hut 8.5 18:49, 6 March 2013 (UTC)[reply]
Ah, but many of the main characters in the cabinet after the 1997 election were at very-nearly-approaching Margaret Thatcher recognition, in a way that I think only David Cameron (and possibly Nick Clegg) now approach. Jack Straw is one of them. A lot of this is personality and publicity based, rather than a general rule about how many people recognise deputy prime ministers in general, and so on. I'd be surprised if most people (more than 50%) could identify anyone in the current cabinet beyond Cameron and Clegg. 86.140.54.54 (talk) 19:09, 6 March 2013 (UTC)[reply]

I found this 2003 report which says that "47% of respondents could not name the deputy prime minister (John Prescott at that time). This ignorance rose to 73% among the 16-to-24 age group." Alansplodge (talk) 17:34, 6 March 2013 (UTC)[reply]

As far as the question about Americans recognizing the British PM: [21]. A surprising 54%. That's misleading though because the poll was in 2006 and the PM was Tony Blair, which had a lot of visibility during the Iraq War, so that would explain the high figure. Shadowjams (talk) 22:55, 6 March 2013 (UTC)[reply]

Paternalistic bureaucracy

Resolved

A friend of mine recently got a company car, which he thought was a nice perk, until he heard the catch. The company would monitor his entire driving performance, including telling him off if he braked too hard. All of this information would be captured by computer, relayed to a central server, and analysed for driving errors or abnormalities. This kind of paternalism seems very common nowadays, from the group conversation that followed. Nearly everyone seems annoyed at it. It is clear enough that the company is covering itself against lawsuits, and it is expected that there will be some bureaucracy in such situations. But since very few people seem to like this kind of invasion, how did it get this bad? Have any politicians tried to tackle the problem, apart from a few extreme libertarians? The mainstream opinion seems to resent the intrusion, so why does the mainstream of politics not respond? I know it's hard to answer "why" questions here, but any references or knowledge about public debates would be interesting, eg. documentaries about the nature and causes of the problem, and what conclusions came from such documentaries. IBE (talk) 17:04, 6 March 2013 (UTC)[reply]

I'm confused why you think this solely has to do with lawsuits. It seems to me there are many reasons why a company may want to monitor their employees driving and lawsuits is only one of them. Nil Einne (talk) 19:25, 6 March 2013 (UTC)[reply]
I suggested lawsuits precisely because I myself cannot think of any other reasons for this interest. Can you tell me, briefly, just three other reasons? I can imagine in a far fetched way that they might be doing it out of the goodness of their hearts, but I would find the effort rather extreme, and the involvement is unwanted on the part of this employee. Hence, I would find benevolence to be a somewhat strange explanation in this case. IBE (talk) 19:59, 6 March 2013 (UTC)[reply]
Well no one ever said anything about benevolence, mostly it will be about financial cost (direct and indirect), but lawsuits are only one factor.
If the company is paying for fuel, then driving style will affect fuel consumption. This may not be a concern if there's only a few cars involved with minimal driving but likely would if there are a lot of vehicles. (In the case of say a delivery, trucking or bus company, the savings can probably be quite substantial, there's even an advert about it from the government's EECA Energywise here in NZ that I've seen before and funnily enough the main page has something about driving economically [22]. I presume 'company car' doesn't mean a van, truck or bus, but such a company may put the same things in their company cars to avoid perceptions of unfairness.) Poor driving will also likely increase maintenance costs for the vehicle.
Poor driving can lead to accidents. Presuming the employee isn't completely responsible for the car, in many cases, the company may either have insurance to cover these or has sufficient reserves that they feel they don't need it. But either way, more frequent accidents will likely lead to higher costs. In fact, the insurance company may be willing to lower premiums if the company has a programme in place to monitor and improve driving. You may try to argue that technically if there are no lawsuits they don't actually have to pay anything even if their employee is totally at fault for the accident. But concentrating at the lawsuit bit misses the point that in the real world most of the time, the company don't even want to hear the word 'lawsuit'. If it's clear the employee is at fault, they will often pay (or their insurance will pay) without getting the courts involved. (Of course if a lawsuit does arise or there is a dispute, the data may be useful in assessing who is correct.)
There may also be criminal charges to contend with, rarely will these involve the company directly (although the PR and downtime may come up) but there may be a minor risk of the employee saying their poor driving was encouraged by the company. In addition, accidents and some forms of poor driving when the employee is working (and for some accidents and poor driving even when they're not) will often mean downtime for the employee and possibly other employees who will have to go to help. These may mean dissatisfied customers/clients, partners or suppliers and other performance failures on the part of the company. Besides that, accidents or simply poor driving (including tickets) may lead to bad PR if the company is identified. Somewhat related but for some companies poor driving may put contracts at risk.
In addition, if there is a substantial amount of driving involved, reasonable performance in that task may be a key job criteria. Even if you have some evidence that the employee is not up to scratch, e.g. their fuel usage is a lot higher, they show up later then expected, (probably not for your friend's case but the cars mileage is a lot higher then expected), having more info (and a well developed policy) will help in managing that. Yes that means reducing the risk of and from a lawsuit if you fire the employee but a company will often also be interested in improving the performance if they can since it may cost less then finding another employee even without a lawsuit (and then there are the cases where it's not worth firing but the better would help).
You can also come up with other random reasons. E.g. if it's a small private company, perhaps the owners have moral reasons (i.e. even if it didn't reduce costs including the risk of bad PR) for wanting their employees to drive well, particularly with company cars. Perhaps the owner's child was killed by a speeding driver.
I have no idea how much of this is likely to apply to your friends case since I know too few details. But note again what I said earlier about delivery vehicles, trucks and buses may also apply in other cases. E.g. if there are non assigned company cars which get regular usage, they may want the monitoring and for reasons of perception etc may put the same thing in company cars provided to managers rarely used on company business.
P.S. So this isn't complete OR a simple search will find sources discussing or offering fleet monitoring and a quick look suggests these mention what I mentioned. [23] [24] [25].
P.P.S. Somewhat related the privacy commissioner of Canada discusses some possible reasons for GPS tracking of vehicles [26] [27] [28].
Nil Einne (talk) 21:10, 6 March 2013 (UTC)[reply]
Short summary, based on the references: for meeting legal responsibilities under OHS legislation (the pdf from Victoria, Australia), employee productivity (for workers in the field taking a detour to their favourite bar), customer service (eg. when exactly will your taxi arrive?), and vehicle maintenance. I can see that I didn't give detail in the question, but most of these do not seem to apply, and as stated, things like excessive braking are covered, so it seems to be either safety or maintenance. The person is not out in the field on sales calls. From the conversation, it also didn't sound like maintenance was the issue. So a large part of it is either indemnity against lawsuits, or a similar legal issue of meeting OHS regulations. This is what I am primarily interested in, since this kind of paternalism seems widespread. From what I have found in conversation, such surveillance is rather unpopular, to put it mildly. IBE (talk) 22:25, 6 March 2013 (UTC)[reply]
The person may not be involved in any of those, but as I said if other people using company vehicles are then it's possible perhaps even likely the company may want to introduce such monitoring to all their cars to avoid perceptions of unfairness etc. Having one policy for the bosses and one policy for the 'plebs' is likely to be even more controversial then simply applying the policy to everyone. BTW, I wouldn't exactly trust a conversation with someone who is obviously aggrieved with the policy to accurately represent (or possible even know) the reasons for the policy. Also, excessive braking is one factor which will increase fuel consumption. Excessive braking may mean things like tail-gating etc which beyond being a safety issue, could easily be a PR issue as I mentioned. PR risk is less likely if the car isn't clearly identified as belonging to the company, but it's still some risk which most companies will prefer to avoid (whether or not it's worth monitoring may be a different issue). Nil Einne (talk) 06:47, 7 March 2013 (UTC)[reply]
Who owns the car? ←Baseball Bugs What's up, Doc? carrots→ 22:15, 6 March 2013 (UTC)[reply]
A company car is always owned by the company. I am not disputing whether it is actually legal for them or not (ie. legal for the company to collect the data). I am curious about the fact that (or my impression that) this kind of paternalism is highly unpopular, yet the government seems to allow it. The government could, for example, tone down the laws relating to personal injury, or OHS, whenever it is clearly the employee who has endangered himself, without anyone else in the workplace being put at risk. I understand that in driving, other road users are involved, but this is generally a police matter, not a workplace safety matter. Companies don't generally focus heavily on things their employees do to others, since those things are left to the authorities. Furthermore, we are not talking about just obeying road rules, but monitoring of all sorts of things like braking. IBE (talk) 22:25, 6 March 2013 (UTC)[reply]
Well, a company car could be leased from another company, but as you imply, the company is responsible for the car, and it certainly seems it's within their rights to manage their fleet as they see fit. ←Baseball Bugs What's up, Doc? carrots→ 23:04, 6 March 2013 (UTC)[reply]
Which isn't the point of the question, but anyways, it is not legal under all circumstances, as Nil Einne's refs show. There are legal limits, when it comes down to managing the people more than the cars. IBE (talk) 23:16, 6 March 2013 (UTC)[reply]
There may be legal limits, but there's no constitutional right to drive someone else's car any old way you want to. They might be concerned about liabilities, they might also be concerned about excessive wear-and-tear on the car, or even about their drivers driving unsafely (as in drunken). Someone who fights the rules too much might find themselves having the car taken away from them. This may not be exactly the same thing, but it's a bit like people who gripe when a company asserts its right to read employee e-mails. The common ground is that if you misuse company property, you can be held accountable for it some way or another. ←Baseball Bugs What's up, Doc? carrots→ 23:21, 6 March 2013 (UTC)[reply]
Pst... Bugs.... he's in Australia. Shadowjams (talk) 03:12, 7 March 2013 (UTC)[reply]
I'd be surprised if Australians have a constitutional right to drive someone else's car. ←Baseball Bugs What's up, Doc? carrots→ 03:16, 7 March 2013 (UTC)[reply]
No, we don't. We have a different rule here - "You Toucha My Car I Breaka You Face". -- Jack of Oz [Talk] 03:44, 7 March 2013 (UTC)[reply]
That works too. Presumably that's the car's owner talking - i.e., the company. ←Baseball Bugs What's up, Doc? carrots→ 12:55, 7 March 2013 (UTC)[reply]
Jack, what's a matter you, eh? Gotta no respect. As far as the constitution goes, we don't have a bill of rights or anything like that, so yes, it did sound US-centric, but I got the gist. The Australian constitution is in fact pretty boring, and just concerns having regular elections. The Australian version of the Gettysburg Address would go something like "Five score years and then some ago, our nation was founded on the principle that we should have elections, that they should be about every three years or so, and that we should take the piss out of ourselves more than anyone else on earth". IBE (talk) 15:51, 7 March 2013 (UTC)[reply]
And that's a fine example of auto-piss removal. But in case you haven't read it in a little while, the Australian Constitution is about a whole lot more than just elections. As for boring, what did you expect? Maybe a joke every second paragraph ("By the way, did you hear about the actress and the bishop? ...")? -- Jack of Oz [Talk] 19:36, 7 March 2013 (UTC)[reply]
Yes, I knew I was exaggerating a little, although as a young chap when we had the big referendum on the monarchy, or on what we thought of James Blundell or something, I was rather surprised at how sparse the constitution was. IBE (talk) 19:46, 7 March 2013 (UTC)[reply]
One point you seem to be missing is stuff like 'excessive braking' may indicate poor behaviour on the road like tail-gating (at least one of the refs I provided mentioned this if it isn't obvious) therefore it is easily a 'road rule' issue and unless the company wants to install camera etc and monitor these, ultimately all they will have is computer algorithms indicating poor driving behaviour which may amongst other things, violate road rules. I'm not sure I understand your point about 'Companies don't generally focus heavily on things their employees do to others, since those things are left to the authorities'. I don't know what your experience is or about companies in Australia in particular, but most companies have quite an interest in ensuring their employees behaviour towards others while working is good, way more then simply not being a legal violation because of the PR implication (including customer/client, partner or supplier perceptions) and possible other risks like downtime for that employee and others who have to help deal with it. If the behaviour is so bad as to be a legal violation, that sort of behaviour will often be way beyond what the company regards as acceptable. In other words, leaving it up to the authorities is not something the company wants to do since a lot of the time, if it reaches the authorities, it's reached a level definitely not acceptable. When the person isn't working for the company, usually if it doesn't relate to something that may affect their jobs, companies are usually wouldn't care so much about that their employees may do, unless the person is going to be linked to the company. Nil Einne (talk) 06:47, 7 March 2013 (UTC)[reply]
Yes this kind of paternalism is obnoxious, yes it's awful, and I bet you more than just "extreme libertarians" are against it. But trade your liberty for safety and this is the path. Shadowjams (talk) 03:12, 7 March 2013 (UTC)[reply]
In the USA, Progressive Insurance has actually been advertising some kind of tracking gizmo that's supposed to get you lower rates if you drive safely. ←Baseball Bugs What's up, Doc? carrots→ 03:17, 7 March 2013 (UTC)[reply]
At the end of the day, it's not the employee's right. Nor are they forced to accept the car against their will. It's theirs to use on whatever terms and conditions the employer specifies. If they don't like that, they can drive their own car, and can do so any way they like. -- Jack of Oz [Talk] 03:52, 7 March 2013 (UTC)[reply]
Exactly. The fact an employer thought not only this was a good idea, but that most prospective employees would see it as worth it, is what bothers me. Shadowjams (talk) 05:33, 7 March 2013 (UTC)[reply]
Was the employee told up front, or did he find it out through the grapevine? ←Baseball Bugs What's up, Doc? carrots→ 05:37, 7 March 2013 (UTC)[reply]
I presume it was specified somewhere in the contract or the employee was clearly informed of it, otherwise in a country like Australia I think there's a risk of it being seen as violating the employees privacy and in particular, using any data obtained from it in managing the employee suspect. (One of the refs I provided about GPS tracking mentions something similar for that case in Canada). Nil Einne (talk) 06:47, 7 March 2013 (UTC)[reply]
I think you're missing the point bubs. The notion that most employees would accept such an agreement is disturbing. The idea that asking to reveal your basic movements is acceptable should be worrying... similarly with Progressive's tracking service. Shadowjams (talk) 10:16, 7 March 2013 (UTC)[reply]
It's YOU that's missing the point. The employee does not own the car, the company does. If you're worried about being tracked while driving someone else's car, then maybe you shouldn't be driving it. Allowing someone to track your personal car, or not, is your choice, since you own that car. Again, if you don't want to be tracked, don't drive something that has a tracking device. Just like with your office PC - don't got to websites that the company doesn't approve of. If it's your own PC, that's different. ←Baseball Bugs What's up, Doc? carrots→ 12:50, 7 March 2013 (UTC)[reply]
By the same argument, nobody forces you to eat at Ye Olde Burger Bar, so if they serve their burgers with a side of hepatitis, it's caveat emptor. Similarly, you are free to chose a hospital, so we should not force hospitals to use properly sterile equipment and procedures. And you don't have to work at BigCompany, so it's no problem if they install cameras in the toilets and showers, and publish the videos to entertain the board and potential customers. Or maybe we do accept that property rights are subject to potential limits, just like all other rights. --Stephan Schulz (talk) 14:03, 7 March 2013 (UTC)[reply]
If they tell you that their burgers are made from horsemeat, or that their equipment is not sterile then you can make an informed choice. Likewise, if the company tells you that their cars are tracked and that your email and internet are subject to monitoring, then you can make an informed choice. Not that the two groups of ideas have anything to do with each other. In your examples, you're a consumer, i.e. the buying public. Using your company's property is totally different. You're an employee, and you are subject to the company's rules. There can certainly be legal restrictions on those rules, such as disallowing restroom cameras, although I could imagine that if security is a high priority, that might be allowed. And there are laws protecting consumers, hence the brouhaha about horsemeat in European burgers. But the notion of "privacy" when using company-owned equipment like cars and computers is a bogus argument. ←Baseball Bugs What's up, Doc? carrots→ 14:26, 7 March 2013 (UTC)[reply]
Well, so far I quite agree with Shadowjams and Stephan, and as usual, I have confirmed that, on matters of opinion, I am always right ;) But we are still missing the point of the question. I am under the impression that most people side with me that monitoring your braking is a bit much, and they don't like this sort of thing. For me, it could make my driving worse, not better, because the "back seat driver" would always be there, just he would be sitting behind a desk in the back seat, with a title like "Health and Safety inspector". The feeling of being nagged would get a bit much, and affect my decisions, even if only slightly. Since it is apparent that a part of the reason is still very clearly the legal framework, I am still curious as to why governments have done nothing about it, and not put more of the onus on the individual. As mentioned in the question, there are ways of providing references (Nil Einne has gone some way towards this, in the last line of his/her second post). If people can't reference this, I don't mind people giving the debate a good flog, but I'd be interested in the actual topic, of the government's response regarding paternalistic laws. I accept some of this paternalism, but it seems to go much further than most people would like, which in a democracy suggests pressure groups etc. may be controlling things. IBE (talk) 16:04, 7 March 2013 (UTC)[reply]
Speaking of "references", there's your totally unreferenced comment, "It is clear enough that the company is covering itself against lawsuits." Clear? It's not clear at all. What's your basis for that claim? Lawsuits by who? What makes you think it isn't about trying to keep their drivers safe and keep the insurance and repair costs down? And what makes you think it's any of the government's business how a private company manages their own vehicle fleet? ←Baseball Bugs What's up, Doc? carrots→ 17:52, 7 March 2013 (UTC)[reply]
Oh dear, well I don't know quite where I've bothered you so, but I acknowledged Nil Einne's work in providing references, and my last post shows a stepdown from my previous position. As I said, part of it, from those interesting references, is clearly the legal framework. That is wider than just lawsuits, and encompasses OHS legislation. That means companies must meet certain legal obligations, which can exist in the absence of personal injury lawsuits. It is still the legal framework, under jurisdictional/ parliamentary control. So the premise of the question is still true enough for it to remain relevant. Sorry to have bothered you. IBE (talk) 18:17, 7 March 2013 (UTC)[reply]
Gentlemen, this is the war room, you can't fight in the war room. Shadowjams (talk) 18:44, 7 March 2013 (UTC)[reply]
That cracked me up ;) and for those who don't get the reference, you really need to stay in more. A good point to put "resolved" on it, but I don't mean to stop all further contributions. I just mean from here, only add something if it's really big, like a huge doco you saw that I can watch online, on the exact topic, etc etc. I'll still check back, I'm just putting resolved since it looks like too much of a political topic, and I don't mean to inflame anyone. We had a spirited discussion, and I enjoyed reading the posts; I was just trying to bring it back to the exact topic since it was my primary interest. IBE (talk) 19:29, 7 March 2013 (UTC)[reply]
The core question was, "Why doesn't the government more closely regulate how companies manage their own property?" and the answer is, "Why should they?" ←Baseball Bugs What's up, Doc? carrots→ 22:39, 7 March 2013 (UTC)[reply]
Well, I can see I was unclear. My apologies to everyone. But I did eventually state what I was referring to: "The government could, for example, tone down the laws relating to personal injury, or OHS, whenever it is clearly the employee who has endangered himself, without anyone else in the workplace being put at risk." That was from roughly my fourth post, but I did state it. I never said they should make any particular laws directly against companies monitoring driving behaviour. I only suggested they could make laws to change the injury lawsuits that could arise. I was not campaigning for it, just curious to know what's going on. I didn't realise it was so unclear at the beginning, because there was a bit too much else in the thread, so I didn't get that was the primary problem. Sorry for the confusion. IBE (talk) 00:36, 8 March 2013 (UTC)[reply]
I should add that I find it slightly amusing that I put "resolved" at the top, and only after that, finally worked out my own question ;) IBE (talk) 01:10, 8 March 2013 (UTC)[reply]
It went from re-solved to solved. Go figure. ←Baseball Bugs What's up, Doc? carrots→ 03:08, 8 March 2013 (UTC)[reply]

Nietzsche one of the best philosophers?

I always find the Nietzsche's works to be poetic than analytic. I do not know what made Nietzsche surpass Heidegger and Moore in the top 10 philosophers. I am not saying that Nietzsche is not as good as Moore or Heidegger, but is not that Nietzsche criticized some of the key elements in analytic philosophy? He said that logic is just an evolutionary product. Why did he say this? And, why do we consider him as one of the best thinkers despite of this? — Preceding unsigned comment added by 112.205.15.149 (talk) 18:09, 6 March 2013 (UTC)[reply]

Your question seems to be more of an invitation to discussion than a request for references. The best source is Nietzsche himself. I think it's in Beyond Good and Evil that he discusses "the history of an illusion" where he criticizes the systematizers through Kant, and rejects them. (I have to apologize, I haven't read him in 10 years and my books of his are in storage, so others should correct me, please.) Nietzsche is perhaps the most brilliant writer I have ever read, and The Antichrist, regardless of one's opinion (I largely agree) is simply a tour-de-force. Unlike, say, Kant, he's a writer who's far better simply to read than any commentary written on him. μηδείς (talk) 20:28, 6 March 2013 (UTC)[reply]
While Nietzsche is peachy,
I also have high regard for Kierkegaard. StuRat (talk) 02:44, 7 March 2013 (UTC) [reply]

There's nothing Nietzsche couldn't teach ya 'bout the raising of the wrist. ←Baseball Bugs What's up, Doc? carrots→ 03:09, 7 March 2013 (UTC)[reply]

Socrates himself was permanently pissed... --Jayron32 03:56, 7 March 2013 (UTC)[reply]

Italy and Netherlands census Muslim population by cities

I was wondering if there was a website that shows and allows you to download an excel spreadsheet that shows you which cities has the most Muslim populations in numbers and in percentage in Italy and Netherlands? — Preceding unsigned comment added by 70.53.230.50 (talk) 18:45, 6 March 2013 (UTC)[reply]

Biblical hermeneutics...

I am wondering if there is such a thing on secular biblical hermeneutics, where secular people without a faith commitment study the Bible academically in a public university somewhere in the United States, and because some secularists find the work personally moving, inspirational, and thought-provoking - making them sink deeply in contemplation - and attempting to apply what they've gained into a modern-day context, thereby influencing how they behave in society. Is there such a thing for secularists? If a secularist wants to hear opinions on the significance of scripture in modern times, then where would the secularist go, and who shall the secularist consult? Is there such a thing like a "secular pastor" or someone who is an expert on the Bible and how it may be relevant in modern times? 140.254.226.228 (talk) 21:37, 6 March 2013 (UTC)[reply]

Chaplain, perhaps? From my understanding of the article, there are secular chaplains, who, I think, tend to people's spiritual needs. 140.254.226.228 (talk) 21:42, 6 March 2013 (UTC)[reply]
(ec) Many universities have religious studies departments, which sounds like what you're talking about. thx1138 (talk) 21:44, 6 March 2013 (UTC)[reply]
From what I know of religious studies departments, they seem to view religions as a psychological, sociological, and anthropological phenomenon. I think they study religions rather than applying religious texts to modern-day times, which would presumably be done by pastors and priests. 140.254.226.228 (talk) 21:48, 6 March 2013 (UTC)[reply]
[[29]] seems to direct to academic journals on biblical criticism. However, the websites seem to be down. Usually in academic journals, people may write on Discussion to reflect on how the research is relevant or significant for further research or to society, and of course, that do not presuppose any theistic belief or deny such supernatural beliefs. Is biblical studies a complete science like anatomy, or is this an ongoing field of study? If this is an ongoing field of study, I wonder how do these academics study the Bible and forge a new understanding of scripture, which involves theological and practical implications. 140.254.226.228 (talk) 22:21, 6 March 2013 (UTC)[reply]
There are Christian atheists and philologists, who study the bible, although the latter not to apply the knowledge in their private lives. OsmanRF34 (talk) 23:41, 6 March 2013 (UTC)[reply]

Ducks

Does Wikipedia have an article on raising ducks? Wakeenahh (talk) 22:33, 6 March 2013 (UTC)[reply]

No, but you may find information on how to raise ducks here. 140.254.226.228 (talk) 22:37, 6 March 2013 (UTC)[reply]
And the Ref Desk also has a resident bird expert, although his specialty is sea gulls. He is User:Kurt Shaped Box. So, you could leave him a note on his talk page: User talk:Kurt Shaped Box. StuRat (talk) 22:46, 6 March 2013 (UTC)[reply]

Thank you, I found an article on Poultry Farming that covers the subject. Wakeenahh (talk) 23:10, 6 March 2013 (UTC)[reply]

As the OP has demonstrated, the easiest way to raise a duck is to become one. Looie496 (talk) 06:30, 7 March 2013 (UTC)[reply]

A way to fight inflation whenever stimulus money is pumped into the economy?

When there’s a lot of inorganic money being printed by the U.S, known as stimulus, in order to pay down debts and deficits, alleviate or stop a recession, or do whatever Bernanke is doing, can inflation be alleviated or even prevented if the dollar where to be backed up by more gold like for example, by putting more gold into places where the country's gold is stored like in Fort Knox? Or will this not help fight inflation while stimulus happens? If it would be effective in alleviating or stopping inflation, would adding more gold to back up the dollar be too costly to do so anyway if the goal was to pay down a U.S debt? Willminator (talk) 23:19, 6 March 2013 (UTC)[reply]

Inflation is caused more by an increase in the velocity of money rather than the simple amount of money. The U.S. has been effectively printing huge amounts of money, several trillions of dollars since 2008, without any increase in inflation, interest rates, or the employment ratio, mostly because it's all being soaked up by corporate profits which are sitting in banks that don't lend because they get interest on excess reserves, or in tax havens waiting for another repatriation holiday which will never come. My opinion is that the only non-double dip recession way out is improved wealth taxation but that's not popular in the House of Representatives this Congress. 71.215.70.112 (talk) 00:14, 7 March 2013 (UTC)[reply]
Which is of course, a leftist and Keynesian analysis. Check out the Chicago school of economics, the Austrian school of economics, and hard currency theory. μηδείς (talk) 02:01, 7 March 2013 (UTC)[reply]
Yeah, by all means, check those guys out!
  • "demand can be satisfied in far greater quantity, much more quickly, much more reversibly, and without the danger of a fiscal collapse and inflation down the road, if the Fed and Treasury were simply to expand their operations of issuing treasury debt and money in exchange for high-quality private debt" -- Professor John Cochrane of the Chicago School, describing the post-stimulus status quo;
  • "Those of us who have looked to the self-interest of lending institutions to protect shareholders’ equity, myself included, are in a state of shocked disbelief. The whole intellectual edifice...collapsed." And a few years later, "Is the answer to complex modern-day finance that we return to the simpler banking practices of a half century ago? That may not be possible if we wish to maintain today’s levels of productivity and standards of living." Alan Greenspan the leading Austrian economist, flip flopping and also describing the status quo.
Do those leftist Keynesians have it all wrong, with their far more accurate forecasts? Sometimes when two parties disagree, one is right and the other is wrong, because truth exists and is reflected in observations. 71.215.90.134 (talk) 02:42, 7 March 2013 (UTC)[reply]
As far as backing up all our money with gold, I fear the US has far too much currency and far too little gold to do that now. Perhaps after a currency collapse, the meager gold reserves the US has could be used to back a new currency. StuRat (talk) 02:39, 7 March 2013 (UTC)[reply]
What could possibly go wrong? Not that you'll find any of those concerns in Wikipedia's hard currency article. Wikipedia articles on these subjects exist primarily in the Libertarian universe where all taxes are theft and nobody needs to depend on courts to enforce contracts and laws or on governments to protect society from plagues of any kind. 71.215.90.134 (talk) 02:54, 7 March 2013 (UTC)[reply]

Curious point: the US broad money supply (M2) has consistently increased at a faster pace than inflation. Perhaps Milton Friedman got it wrong ?

In the 1960s and the 2000s the average inflation rate (2.3% p.a.) and the velocity of money (60.1 – 61.3) were close enough to exact to make no difference. The money supply (M2), however rose an average of 3.7% a year in the 1960s, and 12.1% p.a. in the 2000s.

Those two decades were the ones with the fastest money velocity in the past 50 years, and the ones with the slowest growth in inflation. DOR (HK) (talk) 08:52, 7 March 2013 (UTC)[reply]

M2 includes a large proportion of time deposits, which you don't want to count in velocity when they are being reinvested because they're still just sitting in the bank, as far from the real economy as possible. M2 also doesn't include money market funds which are how most institutions held cash before 2008. Use MZM velocity instead to predict inflation. 70.59.15.208 (talk) 11:59, 7 March 2013 (UTC)[reply]

BEMBE BEACH , ANNAPOLIS MD, USA

I wanted to know why the Beach in Annapolis-MD was named after Bembe as Bembe Beach, Bembe beach road, was there Bembe people from Africa settled there as slaves or what?...Because Bembe is a tribe or a group of people in Africa - Congo D.R. If possible you can create an article about it... — Preceding unsigned comment added by 99.18.172.70 (talk) 23:32, 6 March 2013 (UTC)[reply]

I've looked through some public domain Gazetteers available on Google Books: [30] and [31] from 1852 and 1904 respectively, and there is no place by that name, nor any similar name, in those Gazetteers. So the name is at least newer than 1904. It is possible the name is from someone's personal name (like a prominent local citizen); many place names take after individual people. --Jayron32 02:23, 7 March 2013 (UTC)[reply]


March 7

All notable philosophers = Mathematical geniuses?

Undoubtedly, all prominent philosophers (Machiavelli, Ludwig Wittgenstein and Saul Karl Marx) have high IQ to the point that they have the right to be narcissistic, although none of them is. I think they can answer almost all questions, but despite their high IQ, is it possible that they have some academic or intellectual weaknesses? For example are all notable philosophers mathematical geniuses? or are there some philosophers who hates it or weak at it? — Preceding unsigned comment added by 112.205.15.149 (talk) 03:30, 7 March 2013 (UTC)[reply]

Where is your evidence that "all prominent philosophers...have high IQ"? --Jayron32 03:37, 7 March 2013 (UTC)[reply]
The only notable philosophers I can think of who might be considered brilliant mathematicians are Descartes, Pascal, Leibniz, and Russell. Looie496 (talk) 06:24, 7 March 2013 (UTC)[reply]
You don't count Gödel as a philosopher? He wasn't trained as one, of course, and academic philosophers tend to think he made a lot of mistakes, but he was absolutely fundamental to contemporary philosophy of mathematics. Gotlob Frege, Hao Wang, Alan Turing, Willard Van Orman Quine, Hilary Putnam, and Richard Montague surely get somewhere into the mix as well. --Trovatore (talk) 07:52, 7 March 2013 (UTC)[reply]

I do not think we need to supply evidences to prove the intelligence of different prominent philosophers. Prominent philosophers are those who are academically and historically recognized. These includes Hobbes, Locke, Nozick, and Rawls. Of course, one reason for their prominence in academic discussions is their intellectual capacity that surpasses most people. Now my question is: , is it possible for them to be weak at some subjects specifically mathematics, despite their philosophical intelligence? — Preceding unsigned comment added by 112.205.15.149 (talk) 07:36, 7 March 2013 (UTC)[reply]


Wittgenstein was more "mathematical crackpot" than "mathematical genius". --Trovatore (talk) 07:46, 7 March 2013 (UTC)[reply]

It's possible for a great philosopher to be very weak at maths if they have no interest it and never study it. Or perhaps someone alerts them to one aspect of maths, like set theory, then they might read up on that, but still be pretty ignorant about other aspects, like trigonometry. Itsmejudith (talk) 12:44, 7 March 2013 (UTC)[reply]
I wouldn't construe Trovatore's comment in the direction that Wittgenstein was weak at maths. Being a crackpot means that you have a lot of fringe theories at best, not that you are not good at mainstream theories. OsmanRF34 (talk) 13:43, 7 March 2013 (UTC)[reply]
Somehow, philosophers like Kierkegaard and Sartre don't strike me as potential mathematical geniuses, but maybe I'm wrong. Some schools of philosophy depend strongly on logic, and have as a result significant kinship with mathematics; others are much more litterary (like existentialism, as the two examples I give above, or even Nietzsche), in which case the links to mathematics are much more tenuous. --Xuxl (talk) 15:06, 7 March 2013 (UTC)[reply]

Readers ignore the word "said"

I once was told to be careful about using words other than "said" unless I really want to draw attention to it, because readers basically ignore the word said but will get tripped up reading words like "mentioned" or "added." First, is this true? And second, where can I find a reputable source to say this? As you can imagine, this is a difficult query to Google. 64.106.114.172 (talk) 03:31, 7 March 2013 (UTC)[reply]

That doesn't seem true to me, although words like "alleged" in place of "said" might draw attention to the fact that you don't necessarily believe it. BTW, wouldn't the Language Desk be a better place for this Q ? StuRat (talk) 03:45, 7 March 2013 (UTC)[reply]
Is this a request for references? Maybe someone can suggest a better forum? The language ref desk is certainly no more relevant than this one. μηδείς (talk) 03:49, 7 March 2013 (UTC)[reply]
It's more about how people read words than the words themselves, so I opted for this one instead. I'm not opposed to its being moved, however. 64.106.114.172 (talk) 03:51, 7 March 2013 (UTC)[reply]
You can find it lots of places, for example rules 3 and 4 of Elmore Leonard's 10 rules. Looie496 (talk) 06:15, 7 March 2013 (UTC)[reply]
Also, our article on Tom Swiftys might be interesting in this context. Looie496 (talk) 06:17, 7 March 2013 (UTC)[reply]
Never let it be said stated, asserted or declared that I would follow such a "rule". Using the same word over and over again seems to me a stylistic faux pas and the sign of a limited writer. Clarityfiend (talk) 07:27, 7 March 2013 (UTC)[reply]
You should try the exercise of pulling out some fiction book whose style you particularly liked, ideally one with lots of dialog, and look at how the author handled this. I think you'll be surprised. Looie496 (talk) 16:34, 7 March 2013 (UTC)[reply]
A bit of a mixed bag: Pride and Prejudice has a fair amount of "cry"ing and "reply"ing, but Phineas Finn is mostly "say"ing. In the best and worst of times, it starts out more varied, then seems to settle down mostly to "said". I will admit it occurs much more frequently than I'd been aware of. Clarityfiend (talk) 03:44, 8 March 2013 (UTC)[reply]
The search term you need is "dialogue tags". Consensus among contemporary novelists is exactly what you said - the overuse of creative tags is distracting to the reader, while "said" is neutral and therefore creates better flow, letting the reader focus on what the characters are saying. The best writers go beyond by using very few verbal tags at all. Some references: [32], [33], [34], [35]. 184.147.116.201 (talk) 12:30, 7 March 2013 (UTC)[reply]

Agency staff

As used here, what are "agency staff"? Did Mrs Brown have to call people whom Americans would call "temporary employees", e.g. Manpower Inc.? Nyttend (talk) 03:42, 7 March 2013 (UTC)[reply]

Yes. StuRat (talk) 03:48, 7 March 2013 (UTC)[reply]

Hugo Chavez net worth at death

I'm hearing figures of 2 billion. Is this true? False? Ultimately unknowable? Somewhere in between? Citation battle! Abeg92contribs 04:18, 7 March 2013 (UTC)[reply]

I can't see any reliable source claiming that. I would be wary of such information, since a lot of malicious misinformation has always been spread about him, mainly from US sources. 82.0.112.151 (talk) 07:48, 7 March 2013 (UTC)[reply]
off-topic
The following discussion has been closed. Please do not modify it.
Oh, you mean from the people that "infected" him with cancer? Speaking of "misinformation". ←Baseball Bugs What's up, Doc? carrots→ 12:44, 7 March 2013 (UTC)[reply]
Misinformation can't be bi-directional? 131.251.133.27 (talk) 13:31, 7 March 2013 (UTC)[reply]
No question about it, it can be. The point being, last I knew they hadn't even owned up to which type of cancer he had, so how likely are they to provide useful and valid information about his net worth? For one thing, theoretically an extreme socialist should have almost no "net worth", unless he's (gasp!) a hypocrite. ←Baseball Bugs What's up, Doc? carrots→ 13:44, 7 March 2013 (UTC)[reply]
In other words, Bugs, you have no factual information on this subject, just opinions. --Mr.98 (talk) 14:08, 7 March 2013 (UTC)[reply]
Well, our own Hugo Chávez article reiterates that his type of cancer has not been revealed. Feel free to make corrections to that article if your own factual information is better. ←Baseball Bugs What's up, Doc? carrots→ 14:17, 7 March 2013 (UTC)[reply]
Who is this mysterious "they" and why do you have to "own up" to types of cancer? is cancer a crime now? And who said he was an "extreme socialist"? It's precisely the sad and sorry attempt to create the "hypocrite" claim that's the issue here. It reminds me of all those Hollywood movies on which apparently ideologically motivated revolutionaries or 'terrorists' all turn out to be thieves motivated by money. Because (gasp!) no one can really be motivated by ideals can they?! Everyone's really a capitalist. It's reassuring to believe that. Paul B (talk) 14:34, 7 March 2013 (UTC)[reply]
If they are going to accuse the US government of somehow "infecting" him with cancer, then they have to be more open about it. That's where it stops. As for being motivated by "ideals", one of his "ideals" was buddying up to a nation that wants to destroy Israel. So much for "ideals". ←Baseball Bugs What's up, Doc? carrots→ 14:44, 7 March 2013 (UTC)[reply]
As I say, who is this mysterious "they"? As for Israel, you seem to be saying that no-one can have ideals that happen to differ from your own. Charming. Paul B (talk) 14:47, 7 March 2013 (UTC)[reply]
You're right, genocide is charming, no question about it. ←Baseball Bugs What's up, Doc? carrots→ 14:51, 7 March 2013 (UTC)[reply]
And complacent ranting is not informative. This is the point. We have to acknowledge that even views we disagree with or even deplore can result from ideals: ideals we choose to reject, for sure. It's more comfortable to believe otherwise. Paul B (talk) 14:54, 7 March 2013 (UTC)[reply]
Calling genocide an "ideal that happens to differ with ours" is not informative either. ←Baseball Bugs What's up, Doc? carrots→ 14:58, 7 March 2013 (UTC)[reply]
As per 82.0, I see no reliable sources on the matter (the only referenced source is apparently a think tank or lobbying group with virtually no web presence or secondary search results), and lean toward "ultimately unknowable". Chavez can probably be fairly described as being somewhere on the autocrat spectrum, and so how do you divide up the net worth of various nationalized industries? On the other hand, his Chavismo politics should not be confused with an extremist "no personal possessions" style of socialism. For instance, there are good sources for other billionaires in Venezuela, at least two of whom are valued at $4+ billion USD and acknowledged political opponents of Chavez. So we can fairly rule out "hypocrite" as a useful categorization, at least in this regard. — Lomn 14:29, 7 March 2013 (UTC)[reply]
I like the implication of Bugs' post. Chavez' net worth is in inverse relationship to the extremeness of his views. Itsmejudith (talk) 14:40, 7 March 2013 (UTC)[reply]

In the article it says He was for a time recognised as Prince of Wales. Even after re-reading the article I don't understanding what "time" they are referring to. Can I get some dates they are speaking of as him being "recognised" as Prince of Wales? Thanks!--Doug Coldwell (talk) 12:41, 7 March 2013 (UTC)[reply]

Wikipedia's article King of the Britons gives an uncited note that he was so recognized as Prince of Wales by "treaty with England". --Jayron32 13:03, 7 March 2013 (UTC)[reply]
It looks like then from around 1220.--Doug Coldwell (talk) 13:44, 7 March 2013 (UTC)[reply]

what benefits does the legal precedent of killing american citizens via drone attacks?

Invitation for a forum-like discussion

The previous version of this question was removed as hypothetical. So, in light of Obama's

http://www.washingtonpost.com/blogs/post-partisan/wp/2013/02/05/justice-department-chilling-drone-white-paper/

"On Monday night, NBC News’s Michael Isikoff published a Justice Department memo justifying the “targeted killings” — without due process — of U.S. citizens who are leaders in al-Qaeda or “associated forces” but are “outside the area of hostile activities,” such as Afghanistan."


So, what specific benefits would this precedent have?

(I realize it may have many downsides, however this question is about benefits).

For example, I could think of the following benefits to the united states government randomly killing random civilian americans on american soil without due process or anything. Besides all the negatives (which isn't what this question is about), some positives might be:

- People might work harder, trying to "fit in" to society a bit better so that it would be politically inopportune for the government to just kill them

- People might actually live life better knowing it might be their last due to a random government killing without any due process or anything

- Perhaps some people would try to avoid this, so security and so forth could be increased for everyone, this would be a benefit to everyone as a house rigged to protect against random killing by the government might also defer petty burglars, who might instead join society as part of the normal workforce

- There could be a greater pressure to specify the legal framework and fundamentals of legal theory that makes this "wrong", as perhaps innocent people would probably be killed. so a benefit is that this might bring greater constitutional scrutiny or an amendment to end this.

would there be any other benefits? I've just listed a few off of the top of my head. I certainly appreciate that the issue raised has potentially many more negatives than positives. However, I'm just asking about any positives. 91.120.48.242 (talk) 12:48, 7 March 2013 (UTC)[reply]

This is not a forum to discuss hypothetical issues. This reference desk is designed to get references to help you answer factual questions. --
Fair enough. I made it more specific for references about something that is no longer hypothetical. However the references I receive will therefore be somewhat narrower. Thanks for your help or answers. 91.120.48.242 (talk) 14:33, 7 March 2013 (UTC)[reply]
You're still looking for a debate. U.S. citizens are not being targeted as such. People making war on America are what's being targeted, and they can't hide behind their citizenship to get away with it. The President's job is to protect us from enemies. If those enemies happen to be Americans, he still has his job to do. ←Baseball Bugs What's up, Doc? carrots→ 14:40, 7 March 2013 (UTC)[reply]
In addition, note that US citizens at home are not targeted. Go ahead, booby trap your house, but unless the CIA is absolutely positive that you're working with Al Qaeda and you are training terrorists in some middle eastern country, you have nothing to worry about. Ryan Vesey 14:53, 7 March 2013 (UTC)[reply]
Many localities are already imposing restrictions on the use of drones in America for a variety of reasons, mostly about privacy rights. That patchwork will probably yield to a blanket set of rules at some point. ←Baseball Bugs What's up, Doc? carrots→ 14:56, 7 March 2013 (UTC)[reply]
Why would you live better if you knew you might be randomly killed? It might just as easily make you lose any faith in the point of moral behaviour if the state itself engages in random acts of murder. Paul B (talk) 14:42, 7 March 2013 (UTC)[reply]

Is biblical studies a complete science?

When I use the term "science", I do not intend to refer to the academic subject science. I mean "science" as a "body of knowledge", like Anatomy, which is regarded as a "complete science". I wonder how fast this field of study re-update itself. 140.254.226.183 (talk) 15:31, 7 March 2013 (UTC)[reply]

Not sure what a "complete" science is, as science is never "complete" in the sense of "done". I'd be inclined to call any study of religion a Social science. ←Baseball Bugs What's up, Doc? carrots→ 15:36, 7 March 2013 (UTC)[reply]
Then I suggest that you read History of anatomy and skip to the conclusion in which it states "Anatomy is often regarded as being a complete science, in that we know what and where most of the body is and does with little left to discover." From my experience, I have heard of that claim many times, in textbooks too. There is relatively little to discover in Anatomy, because I think the human body is a closed system. I don't think it implies or states that it is closed to more inquiries; it's just that we may already know all the basic parts/components. 140.254.226.183 (talk) 15:57, 7 March 2013 (UTC)[reply]
The Nobel Prize in Physiology or Medicine is a rather large carrot for finding new things about the human body. Does anyone envisage this prize ever being discontinued because "there's nothing worth discovering any more"? -- Jack of Oz [Talk] 19:20, 7 March 2013 (UTC)[reply]
Most research about the human body these days is more along the lines of biochemistry than anatomy. The layout of the various pieces of the body has been fairly well-established for a while. --Carnildo (talk) 02:22, 8 March 2013 (UTC)[reply]
'Science' has never been used to describe subjects, it's used to describe the process - the scientific process. So physics is a science if it uses scientific methods, but me trying to cook the most delicious meal for my customers is also a science, if I use the scientific method. So I make a hyothesis that adding chocolate makes everything taste better, then I go about testing it, and come up with a better hypothesis based on my results. Science is not attributed to subjects, but is a process. Kinkreet~♥moshi moshi♥~ 15:53, 7 March 2013 (UTC)[reply]
That isn't true. We consistently refer to "the sciences" and the scientific method is not used in the social sciences. Ryan Vesey 15:54, 7 March 2013 (UTC)[reply]
Adding to that, it's impossible to use the scientific method to History. Historical events are not replicable in the present, and present observations may not be the direct effect of the supposed causes. 140.254.226.183 (talk) 16:03, 7 March 2013 (UTC)[reply]
It's not as if you would apply the scientific method to an object. You apply it to a theory or a hypothesis. So, historical theories or hypothesis, can be contrasted somehow with empirical evidence. OsmanRF34 (talk) 16:17, 7 March 2013 (UTC)[reply]
Yeah, but evidence in the Humanities uses a great deal of interpretation and reach a logical conclusion by deduction. I don't think it's the same sort of evidence in the Science or maybe how it is handled that is different. 140.254.226.183 (talk) 16:24, 7 March 2013 (UTC)[reply]
Per the definition put forward by the anatomy reference above, no, Biblical studies is not "complete". Notably, ongoing archaeological efforts continue to inform the field, both by direct historical application and by discoveries of new manuscripts. — Lomn 16:06, 7 March 2013 (UTC)[reply]

That comment about anatomy sounds like nonsense. New discoveries are trumpeted every so often, especially at the micro level. --Dweller (talk) 16:10, 7 March 2013 (UTC)[reply]

The catch word is "especially at the micro level". 140.254.226.183 (talk) 16:18, 7 March 2013 (UTC)[reply]
Micro level is still anatomy. --Dweller (talk) 16:47, 7 March 2013 (UTC)[reply]
Yes, it's still anatomy. Though, I don't think it really contradicts the quote. I see the quote as an aphorism about anatomy. 140.254.226.183 (talk) 17:15, 7 March 2013 (UTC)[reply]
That's OK, I'm happy to disagree with you. --Dweller (talk) 21:42, 7 March 2013 (UTC)[reply]

Anybody care to give links? The main point of this is to actually find something interesting to read in this field. I am still trying to understand Romans. I found an article in the Journal of Biblical Studies, but that was written in the 1940s. I also want something relevant and recent too. Not something that happened 50 years ago or what scholars thought 100 years ago. 140.254.226.183 (talk) 16:15, 7 March 2013 (UTC)[reply]

What exactly is it you are looking for? Is it a commmentary? If so, I've heard that John F. MacArthur writes very good ones. Ryan Vesey 16:27, 7 March 2013 (UTC)[reply]

The missing magic word in this discussion is hermeneutics. The question needs to be directed to Biblical hermeneutics. alteripse (talk) 16:31, 7 March 2013 (UTC)[reply]

I clicked on Biblical hermeneutics, which led me to Historical-grammatical_method. Heh-heh. I guess I was looking in the wrong area. No wonder I couldn't find what I was looking for. Duuuuhhhhhh... This site, which I discovered while browsing through Wikipedia, talks about archaeology. If I want to look for significance, then I probably have read up on theology. 140.254.226.183 (talk) 17:08, 7 March 2013 (UTC)[reply]
Textual criticism (and higher criticism) of the Bible is an ongoing field, perhaps just as strong now as it was in the 19th century. The Editio Critica Maior of the New Testament is an ongoing project. For competing theoretical work, see the work of Robinson and Pierpoint, who favour the Majority Text. For the Old Testament, there are the Oxford Hebrew Bible, Biblia Hebraica Quinta, and the Hebrew University Bible projects. For the Septuagint, there is the Göttingen Septuagint. There are many academic journals that deal with Biblical studies in general and textual criticism in particular. For active journals, see Journal of Theological Studies, published by Oxford University; Journal for the Study of the New Testament and Journal for the Study of the Old Testament published by Sage; Vetus Testamentum, Novum Testamentum and Biblical Interpretation by Brill; and New Testament Studies by Cambridge University. All of these are right in the academic mainstream.--Atethnekos (DiscussionContributions) 20:59, 7 March 2013 (UTC)[reply]
You might be interesting in reading the recent by James Dunn, The Theology of Paul the Apostle (Eerdmans, 2006) [36]. This is under the so-called New Perspective on Paul. --Atethnekos (DiscussionContributions) 21:34, 7 March 2013 (UTC)[reply]
Stackexchange has an active Biblical Hermeneutics question-and-answer site. --ColinFine (talk) 10:32, 8 March 2013 (UTC)[reply]

Does the PRC invoke the Succession of states theory of international law to justify its rule of Tibet?

I’m looking for references to actual PRC government documents or policy statements that specifically mention succession of states. The question came up at the Tibetan sovereignty debate article last summer, and the upshot was that no one found such a reference (which surprised me). Here are sources that I checked where I can’t find mention of the PRC claiming succession of states:

Here are a few that say the PRC claims succession of states, but provide no reference to the actual claim:

--Wikimedes (talk) 04:16, 8 March 2013 (UTC)[reply]

Marijuana tax stamp

So evidently one of my friends has a sibling wanted for "possessing marijuana without a tax stamp". Since possession of marijuana is illegal in the first place, what is the purpose of a tax stamp being required? Wouldn't anyone trying to buy such a tax stamp be arrested for possession of marijuana anyway? (Jurisdiction is Kansas.) Ks0stm (TCGE) 16:58, 7 March 2013 (UTC)[reply]

First result from Googling the phrase and Kansas turns up this concise explanation. It's either a clever revenue scheme or just another way to throw the book at someone doing illegal activities. My guess is the latter. --Mr.98 (talk) 17:05, 7 March 2013 (UTC)[reply]
See Marihuana Tax Act of 1937. Gandalf61 (talk) 17:06, 7 March 2013 (UTC)[reply]
According to that article, that law has been repealed for 40+ years. So it's not that. --Mr.98 (talk) 17:25, 7 March 2013 (UTC)[reply]
(EC with all above) Well a simple search finds [37], [38], [39]. If you add Kansas you get [40], [41], [42], [43], [44]. It's possible some of these laws predate marijuana basically being illegal in every state as well as federally but from the Kansas results a number of which discuss the situatuion in other states, it sounds like many of them are new. In terms of the self incrimination bit, well that was what killed Marihuana Tax Act of 1937. However as per the Kansas results, while that has also sometimes been a problem with the new state laws, in some cases including Kansas it has been addressed or at least attempts were made to address it. For example by forbidding sharing of information and allowing it to be done anonymously. Of course it's not abnormal for tax authorities to expect tax even from illegal activities although having a tax which only targets illegal activities is I think fairly rare. Nil Einne (talk)
I wonder what percentage of marijuana dealers pay this tax. Seems to be quite an admission that the War on Drugs is a failure. Another is drug-free school zones. If the war on drugs had worked, neither would be needed. StuRat (talk) 17:26, 7 March 2013 (UTC)[reply]
Just on the general issue of taxing illegal activities in the U.S. The IRS absolutely expects you to pay proper taxes on illegal activities. See Al Capone for a famous example. He was specifically tried and convicted for not paying taxes on his illegal alcohol smuggling business during prohibition. It turns out it was easier to get evidence of that crime than of the actual illegal activity that produced the money in the first place. IIRC, there were coded books and complex money laundering schemes that made it difficult to proved where the money came from, but the fact that the money could be proven to exist in the first place was all that was needed to prove tax evasion. --Jayron32 17:32, 7 March 2013 (UTC)[reply]
Per the links I provided, very few. Nil Einne (talk) 17:36, 7 March 2013 (UTC)[reply]
Too bad Capone didn't plan better. He could have sold, along with each shipment of booze, a painting with an X on it, and claimed that the payment of thousands of dollars was for the "art", paid taxes on that, and stayed out of prison. StuRat (talk) 17:38, 7 March 2013 (UTC) [reply]
Money laundering's no joke. Check out 18 USC 1956 and 18 USC 1957. Shadowjams (talk) 18:52, 7 March 2013 (UTC)[reply]

Bach , Persian composers

In the book Godel, Escher ,Bach the music of Bach is compared with Godel"s theoreom. I want to know whether classical persian and Indian music composers like Amir Khusrou or Tansen can be treated in same way.Solomon7968 (talk) 17:30, 7 March 2013 (UTC)[reply]

As far as I am aware, any music can be. Afrer all, chose the so-so artist Escher as his visual example. Paul B (talk) 18:20, 7 March 2013 (UTC)[reply]
Escher was chosen for his "conceptualism" and congruence with Hofstadter's themes in the book, not for his pure art-for-art's sake reputation... AnonMoos (talk) 19:38, 7 March 2013 (UTC)[reply]
He was chosen because Hofstadter had no taste. But seriously, Bach is not the only composer who wrote fugues, canons etc. I know nothing about Persian music, but playing with the limits of formal structures is common to all musical traditions. In other words it is not a question of the uniqueness of Bach any more than of the execrable Escher. Paul B (talk) 20:51, 7 March 2013 (UTC)[reply]
He's getting worse by the post. First "so-so", now "execrable". What's your problem with Escher? -- Jack of Oz [Talk] 21:02, 7 March 2013 (UTC)[reply]
Whatever -- Hofstadter found that Drawing Hands deeply resonated with themes in his book, and that other Escher works were both entertaining and thought-provoking, and could be slotted into the book in various ways. He really didn't care about Escher's reputation among professional art critics, nor do most readers of the book (though he did use some Magrittes as well as Escher works). AnonMoos (talk) 03:57, 8 March 2013 (UTC)[reply]

Not much of a musical theorist, but Bach seems to be noted for composing in a particularly mathematical style. I found Can Mathematical Patterns Be Found in Johann Sebastian Bach's Two-Movement Preludes and Fugues?, A Fractal in Bach's Cello Suite, Music, Mathematics and Bach and pages and pages of similar results. No idea if Persian music falls into the same category. Alansplodge (talk) 22:50, 7 March 2013 (UTC)[reply]

It's not so much that he composed "in a particularly mathematical style", but that all musical scores are amenable to mathematical analysis or treatment (which is not the same as saying that the essence of the music itself is so easily captured). The works of some composers are more readily viewed through this prism than those of others, that's all. For my money, there's more mathematics in John Cage's 4'33", which is specified as an exact period of time during which no music is played at all, than anything Bach ever wrote. -- Jack of Oz [Talk] 23:31, 7 March 2013 (UTC)[reply]
You're aware, are you, of Bach's canon Quarendo invenietis, which is presented as a mathematical puzzle which must be solved before the music can be played? There are four solutions; I own a recording which covers all four. That's just one example off the top of my head; there are others. I think you must either know very little about Bach, or very little about maths. (Not to diss Cage, mind you; his work is ingenious in its own right.) AlexTiefling (talk) 00:53, 8 March 2013 (UTC)[reply]
You're right, of course. I know very little about any subject you could possibly name. Some less than others. Particularly the ones I've studied in depth for decades; I know about them least of all. -- Jack of Oz [Talk] 01:28, 8 March 2013 (UTC)[reply]
I do not know about Persian, but since you also mentioned Indian I googled "mathematics ragas" and came up with Mathematics in carnatic music and A Statistical Analysis of Raga Ahir Bhairav so that at least is evidence that music other than Bach's has been looked at in a similar way. 184.147.116.201 (talk) 01:23, 8 March 2013 (UTC)[reply]

did any large-ish startups get started via kickstarter (instead of a more traditional seed round in exchange for equity)?

did any large-ish startups get started via kickstarter (instead of a more traditional seed round in exchange for equity)? I'm thinking dropbox and the like. 178.48.114.143 (talk) 17:38, 7 March 2013 (UTC)[reply]

Our article on Kickstarter includes a list of the most-funded projects, which tops out at $10 million for the Pebble watch. I don't know what you consider the threshold for "large-ish", but I will note that the big Kickstarter projects are all money-for-product instead of money-for-equity. Thus, I expect the answer to your question is "no", or at least "not yet". — Lomn 18:26, 7 March 2013 (UTC)[reply]

How much dependant is my country on the U.S.?

Federated States of Micronesia. How much? In what apart from defense? FMicronesian (talk) 18:32, 7 March 2013 (UTC)[reply]

According to our article Federated States of Micronesia–United States relations, which isn't excellent, the US provides $100 million in funding to the Federated States of Micronesia annually. Ryan Vesey 18:37, 7 March 2013 (UTC)[reply]
That represents 80% of the government's revenue and about 29% of GDP. Put another way, without that funding, the country's economy would shrink by more than a third, perhaps even more due to multiplier effects. Marco polo (talk) 20:53, 7 March 2013 (UTC)[reply]
The US is also your largest trading partner. And, even though you said "apart from defense", don't forget all the money that brings in, from US soldiers spending money there to your citizens serving in, and being paid by, the US military. StuRat (talk) 03:48, 8 March 2013 (UTC)[reply]
From the CIA Factbook:

Economic activity consists primarily of subsistence farming and fishing. The islands have few mineral deposits worth exploiting, except for high-grade phosphate. The potential for a tourist industry exists, but the remote location, a lack of adequate facilities, and limited air connections hinder development. Under the original terms of the Compact of Free Association, the US provided $1.3 billion in grant aid during the period 1986-2001; the level of aid has been subsequently reduced. The Amended Compact of Free Association with the US guarantees the Federated States of Micronesia (FSM) millions of dollars in annual aid through 2023, and establishes a Trust Fund into which the US and the FSM make annual contributions in order to provide annual payouts to the FSM in perpetuity after 2023. The country's medium-term economic outlook appears fragile due not only to the reduction in US assistance but also to the current slow growth of the private sector.

I'd say you need to develop tourism to compensate for reduced US assistance. In particular, you might market vacations to newly rich Chinese, who want a tropical getaway. StuRat (talk) 03:53, 8 March 2013 (UTC)[reply]

People who lived before Jesus

What Christianity (e.g. Roman Catholicism) says about people who lived before Jesus' Incarnation in the sense of their ultimate fate? I mean does Jesus's grace and love extend to them also, despite their polytheistic views?--93.174.25.12 (talk) 19:07, 7 March 2013 (UTC)[reply]

Polytheism didn't suddenly end the day Jesus was born, as any Hindu will confirm. -- Jack of Oz [Talk] 19:11, 7 March 2013 (UTC)[reply]
(edit conflict) I once had a Hindu classmate who was monotheistic. He called his god "God", because he only believed in one god. He put a red dot on his forehead every morning as a way to remind him of God. Sneazy (talk) 19:42, 7 March 2013 (UTC)[reply]
See harrowing of hell. I'm afraid polytheists stay there, unless they convert on seeing Jesus. Paul B (talk) 19:30, 7 March 2013 (UTC)[reply]
(ec)See Bosom of Abraham, Limbo of the Fathers and Harrowing of Hell for mainstream (but by no means universal) Christian opinions on the issue. 1 Peter 3:19-20 and 1 Peter 4:6 are the main scriptural starting points. Tevildo (talk) 19:35, 7 March 2013 (UTC)[reply]
To answer the OP's question, there were many polytheists at the time of Jesus, in the ancient world. Judaism is known to be the world's first monotheistic religion, and Jesus just happens to be born into it. I am not aware of Jesus's love and grace other than the examples in the gospel accounts, and I am not aware of whether or not Jesus had personal contact with polytheistic pagans other than the Romans who supposedly executed him. I think the OP is conflating Jesus with God. It would make more sense to say that God's grace and love extend to other people, not Jesus's love and grace. No person lives in a vacuum. Jesus was influenced by his society and affected his society. Therefore, I suppose in order to accept "Jesus's love and grace", one must first buy into his worldview or understand Judaism. I can't imagine how a Roman pagan would really care about their salvation. Sneazy (talk) 19:42, 7 March 2013 (UTC)[reply]
Never mind. I think this discussion is about Christian theology and Christian cosmology. Excuse my confusion. I thought it was about actual history. Sneazy (talk) 19:52, 7 March 2013 (UTC)[reply]
When I was growing up in a strict Christian sect, I asked this question and was told that such people would be judged as to whether they had lived according to the light they had received. I have no idea whether this is a mainstream Christian idea. --TammyMoet (talk) 20:06, 7 March 2013 (UTC)[reply]
Not in exactly those terms, but I was told something quite similar growing up. The idea was that the conscience was a kind of God-given moral law that non-Christians (specifically, non-Christians who were not aware of Christianity) were bound to obey as best as possible in order to gain salvation. I have no idea how close this is to mainstream Christian teachings. Despite the teaching of Extra Ecclesiam nulla salus, I know there is a broad and fascinating range of opinions in Catholic theology on topics like this. Evanh2008 (talk|contribs) 20:14, 7 March 2013 (UTC)[reply]
Fate of the unlearned has the answer. OsmanRF34 (talk) 20:20, 7 March 2013 (UTC)[reply]
Thanks. 93.174.25.12 (talk) 20:48, 7 March 2013 (UTC)[reply]
See http://wol.jw.org/en/wol/d/r1/lp-e/1102005137#p23, paragraph 19.
Wavelength (talk) 20:59, 7 March 2013 (UTC)[reply]
For one view, that is. That of the Jehovah's Witnesses. Thank you for providing yet another perspective. It's important to recognize that (like anything one could ask along these lines) there is probably no universal Christian belief in these regards, but it is helpful to have all of the myriad different viewpoints so the questioner can have a full picture of the bredth of perspectives. Other interesting perspectives on the matter can be found at Virtuous pagan, which covers the medieval and early modern Christian perspective on the question, as answered by thinkers such as Aquinas and Dante. --Jayron32 21:13, 7 March 2013 (UTC)[reply]
Perhaps we should have a standard disclaimer template to use every time Plasmic Physics gives a specifically JW view and claims it's general to all Christians, or the only correct one. AlexTiefling (talk) 00:45, 8 March 2013 (UTC)[reply]
For the record: I'm not JW, nor do I claim that my views are general to all Christians, nor do I claim that my views are perfectly correct. Plasmic Physics (talk) 01:31, 8 March 2013 (UTC)[reply]

plays at theatres

I wanted to take a friend of mine to the theatre to watch either Cats or Pirates of Penzance, two plays he's wanted to see for a while, but I have no idea where would be showing them. Is there any website that lists what's on at different theatres or something, I tried google but with no luck. I just want anywhere near me and within the next couple of months, but no idea where to look for the information.

86.15.83.223 (talk) 21:55, 7 March 2013 (UTC)[reply]

In the UK? (IP geolocates to England) Try a search at this site. Ghmyrtle (talk) 23:08, 7 March 2013 (UTC)[reply]
Hmm, that doesn't mention a couple of places I had already found showing them over the summer, does that mean their versions are unofficial or something? 86.15.83.223 (talk) 23:36, 7 March 2013 (UTC)[reply]
When a play or musical play is first written, it is protected by the copyright of its authors. Its authors can decide when and where it can be presented. Cats The Musical was first produced in the West End in 1981, and so is still in this category. A production of Cats is either authorized by its authors (or more usually by those they have designated to do so), or it is unauthorized and can be shut down. Productions are expected to pay royalties to the authors. Once a certain number of years has passed (the exact number of years has been changed over time, and also varies by location), the copyrights lapse and the work falls into the public domain. The Pirates of Penzance is now in the public domain. Anyone may produce it in the theatre, and no one pays royalties to its authors. There can be nothing official or unofficial about a production of Pirates of Penzance.
There is no central registry of productions of plays in the public domain. And even the authors of works such as Cats may not be able to tell you at any moment where all the current authorized productions of their work are. But they should be able to tell you where the class 1, major presentations are. For Cats, [45] might help. - Nunh-huh 01:11, 8 March 2013 (UTC)[reply]
An exception might be a school production, where the copyright holders may not see it as real competition, and any attempt to shut it down might be bad publicity, so they let it run. StuRat (talk) 03:42, 8 March 2013 (UTC)[reply]
Even schools pay for performance rights. They are a substantial expense for high school and local theater productions. Just like music, rights are managed by a small number of specialist rights managements groups who have less interest in negative consequences than individual authors might. Rmhermen (talk) 05:57, 8 March 2013 (UTC)[reply]
This is true, but in fact both Cats and The Phantom of the Opera have been made available for school productions. There are schools-only versions of some other shows that are not currently available for amateur productions, notably Les Miserables and (I think) Miss Saigon. There's even one for Avenue Q, with some of the more "adult" content considerably toned down. AndrewWTaylor (talk) 10:43, 8 March 2013 (UTC)[reply]
But the answer to your original question is no. There are many sites that are local to an area (eg http://www.yorkshire.com/what-to-do/artistic/performing-art), to an organisation (eg http://www.noda.org.uk/events), or to a genre (eg for Gilbert & Sullivan, http://math.boisestate.edu/gas/) but it is unlikely that any of them is complete even within its own domain. It takes substantial work to maintain lists such as these, and it also relies on the producers of each show passing the information on to them. And the financial returns from such a site are pretty well limited to advertising, so they are mostly done as public services, which again does not suggest that they will be well-resources. --ColinFine (talk) 10:47, 8 March 2013 (UTC)[reply]

El Gamil (Egypt) fortress?

I have started work on our stub article El Gamil, the home of Port Said's airport and scene of a militarily successful but politically disastrous British airborne assault during the 1956 Suez Crisis. The article's lead states that it is "a fortress" and it appears in Category:Castles in Egypt. However, the only internet references that I can find to a fort or castle at El Gamil seem to have been inspired by Wikipedia. I have found quite detailed accounts of the 1956 combat, when after the capture of the airfield, the fighting continued in a sewage works, a cemetery, a coast guard barracks and a shanty town, but curiously not a fort. Can anyone confirm or deny the existence of a fortification here please? Alansplodge (talk) 21:55, 7 March 2013 (UTC)[reply]

To start: I've found two references from the late 19th century. 1895, mention of "Fort Gameel" and "port+said" 1882-85, "if preparations at Fort Gemil threaten Port Said... it will be destroyed". 184.147.116.201 (talk) 01:36, 8 March 2013 (UTC)[reply]
Sorry, don't know how to fix that second link. It should point to http://books.google.com/books?id=slUoAAAAYAAJ page 252. 184.147.116.201 (talk) 01:38, 8 March 2013 (UTC)[reply]
That spelling, "Fort Gemil", brings more results. Here's another one from the 1880s. "Within eyesight of the town, on the seashore, in the direction of Damietta, stood Fort Gemil, garrisoned by troops variously estimated at numbers ranging from 600 to 2,500..." http://books.google.com/books?id=BqkaZlE7YWsC&pg=PA169&lpg=PA169&dq=%22fort+gemil%22&source=bl&ots=x4GgUzHwP5&sig=KnZTfEFOTU2f-0UTbqHJrC4qOnw&hl=en&sa=X&ei=7kE5UceGC5DE0AGZj4GQDg&ved=0CDYQ6AEwAg#v=onepage&q=%22fort%20gemil%22&f=false 184.147.116.201 (talk) 01:48, 8 March 2013 (UTC)[reply]
Thanks 184, I tried "Gamil fort" but not "fort Gamil". At least I have something to work on now. Alansplodge (talk) 12:36, 8 March 2013 (UTC)[reply]

Crucifixion

how many allied nations represented at Jesus' crucifiction — Preceding unsigned comment added by Lady65 (talkcontribs) 22:33, 7 March 2013 (UTC)[reply]

I'd be surprised if anyone can answer this, but what do you mean by allied nations? Do you mean people from land that was later owned by nations that became the Allies in the World Wars? IBE (talk) 22:40, 7 March 2013 (UTC)[reply]
Do you mean the status quo of the Church of the Holy Sepulchre? --Atethnekos (DiscussionContributions) 23:13, 7 March 2013 (UTC)[reply]
Don't waste your time
The following discussion has been closed. Please do not modify it.
3. OsmanRF34 (talk) 23:20, 7 March 2013 (UTC)[reply]
Any advance on 3? IBE (talk) 00:17, 8 March 2013 (UTC)[reply]
That would be the USA, the USSR, and the UK. Or their predecessors. ←Baseball Bugs What's up, Doc? carrots→ 00:51, 8 March 2013 (UTC)[reply]
  • Is this maybe a bad machine translation? Perhaps Lady65's native language is not English, and something is getting lost here. Crucifixion of Jesus#People present at the crucifixion gives various accounts of the other people present at the Crucifixion of Jesus. They include the two men he was crucified with (known as the Penitent thief and the Impenitent thief) and some of his followers, including the "Disciple whom Jesus loved" (i.e. John the Evangelist), and several women, namely The Three Marys (Jesus' mother, James' mother, and Salome (either Jesus' aunt and/or mother of John the Evangelist) and possibly the Myrrhbearers (Mary Magdalene, Johanna, and Susanna). As far as I know, all of these named individuals were of the Jewish nation; none came from outside Judea. There were some people from outside Judea who would have been present as well, including Simon of Cyrene, the centurion and soldiers who guarded the scene, mocked him on the cross, hung the INRI sign gambled for his clothes, and pierced his side. The centurion and soldiers would have been Romans, but probably not from Italia; I'm pretty sure the eastern legions recruited from the Eastern reaches of the empire, so the centurion and soldiers were likely Greek-speaking. That's just about everyone reported to have been present for some part of the crucifixion event. --Jayron32 02:03, 8 March 2013 (UTC)[reply]
Agreed about machine translation - perhaps "allies" means disciples. Alansplodge (talk) 09:13, 8 March 2013 (UTC)[reply]

There's a list of the nationalities of witnesses of the Pentecost in Acts 2:9-11, but not for the crucifixion... AnonMoos (talk) 03:48, 8 March 2013 (UTC)[reply]

March 8

Tower of London medieval escape

In the article Dafydd ap Llywelyn it says Gruffydd died trying to escape from the Tower of London by climbing down a knotted sheet, and fell to his death in March 1244. Was this a common type of escape in this time period from the Tower of London? Is there other details in a reference on how Dafydd fell to his death? Did the sheets untie? Did he lose his grip? Did others knock him off the sheets? Was he speared? Or some other reason?--Doug Coldwell (talk) 00:54, 8 March 2013 (UTC)[reply]

It was his brother Gruffydd ap Llywelyn Fawr actually. There is some more info there, including an amusing manuscript illustration. Adam Bishop (talk) 01:25, 8 March 2013 (UTC)[reply]
Take all colourful medieval,stories with a ton of salt. I found out here http://books.google.co.uk/books?id=sXBdNsDxJ_cC&pg=PA217&dq=gruffydd+tower+london+death&hl=en&sa=X&ei=OZs5UanqLYLfOpCmgegL&ved=0CDEQ6AEwAA#v=onepage&q=gruffydd%20tower%20london%20death&f=false that the story comes from the Chronica Majora of Matthew Paris. Our article on Paris says that he is "not always reliable". Having said that, the incident would have occurred in Paris's lifetime and it isn't a impossibility. Itsmejudith (talk) 08:11, 8 March 2013 (UTC)[reply]

Female genital mutilation prevalence rates over time in a graph?

Hi. I need a graph of the overall world-wide combined incidence rate of all types of female genital mutilation. Preferably over the past decades, but I'm not sure how feasible that is because I found some tables which might have part of that information in the back of http://www.unicef.org/publications/files/FGM-C_final_10_October.pdf but they are very hard to interpret (e.g. three different years from each country, but not the same years and the rates vary widely by country). Any graph is better than nothing at this point, just to get some idea of the general overall trend. Thanks. 71.212.255.94 (talk) 03:31, 8 March 2013 (UTC)[reply]

Can the U.S. President order an ordinary person to do something?

Our son asked: "What if the President came in and told you that you have to eat salads?" This raised an interesting question: can the President in fact order a person that doesn't report to him to do something (or rather, does the person have to obey, like they generally would have to obey a policeman that asks to, say, move from a particular spot)? This question assumes that no law or executive order has been at point issued to any particular effect. Obviously, there are things a President can do that will affect you, e.g., by coming to your house, he'll subject you to high-level security perhaps without your permission, but the question is about a more direct type of communication, e.g., "Mr. Wales, I order you to eat this cupcake!".Knyazhna (talk) 03:41, 8 March 2013 (UTC)[reply]

No, the President can't order a particular person to do something. The extent to which he can compel all Americans to do something is questionable. There are some things which can be done by executive order, but they are generally quite limited. Most actions would require the approval of Congress, and perhaps the Supremes. StuRat (talk) 04:06, 8 March 2013 (UTC)[reply]


No, he cannot. The President doesn't rule or reign in the U.S., he presides. He does not set law, he cannot enforce his will upon you. That's what having a citizen president means. As John Adams once famously put it, the U.S. has a "government of laws and not of men". The President's duties and obligations are outlined in the U.S. constitution, specifically Article Two of the United States Constitution which does not contain the right to order other citizens around at his whim. Additional commentary by the writers of the Constituion on the scope of the President's powers and duties are in the Federalist Papers, specifically #41-43 which covers the powers of the Federal Government in general, and #67-77 which covers the executive branch and its powers in specific details. --Jayron32 05:08, 8 March 2013 (UTC)[reply]
I think "whim" is the wrong word. The president may have what appears to be a good reason to him (e.g. national security). Would he still not be able to order? I am sure the founding fathers thought he couldn't, but nowadays is that really true? Ornil (talk) 05:27, 8 March 2013 (UTC)[reply]
Even in some national security crisis, I don't think the president would have the constitutional backing to order civilians around without the backing of congress. The president can do some things with executive orders, but he cant just walk up to you and start ordering you around. Hope that helps RetroLord 10:46, 8 March 2013 (UTC)[reply]

How many teachers of English as a foreign language are there?

Implicit in my question: all varieties of the English language; in the world, not just wherever I'm from or wherever you are from; people who would self-identify as teachers but not necessarily professional. Thank you! — Preceding unsigned comment added by 60.43.33.86 (talk) 05:13, 8 March 2013 (UTC)[reply]

I decided to ask this here because I do not believe people will view the talk page for the article. Multiple news sources say that North Korea won't fulfill it's peace treaties and non-agression pact anymore. I also included a report from two days ago that speculated about them canceling the armistice. My question is if the armistice is one of the treaties cancelled and if it is, can that be conveyed in the article properly? --Thebirdlover (talk) 05:15, 8 March 2013 (UTC)[reply]

Adam Smith

Hi! I was reading our article on Adam Smith, but i'm still not too sure on his political ideals, could anyone here state what his political ideas were or if he came up with any? All I can find as of now is some things about Political Economy, am I missing anything? Thanks RetroLord 10:44, 8 March 2013 (UTC)[reply]

He wrote a whole book on morality, which contains ideas relevant to politics, and a multi-volume book on political economy, which contains a great many opinions on political issues. If you start reading The Wealth of Nations you will see how he has a view on many social issues of his time. In fact the book is largely about social policy. He critiques what he calls "the policy of Europe", especially England, where restrictive government legislation interfered with business, and he thinks, also interfered with people's freedom and ability to find work. Itsmejudith (talk) 12:14, 8 March 2013 (UTC)[reply]

what are the causes of the rebellion?

what are the causes that led to the rebellion and the rebel alliance? what could the empire have done to prevent this? 91.120.48.242 (talk) 12:52, 8 March 2013 (UTC)[reply]